You are on page 1of 240

(i)

ENGLISH GUIDE & WORK BOOK


(The Ultimate Guidance for Success)

10
REVISED ENRICHED EDITION 2020-21

1. Drill Book
2. IDEAL Question Bank
Salient Features:
Detailed Summary for Prose, Poetry and Supplementary Reader.
Exhaustive Additional Answers for all types.
Exhaustive Exercises for Self-Evaluation.
Exhaustive PQTs and Grammar Exercises and many more.....

®
SURYA PUBLICATIONS
(A unit of Shyamala Group)

Corporate Office: Registered Office:


No. 1, Sugar Mill Colony, New No. 59, 4th Avenue,
Salai Kumaran illam, Madurai Road, Opp. to Govt. Boys Hr. Sec. School,
Tirunelveli - 627 001. Ashok Nagar, Chennai - 600 083.
Phone : 0462 - 2338899, 2338484 Ph : 044 - 24744484
Mobile : 94431-58484, 95978-39822 Mobile: 94421-58484, 94425-58484
E-mail : suryaguides@yahoo.com E-mail: srigangapublications5@gmail.com
website : www.suryapublications.in

Price ` 300/-
(ii)

Published By
B. ARUMUGAM
© SURYA PUBLICATIONS
(A unit of Shyamala Group)

Authors

G. GEOFFREY DAVIDSON A. GOPAL, M.A., M.Ed., M.Phil.,


Salem. Salem.

T. BACKIYARAJ, A. AROCKIA DOSS,


M.A.(Eng), B.Ed., M.Phil., PG Dip. in ELT M.A.(Eng), M.A. (His), B.Ed.,
Thanjavur. Lecturer in English, Puducherry.

R.C. FLORENCE, M.A. B.Ed. V. REUBAN XAVIER, M.A., M.Phil.,


Cuddalore. Asst. Prof. in English, Tuticorin.

OUR BANK DETAILS

Account Name :4 SURYA PUBLICATIONS


Account Number :4 446971431
Bank Name :4 INDIAN BANK
IFSC Code :4 IDIB000T034
Branch Name :4 TIRUNELVELI JUNCTION

Account Name : SURYA PUBLICATIONS


Account Number : 510909010051752
Bank Name : CITY UNION BANK
IFSC Code : CIUB0000230
Branch Name : PALAYAMKOTTAI

CHENNAI ACCOUNT
Account Name : SRI GANGA PUBLICATION
Account Number : 928507483
Bank Name : INDIAN BANK
IFSC Code : IDIB000A031
Branch Name : ASHOK NAGAR

Account Name : SRI GANGA PUBLICATION


Account Number : 512020010022514
Bank Name : CITY UNION BANK
IFSC Code : CIUB0000230
Branch Name : PALAYAMKOTTAI
(iii)

FOREWORD
FOREWORD

Tamil Nadu Government has made an astronomical stride in facilitating the students
to acquire language skills, the need of the hour by publishing ‘The English Reader for Class X
in 2020. The venture is unique in the sense that all the four language skills have been adroitly
integrated in each unit so that students’ linguistic competencies improve to stupendous level to
match the global standard. It is fully revised updated and enriched.

In accordance with the learning objectives as formulated in the new syllabus, SURYA
Publications of is coming up with a New Guide and Work Book that is student-centric
and teacher-friendly in all aspects.

The Salient Features of Surya’s Guide and Work Book:

 Detailed Summary for Prose, Poetry and Supplementary Reader.


 Exhaustive Additional Meanings, Synonyms and Antonyms.
 Exhaustive Additional Short Answers, Paragraph Answers for Prose, Poetry and Supplementary
Reader.
 Adequate Exercises for Writing Skills, Language Functions and Non-Textual Competencies.
 Exhaustive Exercises with Answers and Exercises for Self-Evaluation for PQT and Grammar.

It is earnestly hoped that SURYA’s Guide and Work Book for Class X
will be of immense help for the all kinds of learners seeking Academic Excellence to face Public
Examination confidently and score top marks in the tests and exams.

SURYA Publishers wish you a pleasant and fruitful academic year embellished
with Astounding Academic Achievements, earnestly looking forward to your valuable suggestions

and feedback.

Publisher
(iv)

10
ENGLISH GUIDE & WORK BOOK
Unit Contents Page No.

Memoriter vi - viii

Prose His First Flight 1 - 23

1 Poem Life 24 - 28

Supplementary The Tempest 29 - 35

Prose The Night the Ghost Got in 36 - 55

2 Poem The Grumble Family 56 - 61

Supplementary Zigzag 62 - 69

Prose 70 - 93

3 Poem I am Every Woman 94 - 98

Supplementary The Story of Mulan 99 - 104

Prose The Attic 105 - 130

4 Poem The Ant and the Cricket 131 - 137

Supplementary The Aged Mother 138 - 145

Prose Tech Bloomers 146 - 164

5 Poem The Secret of the Machines 165 - 169

Supplementary A day in 2889 of an American Journalist 170 - 175

Prose The Last Lesson 176 - 194

6 Poem No Men Are Foreign 195 - 198

Supplementary The Little Hero of Holland 199 - 203

Prose The Dying Detective 204 - 221

7 Poem The House on Elm Street 222 - 225

Supplementary A Dilemma 226 - 232


ENGLISH

10
Memoriter
LIFE
- Henry Van Dyke

MEMORITER
Let me but live my life from year to year,

With forward face and unreluctant soul;

Not hurrying to, nor turning from the goal;

Not mourning for the things that disappear

In the dim past, nor holding back in fear

And happy heart, that pays its toll

To Youth and Age, and travels on with cheer.

So let the way wind up the hill or down,

O’er rough or smooth, the journey will be joy:

POEM
Still seeking what I sought when but a boy,

New friendship, high adventure, and a crown,

My heart will keep the courage of the quest,

And hope the road’s last turn will be the best.

v
ENGLISH

I am Every Woman
Rakhi Nariani Shirke

A woman is beauty innate,

A symbol of power and strength.

She puts her life at stake,


MEMORITER

She’s real, she’s not fake!

The summer of life she’s ready to see in spring.

She says, “Spring will come again, my dear.

Let me care for the ones who’re near.”

Memoriter
Strong is she in her faith and beliefs.

“Persistence is the key to everything,”

says she. Despite the sighs and groans and moans,

She’s strong in her faith, firm in her belief!


POEM

She’s a lioness; don’t mess with her.

She’ll not spare you if you’re a prankster.

Don’t ever try to saw her pride, her self-respect.

She knows how to thaw you, saw you – so beware!

She’s today’s woman. Today’s woman, dear.

Love her, respect her, keep her near...

vi
ENGLISH

THE SECRET OF THE MACHINES 10


Rudyard Kipling

We were taken from the ore-bed and the mine,


We were melted in the furnace and the pit
We were cast and wrought and hammered to design,

MEMORITER
We were cut and filed and tooled and gauged to fit.

Some water, coal, and oil is all we ask,


And a thousandth of an inch to give us play:
And now, if you will set us to our task,
We will serve you four and twenty hours a day!

We can pull and haul and push and lift and drive,
We can print and plough and weave and heat and light,
We can run and race and swim and fly and dive,
We can see and hear and count and read and write!

POEM
But remember, please, the Law by which we live,
We are not built to comprehend a lie,
We can neither love nor pity nor forgive,
If you make a slip in handling us you die!

Though our smoke may hide the Heavens from your eyes,
It will vanish and the stars will shine again,
Because, for all our power and weight and size,
We are nothing more than children of your brain!
vii
ENGLISH

No Men Are Foreign


James Falconer Kirkup

Remember, no men are strange, no countries foreign

Beneath all uniforms, a single body breathes


MEMORITER

Like ours: the land our brothers walk upon

Is earth like this, in which we all shall lie.

They, too, aware of sun and air and water,

Are fed by peaceful harvests, by war’s long winter starv’d.

Their hands are ours, and in their lines we read

Remember they have eyes like ours that wake

Memoriter
Or sleep, and strength that can be won

By love. In every land is common life

That all can recognise and understand.

Let us remember, whenever we are told


POEM

To hate our brothers, it is ourselves

That we shall dispossess, betray, condemn.

Remember, we who take arms against each other

It is the human earth that we defile.

Our hells of fire and dust outrage the innocence

Of air that is everywhere our own,

Remember, no men are foreign, and no countries strange.

viii
ENGLISH

UNIT

1
PROSE
10
His First Flight
- Liam O’Flaherty

B.P.No. 1
WARM UP

His First Flight


Marco Polo, Christopher Columbus, Ibn Battuta, Amerigo Vespucci, Xuanzang, Ferdinand
Magellan, Bartolomeu Dias, Herodotus, Captain James Cook, Vasco De Gama.

What is common among these individuals? Browse Internet or refer books and share some
information about them.

Marco Polo
Marco Polo was one of the first and most famous Europeans to travel to Asia during the Middle Ages.
He travelled farther than any of his predecessors.
Christopher Columbus
d four voyages
across the Atlantic Ocean under the auspices of the Catholic Monarchs of Spain. Columbus discovered
the viable sailing route to America.
Ibn Battuta
Ibn Battuta was a Moroccan Muslim scholar and traveller. He was known for his travelling and
undertaking excursions called the Rihla.

Amerigo Vespucci
Amerigo Vespucci was an Italian explorer, financier, navigator, and cartographer born in the Republic of
Florence. He embarked on his first voyage.

Prose
Xuanzang
Xuanzang was a Chinese Buddhist monk, who travelled to India in the seventh century and described
the interaction between Chinese Buddhism and Indian Buddhism during the early Tang dynasty.
Ferdinand Magellan
Ferdinand Magellan was a Portuguese explorer. In search of fame and fortune, Ferdinand Magellan set
out from Spain in 1519 with a fleet of five ships to discover a western sea route to the Spice Islands.
UNIT 1

Bartolomeu Diasy
Bartolomeu Diasy was a Portuguese explorer. Dias was the first European during the Age of Discovery
to anchor at what is present-day South Africa.

Captain James Cook


Captain James Cook was a naval captain, navigator and explorer who, in 1770, discovered and charted
New Zealand.
1
ENGLISH

Herodotus
Herodotus was an ancient Greek historian. He was born in the Persian Empire (Modern Turkey). He is
called “The Father of History”.

10 Vasco da Gama
Vasco da Gama was a Portuguese explorer. His initial voyage to India was the first to link Europe and
Asia by an ocean route.

Discuss, why it was considered as an adventurous and dangerous thing to travel in those
days
It was considered adventurous and dangerous to travel in those days, because at the time nobody
His First Flight

ventured into this sort of travel due to the unpredictable weather conditions lack of knowledge of
directions to landmarks, etc.

Mahatma Gandhi, Che Guevera, Guru Nanak and Gauthama Buddha are some of the famous
personalities and thinkers who made extensive journeys to understand life. Do you think
journeys can be life changing?
• As it is said travel is 20% adventure and 80% self-discovery, I think the great personalities
like Mahatma Gandhi, Che Guevera, Guru Nanak and Gauthama Buddha have shaped up their
personalities in many ways due to the extensive journeys they made.
• When Gandhi returned to India at the age of 46, he started travelling widely among the country,
oveable throughout
the country.
• A journey of nine months immensely changed Che Guevera’s views on politics, people and himself
and transformed him into a revolutionary.
• Guru Nanak travelled widely in his life time and went as far as Assam in the east, Sri Lanka in
the south, Tibet in the north and Mecca and Baghdad in the west. During his 24 year travels, he
founded a new faith, which is today being followed by the Sikhs.
• Gauthama Buddha came to know about the sufferings of the people and philosophy and irony of
life when he travelled out of the palace, which ended in his enlightenment under the Bodhi tree.
Prose

After that for long 45 years Buddha travelled with his disciples to preach his doctrines.
• Because of their extensive journess, their life got changed. So, I think, journeys can be life
changing.

Share a memorable trip from your life. Have you ever been on an adventurous journey? If
yes, share your experience.
When I was studying Class IX, my classmates and I were taken for a picnic to a zoo in the
thick of the forest. First we were thrilled to see the mighty trees and greenery. After that we saw a
UNIT 1

python which was kept in a glassed cave. Alongside, we saw many kinds of snakes. Seeing this, all my
friends started screaming. When we turned to the left, there was a passage which led to a fenced area,
where we saw elephants, leopard, lion, tiger, boar, hippopotamus and so on. Each one of them was put
up in a separate enclosure. It was very frightening to see the lion. But when we kept on watching over
and again more fiery animals, the fear of seeing them diminished. This was a memorable experience
for me.
2
ENGLISH

“That’s one small step for a man, one giant leap for mankind.” Neil Armstrong - Discuss.
• These were the first words uttered by Neil Armstrong when he landed on the lunar surface-moon.
• These words probably mean that the mission to the moon is a step for a man such as him at a
personal level. But this is an unprecedented achievement for the whole mankind. That is, this sort
10
of mission never took place in the past before his. So Armstrong said these words.

Have you ever seen a bird making its first ever attempt to fly?
Yes, I have seen a bird making its first attempt to fly, after a few days of its birth.

SUMMARY

His First Flight


‘His First Flight’ was a short story written by Liam O’ Flaherty who was an Irish novelist
and short story writer. It is a story about the need for courage and self-confidence to overcome fears
in life. Every journey of a thousand miles begins with but a single step. But that single step is the most
important and difficult one for all us to make. It is about a young seagull which is trying to take its first
flight.
The young seagull looked down desperately at the vast expanse of sea that stretched
down beneath his ledge. He was afraid to take its first flight. His parents had flown away along with his
brothers and his little sister, leaving him alone on the rock without food. They could all fly because they
believed in the power of wings. But he did not realise his power. So he could not fly. He had tried many
id. Though his
parents had tried numerous times to support him to fly, he thought that his wings would not support
him. He was terribly scared.
He was starving and felt that he would die if he did not get any food. He saw his mother
tearing a piece of fish with her beak. When he cried out to her she just screamed back mockingly. The
sight of food maddened him. He cried at her but she just screamed back mockingly. Suddenly, he felt
the joy, seeing his mother approaching him with food. But she stopped opposite him, keeping the fish
just out of his reach. Maddened by hunger, he dived at the fish. But his mother flew upward and he
started falling. He was terrified for a moment but then he opened his wings and realized that he was

Prose
flying. Then he knew the fact that he was born to fly. To the delight and surprise of his entire family,
he had made his first flight.

B.P.No. 5
GLOSSARY
ledge (n) – a narrow shelf that juts out from a vertical surface
shrilly (adv.) – producing a high-pitched and piercing voice or sound
herring (n) – a long silver fish that swims in large groups in the sea
UNIT 1

devour (v) – to eat something eagerly and in large amounts, so that nothing is left
cackle (n) – a sharp, broken noise or cry of a hen, goose or seagull
mackerel (n) – a sea fish with a strong taste, often used as food
gnaw (v) – to bite or chew something repeatedly
trot (v) – to run at a moderate pace with short steps
precipice (n) – a very steep side of a cliff or a mountain 3
ENGLISH

preening (v) – cleaning feathers with beak


whet (v) – to sharpen
plaintively (adv.) – sadly, calling in a sad way
10 swoop (v) – to move very quickly and easily through the air
beckoning (v) – making a gesture with the hand or head to encourage someone to
approach or follow.

ADDITIONAL
His First Flight

flap – move dozing – sleeping


expanse – stretch
certain – sure uttered – said (here made)
muster – gather abreast – in line with
plunge – dive halted – stopped
desperate – worried swooped – moved quickly
monstrous – great
threatening – frightening
seized – caught
devour – consume, eat
ascending – rising soaring – climbing
blazing – brightly, strongly commenced – started
pily
gnawed – chewed fright – fear
exhausted – tired
trotted – ran
daintily – delicately mockingly – teasingly
sheer – steep limp – stiff
dizzy – giddy
pretended – acted
beckoning – signalling, gesturing
plateau – large area of flat and
scraps – pieces
higher land
Prose

praising – admiring

EXERCISE WITH ANSWER

Choose the suitable synonym of the word underlined.

1. He had taken a little run forward to the brink of the ledge.


UNIT 1

a) floor b) edge c) surface d) nest

2. He felt certain that his wings would never support him.


a) sure b) doubtful c) secure d) happy

3. His father and mother had come around calling to him shrilly.
a) softly b) with low voice c) high pitch sound d) sadly
4
ENGLISH

4. The son was now ascending the sky blazing warmly on his ledge.
a) bluntly b) dimly c) hoping d) brightly

5. He had searched every inch, rooting among the rough nest.


a) firm stand b) resting c) sleeping d) low grip 10
6. He then trotted back and forth from one end of the ledge to the other.
a) talked b) laughed c) spotted d) ran

7. After a minute or so he uttered a joyful scream.


a) fell b) laughed c) made d) jumped
His First Flight

8. When she was just opposite to him, abreast of the ledge she halted.
a) following b) in line with c) far from d) without touching

9. He was soaring gradually.


a) climbing b) falling c) telling d) laughing

10. He turned his beak sideways and crowed amusedly.


a) sadly b) slowly c) fast d) happily

Answers
d

Choose the suitable synonym of the word underlined.

1. He could not rise again exhausted by the strange exercise.


a) excited b) enjoyed c) tired d) energised Ans:

2. When she was just opposite to him, abreast of the ledge she halted.

Prose
a) stopped b) hailed c) informed d) fell Ans:

3. His long gray legs stepped daintily, trying to find some means of reaching his parents.
a) strongly b) slowly c) hardly d) delicately Ans:

4. The sun was now ascending the sky blazing warmly on his ledge.
a) falling b) rising c) kicking d) telling Ans:

5. He even gnawed at the dried pieces of eggshell.


a) broke b) looked c) made d) chewed Ans:
UNIT 1

6. His father and mother scolded him, threatening to let him starve.
a) frightening b) lowering c) bending d) seeing Ans:

7. The young seagull was alone on his ledge.


a) flat b) shelf c) roof d) garden Ans:
5
ENGLISH

8. He ran forward to the brink of the ledge and attempted to flap his wings.
a) move b) run c) write d) play Ans:

9. He failed to muster up courage to take that plunge which appeared to him so desperate.
10 a) understand b) inform c) train d) gather Ans:

10. He failed to muster up courage to take that plunge which appeared to him so desperate.
a) open b) dive c) close d) talk Ans:

11. He failed to muster up courage to take that plunge which appeared to him so desperate.
a) happy b) strong c) worried d) joy Ans:
His First Flight

12. He saw his brothers and his sister lying on the plateau dozing.
a) making b) cursing c) sleeping d) digging Ans:

13. The great expanse of sea stretched down beneath.


a) stretch b) expenditure c) knowledge d) failure Ans:

14. The great expanse of sea stretched down beneath.


a) far b) below c) above d) over Ans:

15. He had in fact seen his older brother catch his first herring and devour it.
a) keep b) give c) throw d) eat Ans:

a) cloud b) flat c) cliff d) ocean Ans:

17. The precipice was sheer, and the top of it was farther away.
a) steep b) clear c) rough d) over Ans:

18. He closed one eye then the other and pretended to be falling asleep.
a) shouted b) acted c) informed d) closed Ans:

19. His father was preening the feathers on his white back.
a) plucking b) touching c) wetting d) cleaning Ans:
Prose

20. She screamed back mockingly.


a) insultingly b) happily c) sadly d) coolly Ans:

21. But he kept calling plaintively.


a) happily b) sadly c) coolly d) hardly Ans:

22. His mother had swooped upwards.


a) moved downward b) moved sideways c) moved slowly d) moved quickly Ans:
UNIT 1

23. Then a monstrous terror seized him and his heart stood still.
a) taught b) informed c) caught d) threw Ans:

24. Then he commenced to dive and soar.


a) started b) stopped c) left d) kicked Ans:
6
ENGLISH

25. He screamed with fright and attempted to rise again.


a) wings b) fear c) happiness d) laughter Ans:

ANTONYMS 10
young × old daintily × heavily
afraid × bold slowly × fast
expanse × limited hidden × exposed, open
beneath × above dozing × awake
His First Flight

certain × doubtful tore × attached


bent × straighten mockingly × encouragingly
desperate × confident gradually × suddenly
threatening × safeguard commenced × ended
cowardice × bravery amusedly × uninterestingly
ascending × descending fright × fearlessness
blazing × dim exhausted × energetic
dried × fresh

EXERCISE WITH ANSWER

Choose the suitable antonyms of the word underlined.

1. The young seagull was alone on his ledge.


a) old b) friend c) next d) neighbour

2. So bent his head and ran away back to the little hole under the ledge.
a) folded b) hid c) saw d) straightened

Prose
3. The sun was now ascending the sky blazing warmly on the edge.
a) shinning b) bright c) dimming d) talking

4. He stepped slowly out to the brink of the ledge.


a) mildly b) fast c) fastly d) hardly

5. Now and again, she tore at apiece of fish that lay at her feet.
a) made b) pulled c) pushed d) attached
UNIT 1

6. He screamed with fright and attempted to rise again.


a) fearlessness b) flight c) findings d) fearful

7. He turned his beak sideways and crowed amusedly.


a) happily b) uninterestingly c) interestingly d) thinking

Answers
1. a 2. d 3. c 4. b 5. d 6. a 7. b 7
ENGLISH

EXERCISE for SELF EVALUATION

10 Choose the suitable antonyms of the word underlined.

1. He had been afraid to fly with them.


a) fear b) bold c) ready d) willing Ans :

2. There he had found a dried piece of mackerel.


a) fresh b) drained c) withered d) denied Ans :
His First Flight

3. His father and mother had come around scolding him.


a) frightening b) shouting c) praising d) requesting Ans :

4. He saw his brothers lying on the plateau dozing.


a) sleeping b) talking c) cleaning d) awake Ans :

5. The great expanse of sea stretched down beneath.


a) limited b) stretch c) flat d) sight Ans :

6. The great expanse of sea stretched down beneath.


a) below b) fall c) steep d) above Ans :

wardice.
a) fearfulness b) weakness c) bravery d) timidity Ans :

8. He felt certain that his wings would never support him.


a) sure b) doubtful c) happy d) sad Ans :

9. His father and mother had come around threatening to let him starve.
a) safeguard b) frightening c) aggressive d) nasty Ans :

10. The sun was now ascending the sky blazing warmly on his ledge.
a) raising b) rising c) descending d) accepting Ans :
Prose

11. He stood on one leg with the other leg hidden under his wings.
a) kept b) exposed c) concealed d) unseen Ans :

12. He screamed back mockingly.


a) interestingly b) laughingly c) insultingly d) encouragingly Ans :

13. He was soaring gradually.


a) suddenly b) slowly c) regularly d) cunningly Ans :
UNIT 1

14. He could not rise, exhausted by the strange exercise.


a) tired b) fatigued c) energetic d) high Ans :

15. Then he commenced to dive and soar.


a) started b) ended c) talked d) kept Ans :

8
ENGLISH

TEXTUAL QUESTIONS and ANSWERS

a. Why did the seagull fail to fly? B.P.No. 2 10


The seagull was afraid to fly. He could not muster up courage to take a dive. So it failed to fly.

b. What did the parents do, when the young seagull failed to fly?
The parents called him. They scolded him and threatened him to let him starve on the ledge.

c. What was the first catch of the young seagull’s older brother? B.P.No. 3
His First Flight

The first catch of the young seagull’s older brother was a herring.

d. What did the young seagull manage to find in his search for food on the ledge?
The young seagull managed to find a dried piece of mackerel’s tail at the end of the ledge.

e. What did the young bird do to seek the attention of his parents? B.P.No. 4

The young bird uttered a low crackle to seek the attention of his parents

f. What made the young seagull go mad?


The sight of the food at the feet of his mother made him go mad.

g. Why did the young bird utter a joyful scream?


The young bird uttered a joyful scream because he thought his mother would give him food.

h.
No, the mother bird didn’t offer any food to the young bird.

i. How did the bird feel when it started flying for the first time?
The bird felt his wings spread outwards and he felt the tips of his wings cutting through the air. He felt
a bit dizzy.

j. What did the young bird’s family do when he started flying?


The young bird’s mother swooped past him. His father flew over him screaming. His brothers and sister
were flying around him soaring and diving.

Prose
A. Answer the following questions in a sentence or two. B.P.No. 5

1. How was the young seagull’s first attempt to fly?


The young seagull ran forward to the brink of the ledge and attempted to open his wings but was
afraid. He thought his wings would not support him to fly. So he ran back to the hole under the ledge.

2. How did the parents support and encourage the young seagull’s brothers and sister?
The parents were flying with the seagull’s brothers and sister. They taught them how to perfect the art
UNIT 1

of flight and how to skim the waves and dive for fish. When they caught their fish the parents circled
around them making a proud cackle.

3. Give an instance that shows the pathetic condition of the young bird.
No food was left at the ledge and the young bird was very hungry. He went to the extent of gnawing
at the dried pieces of eggshell.
9
ENGLISH

4. How did the bird try to reach its parents without having to fly?
The bird was moving back and forth to reach its parents. He could reach them only when he moved
northwards along the cliff face.

10 5. Do you think that the young seagull’s parents were harsh to him? Why?
No, I don’t think the young seagull’s parents were harsh to him because they were only making him
realise his inborn potential of flying. Perhaps leaving him alone on the ledge might sound they were
harsh, but the result was heartening. That was the only way left to the parents to make the young
bird fly.

6. What prompted the young seagull to fly finally?


His First Flight

Hunger and it’s attempt to get food prompted the young bird to fly finally.

7. What happened to the young seagull when it landed on the green sea?
When the young bird landed on the green sea its feet sank into water and the water touched its belly.
Then it realised that it was floating.

ADDITIONAL

Answer the following questions in a sentence or two.

The young seagull was alone on his ledge because his parents with two brothers and sister had already
flown away the day before. He had been afraid to fly with them.

2. What did the young seagull see his father, brothers and sister doing?
He saw his brothers and sister lying on the plateau and sleeping. His father was cleaning his feathers.

3. What did the mother do as the final attempt to make the young bird fly?
The mother left the young bird to starve on the ledge. When the bird was hungry the mother flew near
him with food on its beak. Now the young bird had to dive to get the food
Prose

B. Answer each of the following questions in a paragraph of about 100-150 words.


 B.P.No. 6

1. Describe the struggles undergone by the young seagull to overcome its fear of flying.

Lesson His First Flight


Author Liam O’ Flaherty
Theme Realisation of one’s potential
UNIT 1

The will to win, the desire to succeed, the urge to reach your full
potential are the keys that will unlock the door to personal excellence.

The struggles that the seagull underwent to overcome its fear of flying make everyone feel
sympathy for him. To begin with, he was afraid of flying out of the ledge, for he feared he would fall
headlong into the water and die. When he refused to accompany his family, he was threatened to be
10
ENGLISH

left alone on the ledge and they did so. As a result, he was alone on his ledge. As he had not eaten
since the previous nightfall, he was hungry. Hence he was forced to eat a dried piece of mackerel’s tail
at the far end of his ledge. After that, there was not a single scrap of food left on the ledge. He even
started biting the dried pieces of eggshell. He walked back and forth from one end of the ledge to
the other desperately trying to find some means of reaching his parents without having to fly. He even
10
pretended to be falling asleep standing on the edge of the ledge. He waited a moment in surprise,
wondering why she did not come nearer, and there maddened by hunger, he dived at the fish. With
a loud scream, he fell outwards and downwards into space. Still they took no notice of him. All these
incidents state that he underwent indescribable struggles. But hunger stirred him to dive and he
realised that he could fly with his wings.
His First Flight

One isn’t necessarily born with courage, but one is born with potential.

2. Your parents sometimes behave like the young bird’s parents. They may seem cruel and
unrelenting. Does it mean that they do not care for you? Explain your views about it with
reference to the story.

Lesson His First Flight


Author Liam O’ Flaherty
Theme Realisation of one’s potential

Yes, my parents sometimes behave like the young bird’s parents, being cruel and unrelenting.
thing good to
me. As my parents have more experience in life, I would first express my feelings or views. If they
still are unrelenting, I would sit up with them and try to understand what they are trying to do for
me. As a teen, my feeling would be to follow what I feel, but at the same time I would try to give a
second thought about it. And I know one thing for sure that my parents always think of my wellbeing.
Even if certain things are risky or scary, I will follow my parents’ words. I know that if the seagull had
listened to his parents earlier, he would have gone along with his parents for perfecting the art of flight,
catching and tearing fish, sharpening beak and so on. He did not understand his inborn potential that
he was born to fly. I would certainly listen to my parents’ words and do what they tell me to do.

Prose
ADDITIONAL PARAGRAPH

Answer the following question in a paragraph of about 100-150 words.


1. Describe the efforts taken by the mother to prompt the young seagull to fly.

Lesson His First Flight


Author Liam O’ Flaherty
UNIT 1

Theme Realisation of one’s potential

There is no heavier burden than an unfulfilled potential

The seagull’s mother was very much worried about her son’s inhibition about flying. When the
seagull was afraid of flying the mother had a good plan. She took the whole family to the plateau,
scolding him and threatening to let him starve if he did not fly with them. The mother was teaching 11
ENGLISH

her children the art of skimming and diving at his attention but she kept on looking at his movements
on the ledge. She was trying to do all sorts of things to attract and tempt him to fly. She tore at a
piece of fish in order to attract his attention. She kept the food in her beak and took it near the ledge.

10 She suspended herself opposite him. When the young seagull saw this he dived at the fish and at the
same time she swooped upwards. But when she was just opposite to him, abreast of the ledge, she
halted, her legs hanging limp, her wings motionless the piece of fish in her beak almost within reach
of his beak. He fell outwards and downwards and his wings spread to fly. Thus the mother’s attempt
was successful.

Great work is done by people who are not afraid to be great.


His First Flight

B.P.No. 6
VOCABULARY

Read the following sentences.

Set : 1
1. The young seagull uttered a joyful scream. (adjective)
2. The young seagull screamed with joy. (noun)
3. The young seagull screamed joyfully. (adverb)

1. The young bird pretended to be falling asleep. (verb)


2. The young bird made a pretension of falling asleep. (noun)
3. The young bird made a pretentious posture of falling asleep. (adjective)

C. Change the parts of speech of the given words in the chart.


Noun Verb Adjective Adverb
exhaustion exhaust exhaustive exhaustively
Prose

width widen wide widely


madness madden mad madly
perfection perfect perfect perfectly

D. Read the following sentences and change the form of the underlined words as directed.
1. His family was screaming and offering him food. (to adjective)
His screaming family was offering him food.
UNIT 1

2. The young seagull gave out a loud call. (to adverb)


The young seagull called out loudly.
3. The bird cackled amusedly while flying. (to noun)
The bird cackled in amusement while flying.
4. The depth of the sea from the ledge scared the seagull. (to adjective)
12 The sea was deep from the ledge and it scred the seagull.
ENGLISH

5. The successful flight of the bird was a proud moment for the seagull’s family. (to verb)
The bird flew successfully, which was a proud moment for the seagull’s family.

E. Use the following words to construct meaningful sentences on your own.


10
1. coward - Peter cannot swim across the river because he is a real coward
2. gradual - Rita is progress in studies was gradual.
3. praise - We should praise God for all His blessings.
4. courageous - Rajesh is a courageous boy in the class.
5. starvation - Everyone should work hard to avoid starvation in the family.
His First Flight

B.P.No. 7
LISTENING

F. Here is a travelogue by the students of Government Girls Higher Secondary School,


Pattukkottai after their trip to Darjeeling. Listen to the travelogue and answer the
following questions.

i) Fill in the blanks with suitable words.


1. The students visited _________ city. Ans: Darjeeling
2. _________ is the third highest mountain in the world. Ans: Kanchenjunga

4. The drinking water is supplied by _________ lake to the city. Ans: Senchal
5. After Senchal lake, they visited _________ . Ans: Batasia Loop

ii) Do you think they had a memorable and enjoyable school trip?
Yes, they did have a memorable and enjoyable school trip, which is very clear from their excitement
to look of one thing to another.

iii) Name a few places that you wish to visit with your classmates as a school trip.
We would like to visit Mysore Palace, Delhi, Agra and Kochi.

Prose
iv) State whether the following statements are True or False.
1. As the sky was cloudy, they could get the glimpse of the Mount Everest. Ans: False
2. The toy train covers 14 km in three hours. Ans: True
3. Tiger hill has earned international fame for the best sunset view. Ans: True

B.P.No. 7, 8
SPEAKING
UNIT 1

G. Here is a dialogue between a father and his daughter. Continue the dialogue with at
least five utterances and use all the clues given above.
Father : Hi Mary, it has been a very long time since we went on a trip. Let’s plan one.
Mary : Yes, dad. I am also longing to go. Why don’t we plan one for this weekend?
Father : Sure. Tell me, where shall we go?
13
Mary : Some place nearby but for at least two days.
ENGLISH

Father : Hmm… I think we should go to the reserved forest nearby.


Mary : Yeah. I’ve never been to a forest. I have seen it only on the TV and movies. A forest is a
good choice!

10 Father : OK. If we are going to the forest, we must list out what we should carry with us for two
days.
Mary : I think we should carry suitable clothes like, sweater, muffler, scarf and blankets.
Father : What about the food? Do you have any idea, Mary?
Mary : Yeah, For food, I suggest , chappathi, puliotharai and bread and jam.
Father : Yeah, that is a good idea. Then, what are the places you suggest we visit?
Mary : I suggest we visit the waterfalls and streams which are deep into the forest. There is an old
His First Flight

temple, which was built in 18th century and there is a place where elephants pass by.
Father : But how can we reach there?
Mary : First we need to get permission from the forest department. If that’s done, we needn’t
worry about our travel. They will take us by Forest Dept. vehicles.
Father : Mary, what kind of clothes do you suggest we should take with us?
Mary : It’s biting cold at night. So we should take sweaters, mufflers, bed sheets, towels and three
sets of clothes.
Father : Need we take boots?
Mary : Of course, dad. It’s only a footpath at most of the places.

Mary :  Yes, one important thing, we should take First-aid kit and required medicine for any emergency.
Father : That’s nice of you, Mary.
Mary : Thanks, dad.

B.P.No. 8, 9
READING

H. Read the following passage and answer the questions that follow.
Prose

BUNGEE - JUMPING
Bungee jumping is an activity that involves jumping from a tall structure while connected to a
long elastic cord. The tall structure is usually a fixed object, such as a building, bridge or crane; but
it is also possible to jump from a movable object, such as a hot-air-balloon or helicopter, that has the
ability to hover above the ground. The thrill comes from the free-falling and the rebound. When the
person jumps, the cord stretches and the jumper flies upwards again as the cord recoils, and continues
to oscillate up and down until all the kinetic energy is dissipated.
Jumping Heights, located in Mohan Chatti village, in Rishikesh has been rated as one of the
UNIT 1

most preferred bungee jumping destinations in India at a height of 83 meters. It is the only place
in India where bungee jumping can be done from a fixed platform. This is also India’s only fixed
platform Bungee- performed from a professional cantilever, to separate it from entertainment parks,
and create instead, an extreme adventure zone. The Bungee has been designed by David Allardice of
New Zealand.
The Cantilever platform is built over a rocky cliff over-looking the river Hall, a tributary of River
14
Ganges. Bungee-ing amidst the vastness of nature lends an an absolutely breath taking experience.
ENGLISH

Jumping Heights is well known for its safety measures and experienced staff. It costs around Rs 2500
per jump, a bit expensive, but totally worth the experience. The Bungee jumping experience has been
set amidst the astoundingly stunning landscape of Rishikesh. To Bungee jump, one must be at least 12
years and should weigh between 40-110 kg.
10
Questions:

1. What is Bungee Jumping?


Bungee Jumping is an activity that involves jumping from a tall structure while connected to a long
elastic cord.

2. Can Bungee be performed from a movable object? How?


His First Flight

Yes, it can be performed from a movable object such as a hot-air-balloon or helicopter, that has the
ability to hover above the ground.

3. When do you think Bungee becomes thrilling?


Bungee becomes thrilling from the free-falling and the rebound.

4. What is the experience when one falls off the platform?


When one falls off the platform, the cord stretches and the jumper flies upwards again as the cord
recoils, and continues to oscillate up and down until all the kinetic energy is dissipated.

5. Where is the Bungee jumping point located in India?

6.
The minimum age to Bumgee jump is that one must be at least 12 years of age and should weigh
between 40-110 kg.

B.P.No. 9
WRITING

Advertisement

Prose
An advertisement is an audio / visual / audio visual form of marketing communication to promote or
sell a product, service or idea.

An advertisement should include the following to make it attractive.


1. Name of the product / brand / outlet.
2. Address with contact information and websites.
3. Appealing Images (visuals) of the Product / Service / Idea to be advertised.
UNIT 1

4. Target demographics / audience / customers.


5. Feel-Good discounts and offers.
6. Colourful Background.
7. Colourful and readable text.
8. Brief and catchy descriptions and benefits about the product.
9. borders and lines to organize. 15
ENGLISH

10
His First Flight

I. Prepare attractive advertisements using the hints given below.


1. Home appliances – Aadi Sale – 20-50% - Special Combo Offers – Muthusamy & Co., Raja

MUTHUSAMY & CO.,


Home Appliances
Sale • Different Models Dis
20% count
Aadi • Exchange Offer -50
%
• Door Delivery

Special Combo Offers


Prose

Raja Street, Gingee.

2. Mobile Galaxy – Smart phones – accessories – SIM cards – Recharge – Free Power banks
on Mobile purchase – No.1, Toll gate, Trichy.

MOBILE GALAXY
* Smart Phones
UNIT 1

* Accessories
EE
FRr Banks * Sim Cards
* Recharge
e
e has
Pow P urc
le
obi
on M
No.1, Toll gate, Trichy.
16
ENGLISH

J. Write a report of the following events in about 100-120 words.

Report Writing
A report is designed to lead people through the information in a structured way, and also to enable
them to find the information that they want quickly and easily. It is a short, sharp, concise document
10
which is written for a particular purpose and audience.
Format of a report.

• Title of the report • Who…? (Chief guest,)


• Report Writer’s name •  ow…? (the details of the event like Program,
H
His First Flight

• What…? (name of the event) its impact etc.)


• When…? (day and time of the event) • Use simple sentences in the Past Tense.
• Where…? (venue of the event) • Be brief.
• Why…? (the purpose of the event) • Do not exaggerate the event.

1. ‘Educational Development Day’ was organized in your school on 15th July. The District
Collector was the Chief Guest of the event. As part of the event, many competitions were
held and the prizes were distributed to the winners and participants. It was a grand and
successful event. Now, as the member of the organizing committee, write a report on the
event in about 120 words.

By Malar, X-A
ABC Higher Secondary School, Tirunelveli. organized Educational Development Day on
July 15, 2020. The motto of the event was to inculcate the zeal for involving oneself for promoting the
mission of educating the poor and aspiring students. As many as 800 students from various schools
across the district took part in the program. The event started off with prayer by Naresh, a Class X
student. The Headmaster delivered the welcome speech. The occasion was inaugurated by the Chief
Guest, the Honourable District Collector. As part of the event, many competitions relating to education
and its benefits were conducted for the students. There was a questionnaire session, wherein the
participants asked several questions relating to education and educational opportunities. The winners

Prose
and runners-up were honoured with citation and trophies and appreciation certificates were given
away to the participants at the Valedictory function. Vote of thanks was proposed by the Principal. It
was indeed a grand and successful event.

2. You are the School Pupil Leader. You have been asked to write a report on the Inaugural
Ceremony of English Literary Association of your school which was held recently. Write a
report on the same in not more than 120 words.
Inaugural Function of English Literary Association
UNIT 1

By Hariharan, SPL
Champion Higher Secondary School, Trichy conducted the inaugural function of English
Literary Association (ELA) on the school premises at 10 A.M. on August 12, 2020. The aim of this
event was to inculcate the zeal to learn and speak English language fluently in our day-to-day life.
The students of classes VI and above took part in the inaugural ceremony. The school invited Prof.
Manickam as the Chief Guest to inaugurate the association. The Chief Guest in his inaugural address
emphasized the importance of English language in our day to day life and for employment opportunity. 17
ENGLISH

Today all jobs require Communicative English as the first prerequisite. Our degrees after our names
do not matter. Hence it has become obligatory that we learn English for various purposes such as
communicating, one’s ideas expressing creativity, seeking jobs and so on, he added. Welcome address

10 was delivered by the Secretary of the ELA, Master Febin. Vote of thanks was proposed by Ms. Divya of
Class X-B. To mark the occasion a lot of literary competitions were held and winners were given away
prizes by the Chief Guest. Also, a book stall was put up near the venue of the event. It was indeed a
memorable event for both the teachers and the students.

3. You are the Coordinator of the Science Forum of your school. An event had been organized
on account of National Science Day for the members of the forum. Now, write a report on
the observation of “National Science Day” at your school.
His First Flight

National Science Day


By Karim, Class X-C-Coordinator of Science Forum
The Government Girls Higher Secondary School, Tuticorin organized The National Science
Day on February 28, 2020 on its school premises, inviting young ISRO scientist Ms. Pooma as the
chief guest. The event started off with prayer in Tamil by a Class X student. The aim of this massive
event was to instill scientific temperament in children at a very young age. The school had invited
neighbouring schools to take part in it. Approximately, 1000 students participated. The Students put
up a lot of cultural programs like skit, song, dance etc., to mark the occasion. The Chief Guest in her
address urged students to cultivate scientific temperament and to study science seriously and enjoy it
from a young age. She also motivated students to become scientists to develop the nation. As part of
-working models
which captivated the viewers’ attention. A quiz contest was conducted and the winners were awarded
medals and certificates. The Headmaster delivered the welcome address’ while a Class X student
proposed a vote of thanks. It was a grand and successful event.

B.P.No. 12
GRAMMAR

Modals
• We have already learnt about Modals in Class IX. Now, let us revise.
Prose

• A modal verb is used to indicate modality (that expresses a speaker’s general intention) i.e.
likelihood, ability, permission, request, capacity, suggestions, order, obligation, advice etc.
• We use modals to show if we believe something is certain, probable or possible.
• Modals are can, could, may, might, will, would, shall, should, must, ought to, need,
used to & dare to.
• Modals do not change with the person or number of the subject.
UNIT 1

A. Complete these sentences using appropriate modals. The clues in the brackets will
help you.
1. When I was a child, I _________ climb trees easily but now I can’t. (ability in the past)
Ans: could
2. I _________ win this singing contest. (determination) Ans: will
18 3. You _________ buy this book. It is worth buying. (advice or suggestion) Ans: should
ENGLISH

4. Poongothai _________ speak several languages. (ability in the present)Ans: can


5. I swear I _________ tell lies again. (promise) Ans: will not
6. My father _________ play badminton in the evenings when he was at college. (past habit)
Ans: used to 10
7. You _________ do as I say! (command) Ans: should/must
8. _________ I have another glass of water? (request) Ans: May / can
9. Sibi has not practised hard but he _________ win the race. (possibility) Ans: may/can
10. We _________ preserve our natural resources. (duty) Ans: should/ought to

His First Flight


B. Rewrite the following sentences by rectifying the errors in the use of modals.
1. Would I have your autograph?
May I have your autograph?
2. I can be fifteen next April.
I will be fifteen next April.
3. Take an umbrella. It should rain later.
Take an Umbrella. It may rain later.
4. The magistrate ordered that he might pay the fine.
The magistrate ordered that he must pay the fine.

Make me a cup of tea, will you?


6. You may speak politely to the elders.
You should/ought to speak politely to the elders.
7. You will get your teeth cleaned at least once a year.
You should/ought to get your teeth cleaned at least once a year.
8. We could grow vegetables in our kitchen garden but we don’t do it now.
We used to grow vegetables in our kitchen garden but we don’t do it now.

Prose
9. Must I get your jacket? The weather is cold.
May/Can I get your jacket? The weather is cold.
10. Could the train be on time?
Will the train be on time?

C. Read the dialogue and fill in the blanks with suitable modals.
Dad : Shall we go out for dinner tonight?
UNIT 1

Charan : Yes, Dad. We shall go to a restaurant where I can have some ice cream.
Dad : OK. Then, I will be home by 7 p.m. Mom and you should be ready by then.
Charan : Sure. We will. My friend told me that there is a magic show nearby. Will you please take
us there?
Dad : We will not have time to go for the magic show, I suppose. If we have enough time left,
we will/may plan.
Charan : By the way, should we inform our gate keeper about our outing? 19
ENGLISH

Dad : Yes, we should so that he will be aware we aren’t at home.


Charan : Shall I call up Mom and tell her about our plan today?
Dad : You ought to. Otherwise, we might be in trouble when she returns home.

10 Charan : Hmm… by the time you come home in the evening, we will be waiting for you. Hope you
won’t be late. Bye.

D. Read the following dialogues and supply appropriate modals.

Student : Can we leave our bags in the class during the break?
Teacher : Yes, you can but arrange them neatly.
His First Flight

Passenger : My child is 6 years old. Do I have to buy him a ticket?


Conductor : Yes, you must. It costs half of the price of an adult ticket.

Vani : Can we go for coffee after the meeting?


Yoga : No, I can’t. I have to go home.

Salesman : When will I receive my order?


Customer : I can assure you sir, the order will be delivered tomorrow.

Neela : Do you think I should write about my education background in the resume?

Imagine you have been to Thanjavur recently. Based on your experience and the
data given below about Thanjavur, suggest and guide your friend who wishes to visit
Thanjavur and places nearby, using modals in your sentences.

Trains towards Places to visit in Places around Unique Products of


Thanjavur Thanjavur Thanjavur Thanjavur
• Uzhavan Express •B
 rihadeeswarar Temple • Thiruvaiyaru • Art Plates
• Mannai Express (Big temple) • Kumbakonam • Paintings
Prose

• Madurai Express • Museum • Kallanai Dam • Bronze Statues


• Saraswathi Mahal (Library) • Poondi (Church) • Dancing Dolls
• Palace • Manora Pattukkottai

E. Here are a few sentences already done for you. The clues given would be helpful to/
make more sentences on your own.
1. I would suggest that you take the Uzhavan Express to Thanjavur from Chennai.
UNIT 1

2. You will be more comfortable if you could book 3 tier A/C.


3. You could enjoy if you visit the museum. 8. You must visit Kallanai Dam.
4. You should visit the Big Temple. 9. You can see the palace.
5. You mustn’t miss Saraswathi Mahal. 10. 
You could buy art plates and dancing
6. You can buy beautiful Bronze Statues. dolls.
7. You should visit Poondi Church
20
ENGLISH

Active and Passive


In Class IX, we have already learnt about Active and Passive Voices. Now, we shall learn some more
forms of the voice.
Let us recall 10
When we give importance to what people and things do, we use active verb forms. When
we give importance to what happens to people and things, we often use passive verb forms.

F. Change the following sentences to the other voice.

1. The manager appointed many office assistants.

His First Flight


Many office assistants were appointed by the manager.

2. You are making a cake now.


A cake is being made by you now.

3. That portrait was painted by my grandmother.


My grandmother painted that portrait.

4. Malini had bought a colourful hat for her daughter.


A colourful hat had been bought by Malini for her daughter.

5. They have asked me to pay the fine.

6.
The police were taking the militants to prison.

7. His behaviour vexes me.


I am vexed at with his behaviour.

8. Rosy will solve the problem.


The problem will be solved by Rosy.

Prose
9. Our army has defeated the enemy.
The enemy has been defeated by our army.

10. The salesman answered all the questions patiently.


All the questions were answered by the salesman patiently.

G. Change the following into Passive voice.

1. Please call him at once.


UNIT 1

You are requested to call him at once.

2. How did you cross the river?


How was the river crossed by you?

3. No one is borrowing the novels from the library.


The novels are not being borrowed by any one from the library.
21
ENGLISH

4. Will you help me?


You are requested to help me. / Will I be helped by you?

5. Go for a jog early in the morning.


10 You are advised to go for a jog early in the morning.

6. Why have you left your brother at home?


Why has your brother been left at home?

7. Nobody should violate the rules.


The rules should not be violated.
His First Flight

8. Someone has to initiate it immediately.


It has to be initiated immediately.

9. Have you invited Raman to the party?


Has Raman been invited to the party by you?

10. Please do not walk on the grass.


You are requested not to walk on the grass.

11. Cross the busy roads carefully.


You are advised to cross the busy roads carefully.

When will the tickets be booked to Bengaluru by you?

H. In the following sentences the verbs have two objects namely Direct and Indirect
objects. Change each of the following sentences into two passives using direct object
as the subject in one and indirect in the other.

1. John gave a bar of chocolate to Jill.


a. Jill was given a bar of chocolate by John.
b. A bar of chocolate was given to Jill by John.
Prose

2. Pragathi lent a pencil to Keerthana.


a. Keerthana was lent a pencil by Pragathi.
b. A pencil was lent to Keerthana by Pragathi.

3. Sudha told the truth to her friend.


a. Sudha’s friend was told the truth by her.
b. The truth was told to Sudha’s friend by her.

4. They offered the job to Venkat.


UNIT 1

a. Venkat was offered the job by them.


b. The job was offered to Venkat by them.

5. The boss showed the new computer to Kaviya.


a. Kaviya was shown the new computer by the boss.
b. The new computer was shown to Kaviya by the boss.
22
ENGLISH

I. Rewrite the following passage in Passive Voice. B.P.No. 16

A few days ago, someone stole Ambrose’s motorbike. Ambrose had left it outside his house.
He reported the theft to the police. The police told him that they would try to find his motorbike. This
morning, they found his motorbike. The police called Ambrose to the police station. The thieves had 10
painted it and then sold it to someone else. The new owner had parked the motorbike outside a mall
when the police found it. After an enquiry, the police arrested the thieves.
Answer:
A few days ago, Ambrose’s motorbike was stolen. It had been left by Ambrose outside his
house. The theft was reported to the police by him. He was told by the police that they would try to
find his motorbike. This morning, his motorbike was found by them. Ambrose was called to the police

His First Flight


station. It had been painted and then sold by the thieves to someone else. The motorbike had been
parked by the new owner outside a mall when it was found by the police. After an enquiry, the thieves
were arrested.

J. Write a recipe of your favourite dish in passive voice. Remember to list out the
ingredients of the dish you have chosen and their quantity. Use Simple Present tense
to write your recipe.
SemiyaPayasam
Ingredients required for the recipes
i. Milk ½ liter v. Cashews 10gms
ii. Semiya 1 cup vi. Sugar 1 cup

iv. Dry grapes 10 gms


A pan is heated and ghee is poured into it. Dry grapes and cashews are added to it. It is fried
till it changes its colour to golden brown and it is kept aside. Now to the same pan the ghee is added
again. To the ghee semiya is added in the pan and it is heated till it changes its colour.
Now another pan is taken and milk is added to it. The milk is boiled. After the boiling of milk, the
semiya is added. The mixture is boiled. Then to that fried dry grapes and cashews, sugar and badam
powder are added. Now Semiya Payasam can be served.

K. Write a report of an event held at your school using Passive voice. Use Simple Past
Tense to narrate the event.

Prose
A Report
Annual Day Celebration
– By Varun, Std X.
The tenth annual day of MG Matric Higher Secondary School, Madurai was celebrated on
3rd February 2020 at the school gorund. Dr. A. Sudhakar, M.D., was the chief guest of the day. The
programme started with a prayer song which was sung by the school choir. It was followed by the
welcome address which was delivered by the Assistant Headmaster. The chief guest was felicitated
UNIT 1

with a shawl and a memento. After that the annual day report was read by the Headmaster. To make
the audience entertained a lot of cultural programmes were put up by the students. Prize winners of
various events which were conducted throughout the year were given away prizes. Just before vote of
thanks, a wonderful skit titled “Save Water, Save Life” was put up by Class X students. And the vote of
thanks was proposed by Master Ravikumar, School Pupil Leader. It was the most memorable day for
the students and the parents.
23
ENGLISH

UNIT

1
POEM
10
Life - Henry Van Dyke

SUMMARY
The poem ‘Life’ by Henry Van Dyke is a beautiful inspirational poem depicting the poet’s
determination to live a purposeful, positive, and fruitful life.
Life

In the first stanza the poet says that he wants to live his life without any hesitation, always
looking forward and advancing towards his goal without any hurry and distraction. He just wants to
live in the present without mourning for the things in the past and without fearing for the future’s
uncertainties hidden behind the vein of the present. He wants to move forward in life with a happy
heart, paying all the dues demanded by age and continue to travel forward.
In the second stanza the poet further expresses his determination to live a great life. He says
he won’t be daunted by the difficult, uphill road of life; whether he gets a rough road or smooth, he
e, and a crown in
the same manner when he was just a boy. He says his quest will never stop; and continue living in the
hope that life would be the best after the last turn.

B.P.No. 18
GLOSSARY
mourning (v) – feeling or expressing great sadness
veils (v) – to hide or cover something so that you cannot see it clearly or understand
it
Poem

crown (n) – a prize or position offered for being the best


quest (n) – a long search for something that is difficult to find
unreluctant* (adj.) – willing to do something (*This form is generally not used but the poet
has coined it for emphasis)

ADDITIONAL
UNIT 1

forward – ahead
hurrying – speeding up/ hastening
dim – not bright/ (here) unpleasant past
holding back – stopping
veils – covers/hides
24
ENGLISH

pays its toll – to harm/damage someone or something


cheer – joyfulness
wind up – arrive or end up
seeking – searching 10
adventure – unusual and unexpected experience

Poetry Appreciation Questions.

A. Read the following lines from the poem and answer the questions that follow.
1. Let me but live my life from year to year,
With forward face and unreluctant soul;
a. Whom does the word ‘me’ refer to?
The word ‘me’ refers to the poet Henry Van Dyke.

Life
b. What kind of life does the poet want to lead?
The poet wants to lead his life without any hesitation, always looking forward with a willing soul.
2. Not hurrying to, nor turning from the goal;
Not mourning for the things that disappear
a. Why do you think the poet is not in a hurry?
I think the poet is not in a hurry because he wants to enjoy all aspects of life.

One should not mourn for the things one has lost in the past.
3. In the dim past, nor holding back in fear
From what the future veils; but with a whole
And happy heart, that pays its toll
To Youth and Age, and travels on with cheer.
a. What does the poet mean by the phrase ‘in the dim past’?
The poet means by the phrase ‘in the dim past’ that what was in the past is clear in our mind.
The past becomes dimer and dimer in our life.

Poem
b. Is the poet afraid of future?
No, the poet is not afraid of the future.
c. How can one travel on with cheer?
One can travel with cheer if one does not think much the past and the future.
4. So let the way wind up the hill or down,
O’er rough or smooth, the journey will be joy:
UNIT 1

Still seeking what I sought when but a boy,


New friendship, high adventure, and a crown,
a. How is the way of life?
The way of life is not always smooth. It has ups and downs.
b. How should be the journey of life?
The journey of life should be a joyful one, full of unexpected things. 25
ENGLISH

c. What did the poet seek as a boy?


The poet as a boy sought new friendship, high adventures and success.
5. My heart will keep the courage of the quest,
10 And hope the road’s last turn will be the best.
a. What kind of quest does the poet seek here?
The poet seeks quest or opportunities which need courage to accomplish.
b. What is the poet’s hope?
The poet’s hope is that the life will be the best after the last twist in life.
6. In the dim past, nor holding back in fear
From what the future veils; but with a whole
And happy heart, that pays its toll
To Youth and Age, and travels on with cheer.
Life

a. Identify the rhyming words of the given lines.


fear – cheer; whole – toll;
7. Let me but live my life from year to year,
With forward face and unreluctant soul;
Not hurrying to, nor turning from the goal;
Not mourning for the things that disappear

abba.

ADDITIONAL

Read the following lines from the poem and answer the questions that follows.
1. Let me but live my life from year to year,
With forward face and unreluctant soul;
Not hurrying to, nor turning from the goal;
Poem

Not mourning for the things that disappear


a. Identify the rhyming words in the given lines.
year – disappear; soul – goal;
2. So let the way wind up the hill or down,
O’er rough or smooth, the journey will be joy,
Still seeking what I sought when but a boy,
New friendship, high adventure, and a crown,
UNIT 1

My heart will keep the courage pf the quest,


And hope the road’s last turn will be the best.
a. Identify the rhyming words in the given lines.
down, crown; joy – boy; quest – best;
b. Identify the rhyme scheme of the given lines.
26 abbacc
ENGLISH

3. O’er the rough or smooth, the journey will be joy;


a. Pick out the words in alliteration.
journey, joy.

B.P.No. 19
10
B. Answer the following question in about 100 – 150 words.
1. Describe the journey of life as depicted in the poem ‘Life’ by Henry Van Dyke.

Title Life
Poet Henry Van Dyke
Theme Optimism makes the journey of life cheerful

Life is very important so take it seriously.

Life
Henry Van Dyke (1852 – 1933) was an American author, poet, educator and clergyman;
His early works, ‘The story of the other Wise Man’ and ‘The First Christians Tree’, were first read aloud
to his conjugation in New York as sermons.
The poet wants to live his life looking ahead with a burning desire to accomplish something
in his life. He neither wants to hurry nor wants to move away from his goal. He does not want to
mourn or cry over the setbacks in the past. Also he does not worry about the future. He instead prefers
to live his life with a whole and happy heart which cheerfully travels from youth to old age. Hence, it
he path is rough
or smooth. Whatever may be the path, he believes that his journey will be cheerful. He states that he
will continue to seek what he wanted as a boy – new friendship, high adventure and a crown (prize).
His heart will remain courageous and pursue his desires. He hopes that every turn in his life’s journey
will be the best.

My heart will keep the courage of the quest


And hope the road’s last turn will be the best.

Poem
ADDITIONAL

Answer the following question in about 100 – 150 words.

1. How according to Dyke, should we take the ups and downs in life?

Title Life
Poet Henry Van Dyke
UNIT 1

Theme Optimism makes the journey of life cheerful

Keep your eyes on the stars and feet on the ground

Henry Van Dyke (1852 – 1933) was an American author, poet, educator and clergyman;
His early works, ‘The story of the other Wise Man’ and ‘The First Christians Tree’, were first read aloud
to his conjugation in New York as sermons. 27
ENGLISH

The poet wants us to lead our life looking ahead with a zeal to accomplish something in
course of our life. We should not be in a hurry because we should enjoy every bit of our life, whether
it is happiness or sorrow, success or failure as life progresses. We should neither mourn the things lost

10 in the past nor have fears of the things that may or may not happen in the future. He uses the phrase
‘the dim past’ to suggest that the past has always some unfulfilled things or failures about which we
should not worry. We should rather be hopeful to face the future. In addition, the poet goes to state
that one can travel on to the future with a whole and happy heart from youth to old age. If we have
will power and willingness to travel through the ups and downs of life, our life will be a high adventure.

...but with a whole


And happy heart, that pays its toll.
To Youth and Age, and travels on with cheer.

C. Based on your understanding of the poem, complete the following passage by the
Life

using the phrases given in the box.

youth to old age up or down the hill to hurry nor move away
high adventure joyful mourn looking ahead

The poet wants to live his life looking ahead, willing to do something. He neither wants to
hurry or move away from his goal. He does not want to mourn the things he has lost, not hold back
rt which cheerfully
travels from youth to old age. Therefore, it does not matter to him whether the path goes up or down
the hill, rough or smooth, the journey will be joyful. He will continue to seek what he wanted as a boy
- new friendship, high adventure and a crown (prize). His heart will remain courageous and pursue his
desires. He hopes that every turn in his life’s journey will be the best.
Poem
UNIT 1

28
ENGLISH

UNIT

1
SUPPLEMENTARY
10
The Tempest
- An Extract from Charles Lamb’s Tales From Shakespeare

SUMMARY

The Tempest
‘The Tempest’ is a play written by William Shakespeare. The present lesson is an extract from
Charles Lamb’s ‘Tales from Shakespeare’. Prospero and his daughter Miranda lived in an isolated island.
She had no contact with any human being expect her father. They lived in a cave. Prospero had many
books related to magic. With his magical power he released many spirits who had been imprisoned by
Sycorax in the bodies of large trees. Ariel was the chief of those spirits.

Ariel enjoyed tormenting an ugly monster Caliban, son of Sycorax. Caliban was doing laborious
work for Prospero. One day Prospero raised a powerful storm with the help of the spirits. It affected
a ship. He told his daughter that the ship had many human beings like them. When she saw the ship
orm so that the
people would be safe. Prospero told her that it was only to frighten them not to and hurt them.

Now Prospero informed her what had happened twenty years ago. Prospero was the Duke of
Milan, and Miranda was the heir to the throne. His brother Antonio wanted to become the Duke. He
got the help of the king of Naples to remove Prospero. So he took Prospero and Miranda in a ship and
forced them into a small boat without sail. He thought that he let them to perish. His faithful courtier
Gonzalo had kept in the boat, water, food, dress and some books without the knowledge of others.

Supplementary
The food lasted till they reached the deserted island. Now the king of Naples, his son Ferdinand and
the cruel brother were in the ship affected by the storm.

Prospero made his daughter sleep with his magic power and got a detailed report from Ariel
about the ship. Ferdinand jumped out of the ship in fear but now he was safe in the island. Others
were safe but in different places of the island. Each one thought that the other was dead. The cruel
brother was searching for his son with little hope of finding him. Prospero wanted Ferdinand to meet
Miranda. Ariel led Ferdinand to the place of Prospero. Miranda for the first time saw a human being like
her father. Ferdinand thought he was in an enchanted island and Miranda was the goddess.

When they started talking, Prospero came there and took the boy aside. He told him that he was
UNIT 1

a spy and he would be tied. Miranda was trying to speak in favour of Ferdinand. Prospero gave him
the hard work of carrying heavy logs of wood. He as a prince was not used to such work. So he was
very tired. Miranda felt sad for him and volunteered to help him. Prospero was invisible and listened
to their conversation. Ferdinand told her that he was the heir to the throne of Naples and she would
be his queen. Then Prospero appeared before them and assured Ferdinand that she would give his
daughter to him and make him very rich.
29
ENGLISH

Then Ariel informed Prospero what had happened to his cruel brother and the King of Naples.
They were wandering hungry here and there. By the time Ariel set a delicious banquet before them.
When they were about to eat it Ariel appeared before them in the shape of a harpy monster and made

10 the food disappear from there. Then the harpy spoke to them about the cruelty they had done to
Prospero and his daughter. When they heard that, they repented for the crime they had committed.
Prospero asked Ariel to bring those people before him. He brought old Gonzalo also with them.
Prospero forgave them for their cruelty. The King of Naples was very happy to see his son Ferdinand
with a beautiful girl. Prospero said that he would give his daughter as a gift to him. They all left happily
in their own boat which was hidden by Ariel. Prospero and Miranda left the island next day. Ariel was
set free from his service to Prospero.
The Tempest

B.P.No. 26
GLOSSARY
tormenting (v) – making someone suffer or worry a lot
dreadful (adj.) – extremely bad or unpleasant
duke (n) – a man of very high social rank in some European countries; a king
deprive (v) – to take something important or necessary away from someone
fatigue (n) – extreme tiredness
vexation (n) – worry or anger

voracious (adj.) – very eager for something


repent (v) – to be very sorry for something bad you have done.

A. Choose the correct answer. B.P.No. 26

1. __________ was the chief of all spirits.


a) Sycorax b) Caliban c) Ariel d) Prospero Ans : (c)
Supplementary

2. __________ raised a dreadful storm.


a) Caliban b) Prospero c) Miranda d) Sycorax Ans : (b)

3. Miranda was brought to the island __________ years ago.


a) fourteen b) ten c) twelve d) five Ans : (c)

4. Prospero ordered Ariel to bring __________ to his place.


a) Gonzalo b) Ferdinand c) King of Naples d) Antonio Ans : (b)

5. __________ had provided Prospero formerly with books and provisions.


a) Antonio b) Ferdinand c) Gonzalo d) Antonio Ans : (c)
UNIT 1

6. The second human being that Miranda saw on the island was __________.
a) Ariel b) Prospero c) Ferdinand d) Gonzalo Ans : (c)

B. Identify the character or speaker.


1. He imprisoned the spirits in the bodies of large trees. Ans. Sycorax
30 2. He was the chief of all spirits. Ans. Ariel
ENGLISH

3. I was Duke of Milan, and you were a princess. Ans. Prospero


4. What a trouble must I have been to you then! Ans. Miranda
5. Now pray tell me, sir, your reason for raising this sea-storm? Ans. Miranda
6. I will soon move you. Ans. Ariel 10
7. I will tie you neck and feet together. Ans. Prospero
8. I must finish my task before I take my rest. Ans. Ferdinard
9. He repented and implored his brother’s forgiveness. Ans. Antonio

C. Answer the following questions in one or two sentences.

The Tempest
1. Who were the inhabitants of the island?
The inhabitants of the island were Prospero and Miranda.

2. What powers did Prospero possess?


Prospero possessed the power of magic, with which he was able to release many good spirits from a
witch called Sycorax who had imprisoned them in the bodies of large trees.

3. Who was Caliban? What was he employed for?


Caliban was an ugly monster and son of Ariel’s old enemy Sycorax. He was employed like a slave to
fetch wood and do the most laborious jobs.

4. Who were on the ship? How were they related to Prospero?


s Prospero’s cruel
brother who with the help of the king of Naples sent Prospero and his daughter out of dukedom.
Gonzalo was the faithful courtier of Prospero. He kept food, water and books in the boat.
5. Why had Prospero raised a violent storm in the sea?
Prospero raised a violent storm in the sea because he came to know that his cruel brother, king of
Naples was in a ship. He wanted to wreck the ship and bring all the travellers to the island.

Supplementary
6. How did Miranda feel when her father raised the storm to destroy the ship?
Miranda felt greatly distressed when her father raised the storm to destroy the ship. She pleaded with
him to save the ship and people in it.
7. What was Ariel ordered to do with the people on the ship?
Ariel was ordered to wreck the ship but not to harm anybody.
8. Give two reasons why Miranda was so concerned about Ferdinand.
Ferdinand was the first human being she saw except Prospero and Caliban. She was very much
attracted towards the handsome young man.
9. Why did Prospero set Ferdinand a severe task to perform?
UNIT 1

Prospero wanted to test the depth of Ferdinand’s love for Miranda. So he set him a severe task to
perform.
10. How was Gonzalo helpful to Prospero when he left Milan?
Gonzalo was Prospero’s faithful courtier who had kept water, food and books in the small boat for
Prospero and his daughter.
31
ENGLISH

D. Answer the questions in a paragraph of about 100 – 150 words.

1. Write a detailed character sketch of Prospero.

10 main character – duke of Milan – sent out of Milan – reader – magical power – released
spirits – helped his cruel brother – brought up his daughter – brought everyone together

Title The Tempest


Author William Shakespeare
Theme Forgiveness is sweet revenge

Prospero was the main character in ‘Tempest’ a play written by William Shakespeare. He was
The Tempest

the Duke of Milan but he was sent out of the country by his cruel brother Antonio with the help of the
King of Naples. Prospero was fond of reading especially books of magic. He attained the power to
use magic on others. He was a sympathetic man so he released the spirits which were imprisoned by
Sycorax. Though he was ill treated by his brother, at the end of the play he helped them. He raised a
storm to wreck the ship in which his brother and the king of Naples travelled. But he did not want to
hurt any of them. He was a good father. He brought up his daughter Miranda very well. He wanted to
be together with his enemy king, the King of Naples so he was ready to give his daughter to Ferdinand.
He forgave those who caused him pains and sufferings.

Darkness cannot drive out darkness; only light can do that.


g.

2. Narrate how Prospero made his enemies repent to restore his dukedom.
sent out of Milan – reached deserted island – released spirits – magical power – cruel
brother and King of Naples – brought to island – Ferdinand met Miranda – others repented
– brought to Prospero – forgiven

Title The Tempest


Author William Shakespeare
Supplementary

Theme Forgiveness is sweet revenge

Prospero was sent out of Milan by his cruel brother Antonio with the help of the King of Naples.
He went to a deserted island with his daughter Miranda. He lived there with the spirts he released
with his magical power. One day after twelve years of his life in the desert he came to know that his
brother and the King of Naples were travelling in a ship. He raised a storm to wreck the ship but he
did not want to hurt anyone. They were all brought to the island. Each one thought that other one
was dead. Ferdinand was brought to meet Miranda and they fell in love with each other. This was the
plan of Prospero. The others were moving up and down hungry in the island. Ariel brought delicious
food to them but when they were about to eat he made it disappear. Then he made them realise the
UNIT 1

crime they had committed to Prospero and his daughter. They repented for their crime. Prospero asked
Ariel to bring them all to him. Then he forgave them and was ready to give his daughter in marriage
to Ferdinand, the son of the king of Naples.

Without forgiveness, life is governed by an endless cycle of resentment and retaliation.

32
ENGLISH

ADDITIONAL PARAGRAPH

1. Write a paragraph depicting the character of Miranda by developing the following hints. 10
beautiful girl – came to the island – with father Prospero – in a cave – no human face
except father – father raised a storm – wreck a ship –pleaded to save the ship – not happy
to see the suffering – attracted towards Ferdinand – showed kindness to Ferdinand –
happy to become the queen of Ferdinand.

Title The Tempest

The Tempest
Author William Shakespeare
Theme Forgiveness is sweet revenge

Gravitation is not responsible for people falling in love – Albert Einstein

Miranda was the daughter of Prospero who was the Duke of Milan. Antonio and the King
of Naples sent out Prospero and Miranda from Milan and they reached a deserted island and lived in a
cave. She came to the island when she was very young so she had not seen any human being other
than his father. Once her father raised a storm in the sea and it affected a ship and made sink it.
When she saw it she pleaded with her father to stop the storm and save the ship. He told her that
other than her
father. She was attracted towards him. Prospero wanted to test his love for Miranda so he gave him a
laborious task. Miranda requested her father to show kindness to Ferdinand. Prospero was not willing
to reduce the task. She told Ferdinand that she would work for him if he rested. Miranda was a loving
and caring princess.

Love, having no geography, knows no boundaries.

Supplementary
E. Rearrange the following sentences in coherent order.

3 He ordered Ariel to torment the inmates of the ship.


7 Miranda was attracted by Ferdinand and had more concern towards him.
1 Prospero and Miranda came to an island and lived in a cave.
10 Prospero forgave them and restored his dukedom, Milan.
4 He raised a violent storm in the sea to wreck the ship of his enemies.
8 Prospero wanted to test Ferdinand and gave a severe task to perform.
2 Using his powers, Prospero released the good spirits from large bodies of trees.
UNIT 1

9 T
 he King of Naples, and Antonio the false brother, repented the injustice they had done
to Prospero.
5 Ariel was instructed to bring Ferdinand, the prince of Naples to his cave.
6 Ferdinand was the second human whom Miranda had seen after her father.
33
ENGLISH

ADDITIONAL

10 I. Rearrange the following sentences in coherent order.


I 1. Prospero lived in a cave.
2. Antonio, his brother sent him out of Milan.
3. Prospero was the Duke of Milan.
4. He came to a deserted island with his daughter.
5. Miranda was his daughter. Answer: 3, 5, 2, 4, 1
The Tempest

II 1. He was the leader of all the released spirits.


2. Ariel was one of the spirits released by Prospero.
3. Prospero was a kind person and he had magical power.
4. He did everything demanded by Prospero.
5. Prospero released the spirits imprisoned by Sycorax. Answer: 3, 5, 2, 1, 4

III 1. Ferdinand jumped into the sea.


2. Antonio, the King of Naples and his son Ferdinand travelled in a ship.
3. Ferdinand met Miranda.
4. He was brought safe to the island by Ariel.
Answer: 2, 5, 1, 4, 3

II. Identify the character.


1. “Then bring them here, Ariel.” – Prospero
2. “If you will sit down, I will carry your logs the while.” – Miranda
3. “”O my dear lady, I dare not.” – Ferdinand
4. “I left them searching for Ferdinand – Ariel
Supplementary

5. “I have a gift in store for you.” – Prospero

II. Reading Comprehension.

Read the following passage and answer the questions given below.
1. “Come on, young man,” said Prospero to the Prince; “you have no power to disobey me.”
Prospero had commanded Ferdinand to pile up some heavy logs of wood. Kings’ sons not being much
used to laborious work, Miranda soon after found him almost dying with fatigue. “Alas!” said she, “do
not work so hard; my father is at his studies, he is safe for these three hours; pray rest yourself. “O
my dear lady,” said Ferdinand, “I dare not. I must finish my task before I take my rest.” “If you will sit
down,” said Miranda, “I will carry your logs the while.” But this Ferdinand would by no means agree to.
UNIT 1

Questions:
1. What did Prospero say to the Prince?
2. How did Prospero command Ferdinand?
3. How did Miranda find Ferdinand during laborious work?
4. What did Miranda say to Ferdinand?
34
5. What was the response of Ferdinand?
ENGLISH

Answers:
1. Prospero said to the Prince that he had no power to disobey him.
2. Prospero commanded Ferdinand to pile up some heavy logs of wood.
3. Miranda found Ferdinand fatigued. 10
4. Miranda said to him to take rest as her father was at his studies.
5. Ferdinand refused to take a rest.

2. “No, my love,” said Prospero, “you were a little angel that did preserve me. Your innocent
smiles made me bear up against misfortunes. Our food lasted till we landed on this desert island,
since when my chief delight has been in teaching you, Miranda, and well have you profited by my
instructions.” “Heaven thank you, my dear father,” said Miranda. “Now tell me, sir, your reason for

The Tempest
raising this sea-storm?” “Know then, “said her father, “that by means of this storm, my enemies, the
King of Naples, and my cruel brother, are cast ashore upon this island.” Having so said, Prospero gently
touched his daughter with his magic wand, and she fell fast asleep.

Questions :
1. What did Prospero applaud his daughter?
2. How did Prospero withstand his misfortunes?
3. How long did their food last?
4. What did Miranda ask her father?
5. What did Prospero do to his daughter?


1. Prospero applauded his daughter saying that she was a little angel that preserved him.
2. Prospero withstood his misfortunes because of Miranda’s innocent smiles.
3. Their food lasted till they landed on that desert island.
4. Miranda asked her father why he raised that sea-storm.
5. Prospero gently touched his daughter with his magic wand and she fell fast asleep.

3. Ariel then went to fetch Ferdinand. “O my young gentleman,” said Ariel, when he saw him, “I

Supplementary
will soon move you. You must be brought, I find, for the Lady Miranda to have a sight of your pretty
person. Come, sir, follow me.” He followed in amazement the sound of Ariel’s voice, till it led him to
Prospero and Miranda, who were sitting under the shade of a large tree. Now Miranda had never
seen a man before, except her own father. “Miranda,” said Prospero, “tell me what you are looking at
yonder.” “O father,” said Miranda, in a strange surprise, “surely that is a spirit, Lord! How it looks about!
Believe me, it is a beautiful creature. Is it not a spirit?”

Questions : Answers :
1. Who went to fetch Ferdinand? 1. Ariel went to fetch Ferdinand.
2. What did Ariel tell Ferdinand? 2. Ariel told Ferdinand that Lady Miranda
wanted to have a glimpse of him.
UNIT 1

3. What did Ferdinand do?


3. Ferdinand followed Ariel in amazement.
4. Where were Prospero and Miranda
4. Prospero and Miranda were sitting under
sitting?
the shade of a large tree
5. Had Miranda seen a man before in the
5. No, she had not seen a man before in
island?
the island.
35
ENGLISH

UNIT

2
PROSE
10
The Night the Ghost Got In
- James Grover Thurber
The Night the Ghost Got In

SUMMARY

‘The Night the Ghost Got In’ is a short story written by James Thurber. It is a fictionalized
account of a night in his childhood.
On the night of November, 17 1915, Thurber was taking a bath. He heard some footsteps.
It was around 1 in the morning. He thought that it must be his father and brother, Roy. They had gone
to Indiana and were expected back at any time. When he put his head out of the door, he did not see
anyone. He wrapped a towel around his waist and walked out into the hallway. He heard footsteps
again. He woke his brother, Herman, to help him find out what was going on. They did not see anyone
and did not hear anything anymore, so they went back to their rooms and closed the doors.
hat was
going on. She believed there were burglars in the house and threw a shoe into the window of the
neighbour’s house. The neighbour was very angry. When he came to know that she had done it to
wake him up to call the police he was calm. She asked him to call the police. The police arrived with
few reporters, and things just went wild. The grandfather ended up shooting one of the policemen
in the arm, thinking that he was a deserter. They found no evidence and left. The next morning at
breakfast, the family did not think the grandfather would remember anything. But he sat down and
asked what the cops were doing in the house last night.

B.P.No. 34
Prose

GLOSSARY

hullabaloo (v) – lot of loud noise made by people who are excited.
patrolman (n) – a patrolling police officer.
attic (n) – a space or room inside or partly inside the roof of a building
slammed (v) – shut a door or window forcefully and loudly.
gruffly (adv.) – sadly
UNIT 2

intuitively (adv.) – without conscious reasoning, instinctively


whammed (v) – struck something forcefully
bevelled (v) – reduced to a slopping edge
rending (v) – tearing to pieces

36 yanked (v) – pulled with a jerk


ENGLISH

zither (n) –  a musical instrument consisting of a flat wooden sound box with
numerous strings stretched across it, placed horizontally and played
with fingers
guinea pig (n) – a domesticated tailless South American rodent originally raised for food 10
hysterical (adj.) – affected by wildly uncontrolled emotion
creaking (v) – making a squeaking sound when being moved
indignant (adj.) – feeling or showing anger or annoyance at what is perceived as unfair

The Night the Ghost Got In


treatment

holster (n) – a holder made of leather for carrying handgun


rafter (n) – a beam forming part of the internal framework of a roof
deserter (n) – a person who leaves the armed force without permission.

ADDITIONAL

advent – arrival gruffly – abruptly, angrily


caused – affected intuitively – automatically
quick-cadenced – noise of quick walk frothing – foaming

enormously – extremely, very


faint – feeble
hoarse – rough
plate-rail – a bar to stop the falling
whooping – screaming
of the plates
emerged – appeared.
trod – walked
intervene – interfere
burglar – thief retreat – withdrawal
tiptoed – crept (without making indignant – angry
noise)

Prose
hissed – whispered reluctant – unwilling

despondent – hopeless distinctly – clearly

beagle – dog with short legs obviously – clearly

ceased – stopped phony – deceiving

alarm – terror poke – dig,


wispy – thin
gripped – grasped gazed – stared
UNIT 2

instantly – suddenly blaspheming – cursing


aroused – woke up glared – frowned
ventured – volunteered

37
ENGLISH

EXERCISE WITH ANSWER

10 Model : 1

Choose the appropriate synonym of the word underlined.


The Night the Ghost Got In

1. Its advent caused my mother to throw a shoe through a window.


a) informed b) affected c) asked d) compelled

2. I could see the faint shine of plates on the plate-rail.


a) feeble b) bright c) thick d) big

3. ‘Past!’ I hissed, in the dark, shaking him.


a) kissed b) said c) whispered d) shouted

4. He wanted to go back to bed, I gripped his arm.


a) removed b) advised c) asked d) grasped

5. Herman rushed to his room and slammed the door.


a) banged b) opened c) broke d) made

6. She picked up the shoe and whammed it through a pane of glass.

7. Bodwell was at the window in a minute, shouting frothing little.


a) easing b) talking c) running d) foaming

8. Police were all over the place; doors were yanked open.
a) jerked b) holed c) gunned d) slowly

9. He believed that General Meade’s men were beginning to retreat and even desert.
a) tell b) show c) withdraw d) keep
Prose

10. The night had been distinctly a defeat for them.


a) only b) clearly c) merely d) immediately

11. I can see their viewpoint – phony.


a) clearly b) immediately c) slowly d) deceiving

12. He gazed at me a long time as if I were a slot machine.


a) stared b) shouted c) jumped d) hated
UNIT 2

Answers
1. b 2. a 3. c 4. d 5. a 6. b 7. d 8. a 9. c 10. b 11. d 12. a

38
ENGLISH

EXERCISE for SELF EVALUATION


Choose the appropriate synonym of the word underlined. 10
1. Its advent caused my mother to throw a shoe through a window.
a) sight b) venture c) arrival d) talk Ans :

The Night the Ghost Got In


2. There were the steps of a man walking rapidly around the dining-table downstairs.
a) quickly b) slowly c) unnoticed d) stealthily Ans :

3. At regular intervals a board creaked, when it was trod upon.


a) seen b) kept c) left d) walked Ans :

4. After the walking had gone on for perhaps three minutes, I tiptoed to Herman’s room.
a) suggested b) crept c) gave d) offered Ans :

5. ‘Awp’, he said, in the low, hopeless tone of a despondent beagle.


a) doubtful b) careful c) hopeless d) happy Ans :

6. The steps has ceased.


a) started b) broke c) created d) stopped Ans :

7. Herman looked at me in some alarm:

unning, 8.
a) suddenly b) slowly c) happily d) sadly Ans :

9. The slamming of the door aroused mother:


a) irritated b) watered c) woke up d) kicked up Ans :

10. She peered out of her room.


a) slept b) laughed c) shouted d) looked Ans :

11. Herman ventured out of his room.

Prose
a) volunteered b) stayed c) looked d) jumped Ans :

12. ‘Nothing,’ he said, gruffly, but he was, in colour, a light green.


a) loudly b) softly c) angrily d) quickly Ans :

13. ‘Burglars!’ she shouted, intuitively.


a) sadly b) automatically c) slowly d) softly Ans :

14. The thrill of heaving a shoe through a window glass had enormously taken her fancy.
UNIT 2

a) quickly b) slowly c) happily d) extremely Ans :

15. ‘Open up!’ cried a hoarse voice.


a) rough b) sweet c) slight d) loud Ans :

16. ‘There were two or three of them,’ mother said, ‘whooping and carrying on slamming
doors.’
a) laughing b) crying c) screaming d) jumping Ans : 39
ENGLISH

17. A half-dozen policemen emerged out of the darkness of the front hallway upstairs.
a) disappeared b) appeared c) slipped d) acted Ans :

18. Five or six cops sprang for the attic door before I could intervene or explain.
10 a) convince b) wait c) tell d) interfere Ans :

19. The cops must have realized at once that the indignant white-haired old man belonged to
the house,
The Night the Ghost Got In

a) angry b) soft c) waiting d) shouting Ans :

20. The cops were reluctant to leave without getting their hand on somebody besides
grandfather;
a) willing b) unwilling c) ready d) happy Ans :

21. They obviously did not like the layout;


a) quietly b) doubtfully c) clearly d) sadly Ans :

22. They began to poke into things again.


a) dig b) throw c) receive d) keep Ans :

23. A reporter, a thin-faced, wispy man, came up to me.


a) thick b) broad c) huge d) thin Ans :

24. The cops followed him, the one grandfather shot holding his now- bandaged arm, cursing

a) reading b) cursing c) shouting d) laughing Ans :

25. Over his third cup of coffee, he glared at Herman and me.
a) frowned b) looked c) laughed d) mocked Ans :

26. I could hear a rending of wood and a flash of glass on the floor of the hall.
a) carrying b) throwing c) tearing d) keeping Ans :
Prose

ANTONYMS

advent × departure quieted × shouted


began × ended enormously × small
remember × forget hoarse × soft
rapidly × slowly heavy × light
faint × bright thick × thin
UNIT 2

expected × unexpected emerged × disappeared


ceased × started retreat × advance
instantly × gradually evidently × doubtfully
palely × brightly reluctant × willing
demanded × requested
40
ENGLISH

EXERCISE WITH ANSWER

Choose the appropriate antonym of the word underlined. 10


1. Its advent caused my mother to throw a shoe through a window of the house next door.
a) arrival b) departure c) keeping d) hiding Ans : b)

The Night the Ghost Got In


2. I could see the faint shine of plates on the plate-rail.
a) bright b) gloomy c) quick d) slow Ans : a)

3. The steps had ceased.


a) stopped b) slipped c) started d) broken Ans : c)

4. I supposed at first that it was my father or my brother Roy, who had gone to Indianapolis
but were expected home at any time.
a) inexpected b) accepted c) ready d) unexpected Ans : d)

5. But finally he quieted down and called the police for us over an extension phone by his
bed.
a) shouted b) calmed c) left d) kept Ans : a)

6. Finally the cops put their shoulders to our big heavy front door with its thick bevelled glass

a) thick b) light c) huge d) attractive Ans : b)

7. Finally the cops put their shoulders to our big heavy front door with its thick bevelled glass
and broke it in:
a) dirty b) clean c) thin d) slopped Ans : c)

Answers
1. b 2. a 3. c 4. d 5. a 6. b 7. c

Prose
EXERCISE for SELF EVALUATION

Choose the appropriate antonym of the word underlined.


1. They began about a quarter past one o’clock in the morning,
a) ended b) started c) called d) shouted Ans :

2. Grandfather was in the attic, in the old walnut bed which, as you will remember, once fell
UNIT 2

on my father.
a) remind b) remain c) keep d) forget Ans :

3. They were the steps of a man walking rapidly around the dining-table downstairs.
a) quickly b) slowly c) promptly d) swiftly Ans :

4. Instantly the steps began again, circled the dining-room table like a man running,
a) immediately b) suddenly c) slowly d) calmly Ans : 41
ENGLISH

5. The light still shone palely down the stairs.


a) brightly b) lightly c) heavily d) happily Ans :

6. ‘What on earth re you boys doing?’ she demanded.


10 a) ordered b) fascinated c) interested d) requested Ans :

7. The thrill of heaving a shoe through a window glass had enormously taken her fancy.
a) in a big way b) in a small way c) happily d) sadly Ans :
The Night the Ghost Got In

8. ‘Open up!’ cried a hoarse voice.


a) rough b) thick c) soft d) quick Ans :

9. A half-dozen policemen emerged out of the darkness of the front hallway upstairs.
a) appeared b) disappeared c) came d) keep Ans :

10. General Meade’s men were beginning to retreat and even desert.
a) advance b) withdraw c) march d) hide Ans :

11. Grandfather had evidently jumped to the conclusion that the police were deserters from
Meade’s army,
a) clearly b) surely c) willingly d) doubtfully Ans :

12. The cops were reluctant to leave without getting their hand on somebody besides
grandfather;
:

ADDITIONAL

Root Words Synonyms Antonyms


rapidly hastily slowly
expected awaited unexpected
ghost devil angel
Prose

desponent miserable cheerful


gripped clutched released
sprawling stretching contracting
ventured adventure idleness
quieten calm disturb
extension expansion reduction
UNIT 2

banging slam -
hysterical agitated peaceful
desert abandon stay
cursed imprecation blessed
reluctant unwilling willing
42 frank blunt secretive
ENGLISH

fresh new old


glared stare blink
demanded appeal deny
10
TEXTUAL QUESTIONS and ANSWERS

The Night the Ghost Got In


a. Where was the author when he heard the noise? B.P.No. 30

The author was in the bath room when he heard the noise.

b. What did the narrator think the unusual sound was?


First he thought it was the noise created by his father and his brother who must have returned from
Indianapolis. Then he thought it must be some burglar. Again he thought it must be a ghost

c. What were the various sounds the brothers heard when they went downstairs? B.P.No. 31
They heard the sounds of someone walking.

d. Who were the narrator’s neighbours?


Mr. and Mrs. Bodwell were the narrator’s neighbours.

B.P.No. 32 e.

Bodwell shouted angrily and said that he would sell the house and go back to Peoria.

f. What did the Bodwells think when they heard the mother shout?
When Bodwell heard the mother shout he thought that some burglars had entered his house.

g. What was the grandfather wearing? B.P.No. 33

The grandfather was wearing a long flannel nightgown over long woolen pants, a nightcap, and a
leather jacket around his chest.

Prose
h. What conclusions did grandfather jump to when he saw the cops?
When the grandfather saw the cops he concluded that they were deserters.

i. Were the policemen willing to leave the house? B.P.No. 34

No, they were not willing to leave the house without getting their hand on somebody besides the
grandfather, because the night had been a defeat for them.

j. What made the reporter gaze at the author?


As the author wore one of his mother’s dress, which looked funny, the reporter gazed at the author.
UNIT 2

When the author said they had ghosts, the reporter gazed at him a long time.

A. Answer the following questions in a sentence or two. B.P.No. 34

1. Why was the narrator sorry to have paid attention to the footsteps?
As he paid attention to the footsteps he made a big confusion in his family. His misjudgment even
brought police to his house. So the narrator was sorry to have paid attention to the footsteps. 43
ENGLISH

2. Why did Herman and the author slam the doors?


Herman and the author did not see anyone and did not hear anything anymore, so they went back to
their rooms and slammed the doors.

10 3. What woke up the mother?


Slamming of the door woke up the mother.

4. What do you understand by the mother’s act of throwing the shoe?


The Night the Ghost Got In

The mother wanted to call the police for help but she could not use her phone as it was downstairs.
She had to wake up her neighbour to call the police. She had no other way rather than throwing the
shoe at their window glass to wake them up.

5. Why do you think Mrs. Bodwell wanted to sell the house?


Mrs. Bodwell thought that her neighbour had broken the window glass to trouble them. She did not
want to tolerate such behaviour. So she wanted to sell the house .

6. How did the cops manage to enter the locked house?


The cops broke open the door and entered the house.

7. Why were the policemen prevented from entering grandfather’s room?


The grandfather was sleeping in the attic. He was against the deserters. If he saw the cops he would

8. Who used the zither and how?


Roy had won the zither in a pool tournament. It was used by the guinea pig to sleep.

9. Mention the things the grandfather imagined.


The grandfather imagined that the cops were the deserters from meade’s army, trying to hide away in
his attic.

ADDITIONAL
Prose

Answer the following questions in a sentence or two.

1. Where was the mother sleeping?


The mother was sleeping in a room upstairs.

2. Whom did the author inform about the footsteps first?


The author woke up his brother Herman and informed him about the footsteps first.
UNIT 2

3. What was the reaction of the mother when he heard about the footsteps?
When the mother heard about the footsteps she said that there must be some burglars

4. Why couldn’t the mother phone up to the police?


The mother could not phone up to the police because the phone was downstairs and they were afraid
to come to that place.
44
ENGLISH

5. How did the police come to the house?


The police came in a Ford sedan, two motorbikes and a patrol wagon with some reporters.

6. What did the grandfather did with a policemen?


The grandfather smacked alongside of the head of a policeman and he opened fire and hit another
10
policeman’s shoulder

7. What did the grandfather complain to the author and his brother the next day morning?

The Night the Ghost Got In


The grandfather complained that none of them bothered to leave a bottle of water beside his bed.

B. Answer the following questions in about 100-150 words. B.P.No. 35

1. Describe the funny incident that caused the confusion in the house.

Prose The Night the Ghost Got In


Author James Grover Thurber
Theme Dramatic fuss about mysterious sounds

In the confusion we stay with each other, happy to be


together, speaking without uttering a single word.

‘The Night the Ghost Got In’, is a short story written by James Thurber. It is a fictionalized
as taking a bath
suddenly he heard some footsteps. It was around 1 in the morning. He thought that it must be his
father and brother, Roy. They had gone to Indiana and were expected back at any time. When he put
his head out the door, he did not see anyone. He wrapped a towel around his waist and walked out
into the hallway. He heard footsteps again. He woke his brother, Herman, to help him find out what
was going on. They did not see anyone and did not hear anything anymore, so they went back to their
rooms and closed the doors. Their mother was awakened by the slamming of the doors and wanted to
know what was going on. She believed there were burglars in the house. Somehow with the help of the
neighbour she informed the police. The police arrived with a few reporters and things just went wild.
The grandfather ended up shooting one of the policemen in the arm, thinking that he was a deserter.

Prose
They found no evidence and left.

If you’re not confused, you’re not paying attention.

2. Narrate the extensive search operation made by the policemen in the house.

Prose The Night the Ghost Got In


Author James Grover Thurber
Theme Dramatic fuss about mysterious sounds
UNIT 2

Confusion is a word we have invented for


an order which is not yet understood

‘The Night the Ghost Got In’, is a short story written by James Thurber. It is a fictionalized
account of a night in his childhood. The author was in the bathroom and around I in the morning he
heard some steps. He and his brother searched here and there but no one was seen. So they closed 45
ENGLISH

their room doors with a bang. The sound of the bang woke up the mother. When she heard about
the footsteps she thought there must be some burglars. Somehow with the help of the neighbour
she called the police and they came in a short time. The police broke open the door and entered

10 the house. They could not find anybody. They opened all the drawers and windows and searched
everywhere. They pulled beds from the walls and they removed the clothes from the closets. They
saw the zither which was used by guinea pig to sleep. The cops were reluctant to leave without getting
their hand on somebody besides grandfather; the night had been distinctly a defeat for them. Finally
The Night the Ghost Got In

they could not find anybody.

Confusion is the welcome mat at the door of creativity.

ADDITIONAL PARAGRAPHS

Answer the following questions in about 100-150 words.

1. Narrate the story ‘The Night the Ghost Got In’ in your own words.

Prose The Night the Ghost Got In


Author James Grover Thurber
Dramatic fuss about mysterious sounds

If you look confused it is because you are thinking.

‘The Night the Ghost Got In’, is a short story written by James Thurber. It is a fictionalized
account of a night in his childhood. The author was taking a bath. He heard some footsteps. It was
around 1 in the morning. He thought that it must be his father and brother, Roy. They had gone to
Indiana and were expected back at any time. He wrapped a towel around his waist and walked out
into the hallway. He heard footsteps again. He woke his brother, Herman, to help him find out what
was going on. They did not see anyone and did not hear anything anymore, so they went back to their
rooms and closed the doors. Their mother was awakened by the slamming of the doors and wanted to
know what was going on. She believed there were burglars in the house. Somehow with the help of the
Prose

neighbour she informed the police. The police arrived with a few reporters and things just went wild.
The grandfather ended up shooting one of the policemen in the arm, thinking that he was a deserter.
They found no evidence and left.

The more I think the more I get confused.

2. How did Grandfather behave the next morning?

Prose The Night the Ghost Got In


UNIT 2

Author James Grover Thurber


Theme Dramatic fuss about mysterious sounds

Grandfather was fresh as a daisy and


full of jokes at breakfast next morning.
46
ENGLISH

When the cops rushed to Grandfather’s attic, the author realized that it would be bad
if they entered Grandfather’s room unannounced, or even announced. The reason was that he was
going through a phase in which he believed that General Meade’s men, because of the attack by
Stonewall Jackson, were beginning to retreat and even desert. But they did not listen. They entered
and Grandfather believed that the police were deserters from Meade’s army, trying to hide away in his
10
attic. When they did so, Grandfather took his zither gun and fired at them in which one police man
was shot in the arm. Then he fired once or twice more in the darkness and went back to bed. The

The Night the Ghost Got In


next morning Grandfather was fresh and was full of jokes at breakfast. They thought at first that he
had forgotten all about what had happened. But he had not. Over his third cup of coffee, he stared at
the narrator and Herman and asked them why the cops were wandering round the house the previous
night. He complained that none of them remembered to keep a bottle of water beside his bed. He said
that he felt very thirsty and so he was looking for water in the dining room the previous day night.

Tension and expectation are inseparable.

3. Describe the incidents that led to the rousing of the mother and the calling of the police.

Prose The Night the Ghost Got In


Author James Grover Thurber
Theme Dramatic fuss about mysterious sounds

‘Burglars!’ She shouted intuitively.

The narrator hearing the fool steps of someone suspected that it was a burglar. He
immediately roused his brother Herman saying there was something downstairs. After sometime they
heard the footsteps. They got frightened and rushed inside rooms and slammed the doors. Their
slamming of the doors aroused their mother. After enquiring them, she suspected a burglar in the
house. She wanted to call the police. But the phone was downstairs and no one tried to go downstairs
because they were frightened. She then made a quick decision of throwing a shoe through a pane of
glass. The glass broke into the bed room of the Bodwells. She tried to convey to them that there was
a burglar in the house. But Mr. Bodwell was furious with her, screaming and shaking his fist. He called

Prose
the police, thinking that there was a burglar in his house. The police arrived in a commendably short
time.

If you look confused it is because you are thinking.

B.P.No. 35
VOCABULARY
UNIT 2

Slang Expression
Slang is a type of language consisting of words and phrases that are regarded as very informal
and more common in speech than writing. They are typically restricted to a particular context or
group of people.

47
ENGLISH

C. Look at the following expressions from the text. With the help of your teacher rewrite
them in standard English. One has been done for you.
1. ‘Musta got away –whatt’d he like? Must got away-what was he like?
10 2. ‘Looky here, Joe Look in here, Joe.
3. ‘No sign o’ nothing’ ` No sign of anything.
4. ‘Back t’ the lines ye goodaam’ Back to the lines you good man (godown)
The Night the Ghost Got In

5. ‘What was the idée of all them cops tarryhootin’ What was the idea of all of the cops tarry
round the house last night?’ hooting round the house last night?

Singular and Plural Forms


In this lesson, we find plural forms such as furniture, houses, windows, burglars, boxes, shelves,
policemen. You may notice that these words have taken up different suffixes to form plurals. This is
because English words have different origins.

D. Complete the given tabular column with the suitable plural forms.
Singular Plural Singular Plural
chair chairs formula formulae
box boxes child children
eskimo eskimos deer deer

radius radii hero heroes

B.P.No. 36
LISTENING

E. Listen to the story and answer the following.

1. The rich man was from....


a) Nagaland b) Thailand c) Finland Ans : b) Thailand
Prose

2. Where did Chulong catch the bird?


Chulong caught the bird from a bush in his garden.

3. Why did Chulong catch the bird?


Chulong caught the bird because it was a strange bird and he wanted to make money by selling it .

4. What will happen to the bird in imprisonment?


The bird will lose its beauty and sweet voice in imprisonment.
UNIT 2

5. What did the bird suggest Chulong, in exchange for its freedom?
The bird suggested that it would give him three simple rules in exchange of its freedom.

6. Does Chulong want to earn money honestly?


No, Chulong does not want to earn money honestly.
48
ENGLISH

7. What were Chulong’s plans for the bird?


Chulong’s plans for the bird were to catch it and sell it for good money.

8. Who is wise according to you?


The bird is wiser than Chulong
10
9. Was the bird a crow?
No, the bird was not a crow.

The Night the Ghost Got In


10. What are the three rules given by the bird?
• Never believe anything others say
• Never be sad for something which you do not have
• Never throw away what you have in hand.

B.P.No. 36
SPEAKING
F. Quiz: Who am I?
Sample questions to ask. Answers must be ‘yes’ or ‘no’ only.
• Are you a male (female)?

• Are you a singer (dancer, actor)?


• Are you a historical figure?
• Are you young (old)?
• Are you alive now?
• Does your name start with ‘___’ ?
• Is he/she ____ ?

G. Use this passage to play the game. You can collect information on other famous
personalities and play too.  B.P.No. 36

Prose
Charlie Chaplin was born on April 16, 1889, in London England. His birth name was Charles
Spencer Chaplin, though he had many nicknames growing up such as Charlie, Charlot, and The Little
Tramp. His father, Charles Chaplin, and his mother, Hannah Chaplin, were inducted into the music hall
of fame, leading the way to his exposure even as a young boy. His first onstage moment was when he
was 5 years old; he sang a song that was intended to be sung by his own mother; she had become ill
at the time of the performance, so little Charlie Chaplin stood instead and performed for his mother.
Charlie Chaplin came to the United States in 1910, at the age of 21. He was brought to
UNIT 2

New York, which was known to be a great place to start out for anyone trying to become a professional
actor. Two years later, in 1913, Chaplin signed his very first contract at Keystone and it was no time
before he headed to Hollywood. His first movie premiered in 1914, “Making a Living,” and went on to
make over 35 movies total in that year alone. Charlie Chaplin grew to become one of the most popular
and successful actors of all time. The moment that really kicked off his long career was in 1921 when
he starred in, and produced, his first full length film called “The Kid.” From then on, most people all
over the world knew Charlie Chaplin and loved his movies. He had a great career and life, dying on 49
ENGLISH

December 25, 1977, in Vevey, Switzerland. He had apparently died of natural causes in his sleep from
old age.
Questions:
10 • Are you a male? • Are you an actor?
• Are you a foreigner? • Are you living?
• Are you from US? • Are you a comedian?
The Night the Ghost Got In

• Are you from London?

B.P.No. 38
READING

H. Read the incident again and answer the following questions.

1. What was the writer always asked to do whenever he planned to go abroad?


The writer was asked to bring something like a vaccine, ceramic paint and tie.

2. What did Gilson want the writer to bring for him?


Gilson wanted the writer to bring for him a tie with a letter G on it.

3. When did the writer remember the fact that he had to buy something for Mr. Gilson?
at he had to buy
something for Mr. Gilson.

4. Why were the other passengers in the flight gazing at the writer?
The passengers were gazing at the writer because he was late to the flight and the flight was waiting
for him.

5. What is the humour element in the above incident?


He faced much trouble and spent a lot to get the tie but he left it in the taxi and got onto the plane.

I. Suggesting titles:
Prose

Title summarises the story. Each paragraph is a part of the story. Look at the following expressions and
find out the paragraphs that best suit these expressions.

1. Oh, No! But it happens!


 “Oh, so you’re going abroad? Can you bring me back…..?” I’ve been asked to bring back
a vaccine for a course. Once I searched the suburbs of Paris for two days for a special brand of
ceramic paint. Having spent a lot of money for Cartier lighter refills, I had them confiscated at
UNIT 2

the airport just before boarding because the gas might be dangerous in the air.

2. Don’t let out your travelling dates.


 Now, two months before a trip, I stop talking to people so they won’t suspect I’m about
to travel. But someone always catches me.” I’ve heard you’re going to New York, and I want you
to get something for me. It’s just a little thing you can find anywhere. I don’t know exactly how
50 much it costs, but it shouldn’t be much. We’ll settle up when you get back”.
ENGLISH

3. Anyway, people will be people.


 What Gilson asked me to buy was, in fact a little thing: a tie. But not just any tie. He
wanted a tie with a small embroidered G. Any colour would do, as long as it had his initial. Look,
this is a special flight, I explained. We are only staying Saturday through Tuesday. On the day 10
we arrived I didn’t have time to think about the tie, but strolling around on Sunday I did see one
shop window. They were cheap, just a dollar, but all the shops were closed.

4. Search begins

The Night the Ghost Got In


 It was only when I saw our airport bus waiting outside the hotel that I remembered the
tie. I told the group to go on. I would get a taxi to the airport. And so I went in search of a
nearby shop where I had seen ties. But I couldn’t find it. I walked further down the street-one,
two, three blocks - all in vain. Back at the hotel, a bit anxious now, I took my suitcase, got a taxi
and asked the driver to rush to the street where I had seen them. The driver stopped at each
shop we passed so I could look from the window. The stores had all sorts of ties, but not the
kind I was looking for. When I finally thought I had located the right shop, I decided to go in
and check. The driver refused to wait. Parking was prohibited, he said. I promised to double the
fare, jumped out and ran into the shop. Was I going to miss the plane just for a damned tie? The
salesman was unbearably slow. When I realized that the smallest change I had was a ten dollar
note, I grabbed ten ties of different colours so I wouldn’t have to wait for change. I rushed out
with the ties in a paper bag.

 On the street I looked around. The taxi had vanished, taking my suitcase. What is more, I
was going to miss the plane. I ran to the corner, and hope flared up again: the taxi was waiting in
the next street. Quick to the airport! As I settled down inside the taxi. I sighed with relief. Gilson
was going to have enough initialized ties to last him a lifetime.

6. Hurry invites worry.


 When I reached the airport, I paid the taxi driver the double fare and grabbed my suitcase.
Panting, I boarded the plane under the reproachful gaze of the other passengers, all primly
seated with their seat belts fastened. Ready to take off. Departure had been delayed because

Prose
of me. “At least I hope you found your tie”, said one who knew the story. “I did”, I answered
triumphantly. After making myself comfortable, I reached for the paper bag to show the ties. I
had left it behind; in the taxi.

J. Look at the following situations the writer was in. He could have avoided the situation
and saved himself. Glance through the write up again and comment on what the writer
should have done in the following situations.

• Gilson asked the writer to bring a tie.


UNIT 2

He should be firm to say no

• On the day of arrival, the writer had no time to think about the tie.
He should have tried to spend some time to search for the tie.

• The writer remembered about the tie when the bus was leaving for the airport.
He should not have gone in search of the tie. He should have taken the bus to the airport. 51
ENGLISH

• The writer walked down in search of the shop.


He should not have gone in search of the tie at that time.

• The writer rushed out with the tie in a paper bag.


10 He should not have gone to the shop to buy a tie.

K. State whether the following statements are true or false.


Ans : False
The Night the Ghost Got In

1. The narrator searched for three days to buy ceramic paint.


2. The author was going to New York. Ans : True
3. Gilson asked the narrator to buy a tie. Ans : True
4. The taxi driver took away the narrator’s suitcase. Ans : False
5. Departure was delayed because of the author. Ans : True
6. The author left the ties in the taxi. Ans : True

B.P.No. 39
WRITING

M. Write a speech for your school Literary Association Celebration with the given lead.
 B.P.No. 40

2. Purpose
3. Audience
4. Language – Some Good Describing Words (Adverbs and
Adjectives), Emotive Words, Imagery etc.
5. Ending
Distinguished Chief Guest, respected Principal, beloved Supervisor, Teachers, Parents and
my dear friends! Good evening!
On this auspicious occasion I stand before you with extreme pleasure on behalf of English
Prose

Literary Association of our School. First of all, let me express my deep gratitude to you all for selecting
me President of ELA.
E L A has been a prestigious Association in our school since its inception. It has been
working rigorously during the past few years focusing on the improvement of reading, writing, speaking
and listening skills of the students. Our vision is to make our school’ a full-fledged English medium
school where each student speaks and communicates through English. We know that it is a Herculean
task still we aspire and labour for it. A Chinese proverb says “if you are planning for a year sow corns;
if you are planning for a decade plant trees and if you are planning for a life time, educate people.”
UNIT 2

Language is the medium through which educational process fulfills its aims. So it is highly essential
that students should be able to handle language confidently. The E L A has been conducting many
varied activities and competitions to improve the language skills of the students. “Practice makes a
man perfect”. We do believe in the proverb and constantly endeavour to achieve our aim.
To conclude, let me once again express my heartfelt gratitude to the management,
teachers, and students who all honoured me by giving this great responsibility. I will try my level best
52
to fulfill the visions and missions of E L A. Once again thank you one and all.
ENGLISH

B.P.No. 40
GRAMMAR

A. Nagarajan and Dhanalakshmi want to buy a new house. They have come to see a
10
house for sale. Complete the conversation below by adding a, an or the. B.P.No. 42

Nagarajan : W
 ell, here we are, No.8, Kaveri Street. I think this is the house we saw online.

The Night the Ghost Got In


What do you think of the location?
Dhanalakshmi : It is in a nice neighbourhood. And it’s close to the railway station.
Nagarajan : And the bus stop is not too far away.
Dhanalakshmi : How many rooms are there?
Nagarajan : There are three rooms, a kitchen and a balcony.
Dhanalakshmi : There is a lawn behind the house, right?
Nagarajan : T
 hat’s right. The lawn is actually quite large. Did you see any photos of the living
room, online? What does it look like?
Dhanalakshmi : The living room looks great. It looks bright and airy. It has a nice view of the hills.
But the kitchen looks a little small.

Dhanalakshmi : No, but there is an attic, where we can store things.


Nagarajan : I hope this house is a better option.
Dhanalakshmi : Let’s wait for the real estate agent. She said, she would be here at three o’clock.
Nagarajan : Look there she is!

B. Few articles are missing in the given passage. Edit the passage given below by adding
suitable articles wherever necessary.

Prose
My neighbourhood is a very interesting place. My house is located in an apartment building
downtown near many stores and offices. There is a small supermarket across street, where my family
likes to go shopping. There is also a post office and a bank near our home. In our neighbourhood there
is a small, Green Park where my friends and I like to play on weekends and holidays. There is a small
pond near the park and there are many ducks in the park. We always have a great time. In addition
there is an elementary school close to our home where my little brother studies in the third grade.
There are so many things to see and do in my neighbourhood that’s why I like it. It’s really a great
place.
UNIT 2

Prepositional Phrases
These prepositions are formed by two-word or a three-word combination such as according to,
along with, at the time of, because of, owing to, instead of etc. These kinds of prepositions are
used frequently in our day to day life.
53
ENGLISH

Examples
Preposition Meaning Example
according to as stated by, According to the weatherman, we can expect more
10 on the authority of cold weather this week.
along with together with We have to give importance to Physical Education
along with all the academic subjects.
because of on account of We stayed at home because of the bad weather.
The Night the Ghost Got In

owing to because of I can’t accept your invitation owing to a previous


engagement.
instead of in place of, substituting for I wish I were going to the party instead of my brother.
in the event of in case of The match will be stopped in the event of heavy rain.

C. Refer to the dictionary to find out the meaning of the following prepositions and match
them with the correct meaning.
Preposition Meaning Answer
due to as a substitute for because of
except for in the interest of with the exception of
with reference to irrespective of referring to
in spite of added to irrespective of

in place of referring to as a substitute for


regardless of with the exception of disregarding the difficulty
for the sake of disregarding the difficulty in the interest of

D. Fill in the blanks by choosing the most appropriate prepositional phrase from the given
options.

1. Everything falls to the ground __________ earth’s gravitational pull.


a. in addition to b. because of c. cause of Ans: (b)
Prose

2. The trial was conducted __________ the procedure of law.


a. in accordance with b. due to c. despite of Ans: (a)

3. There is a temple right __________ my house.


a. in back of b. apart from c. in front of Ans: (c)

4. As a __________ of his hard work, he achieved the target.


a. instead of b. result of c. apart from Ans: (b)
UNIT 2

5. Failure is often the __________ negligence.


a. effect of b. consequence of c. reason of Ans: (b)

6. Children are given toys __________ sweets on Children’s Day.


a. on top of b. in addition to c. due to Ans: (b)

7. The parents must be informed __________ any indiscipline conduct of their wards.
54 a. because of b. in case of c. in spite of Ans: (b)
ENGLISH

8. He didn’t turn up __________ his busy schedule.


a. consequence of b. due to c. except for Ans: (b)

9. Global warming is __________ the green house emission.


a. an effect of b. in spite of c. in addition to Ans: (a) 10
10. __________ several warnings, he continued to swim.
a. due to b. in spite of c. because of Ans: (b)

The Night the Ghost Got In


E. Edit the following passage by replacing the underlined incorrect words with correct
prepositional phrases.

Janu is studying in class X. In the event of the teachers According to

she is a disciplined student. In addition to her poverty, she in spite of

is always neat. Many students like her in case of due to

her simplicity. According to her studies, she also In addition to

participates in sports. She gets on with everyone in case of regardless of

age and gender in the school. In opposition to taking leave, she ensures In case of

that

Prose
UNIT 2

55
ENGLISH

UNIT

2
POEM
10
The Grumble Family
- Lucy Maud Montgomery
The Grumble Family

SUMMARY

‘The Grumble Family’ has been written by Lucy Maud Montgomery. The poet gives a vivid
picture of neighbourhood scenes. She says that there is a family which nobody wants to meet. They
live on ‘Complaining’ street in a city called ‘Never-Are-satisfied’. The river of ‘Discontent’ runs beside it.
They always complain about everything. They will never be satisfied and they are always discontented.
They find fault with everything. They grumble at everything whether they live a high or
low life. The weather is always too hot or cold for them. They scold summer and winter in the same
way. Nothing will go well with these people.
If they are pleased with something they will growl saying that they have nothing to grumble
edge the family. If
someone stays with them for long he will also get their way of life. He will also start grumbling about
everything. So it is better for us not to go near such a family.
Let us learn to live with a smile even if something goes wrong. We should never be the
part of the grumble family whether our life is high or low.

B.P.No. 47
GLOSSARY

discontent (adj.) – dissatisfaction with one’s circumstances


Poem

amiss (adj.) – not quite right


growl (v) – make a low guttural sound in the throat
grumble (n) – a complaint about something in a bad-tempered way
gloomy (adj.) – depressing or frightening

queerest (adj.) – the strangest or the most unusual


acknowledge (v) – accept or admit the existence or truth of
UNIT 2

terrible (adj.) – extremely bad or serious


wandering (v) – walking or moving in a leisurely or aimless way

56
ENGLISH

Poetry Appreciation Questions.  B.P.No. 47

A. Read the following lines from the poem and answer the questions given below.
1. There’s a family nobody likes to meet; 10
They live, it is said, on Complaining Street.
a. Where does the family live?
The family lives on Complaining Street.

The Grumble Family


b. Why do you think the street is named as ‘Complaining Street’?
 he street is named as Complaining Street because the people who live there always complain
T
about everything.
2. They growl at that and they growl at this;
Whatever comes, there is something amiss;
a. What does the word ‘growl’ mean here?
‘Growl’ means bark or snarl. Here it means they find fault with everything.

b. Why do they find everything amiss?


They find fault with everything so everything seems to be amiss to them.
3. Nothing goes right with the folks you meet


Nothing goes well with the people who live on the complaining street.

b. What does the word ‘gloomy’ mean here?


‘Gloomy’ means darkness. Here it means unhappy
4. The worst thing is that if anyone stays
Among them too long, he will learn their ways;
a. What is the worst thing that can happen if anyone stays with them?

Poem
If anyone stays with them he will get the habit of complaining about everything.

b. What are the ways of the Grumble family?


The grumble family never gets satisfied. They always grumble about everything.
5. And so it were wisest to keep our feet
From wandering into Complaining Street;
a. What is the wisest thing that the poet suggests?
UNIT 2

The wisest thing that the poet suggests is not to go near the complaining street.

b. What does the phrase ‘to keep our feet from wandering’ refer to?
‘To keep our feet from wandering’ means ‘to keep off our feet from wandering near the complaining
street’.

57
ENGLISH

6. Let us learn to walk with a smile and a song,


No matter if things do sometimes go wrong;
a. What does the poet expect everyone to learn?
10 The poet expects everyone to lead a happy life with a smile.

b. What should we do when things go wrong sometimes?


We should smile and lead a happy life even if things go wrong sometimes.

ADDITIONAL
The Grumble Family

A. Read the following lines from the poem and answer the questions given below.
1. In the city of Never-Are-Satisfied,
The River of Discontent beside.
a. Why is the city called ‘Never-Are-Satisfied’?
 he poet talks about the people who are never satisfied in their life. So the city they live in is
T
called so.

b. Which river is talked about? Why?


s also discontented
like the people around.
2. And whether their station be high or humble,
They are all known by the name of Grumble.
a. What is mentioned in the above lines?
Whatever is their life whether it is high or low they grumble at everything.

b. When do we grumble normally?


We grumble normally when we are discontented.
3. The weather is always too hot or cold;
Poem

Summer and winter alike they scold.


a. How do they feel about the weather?
They always feel either too hot or too cold.

b. What do they scold?


They scold the summer and winter alike.
4. But the queerest thing is that not one of the same
UNIT 2

Can be brought to acknowledge his family name;


a. What is the queerest thing?
The queerest thing is that no member of the family will be ready to acknowledge the family.

b. Pick out the rhyming words from these lines.


58 same – name
ENGLISH

Literary Devices
1. Complaining Street, Never-Are-satisfied, River - Personification
2. Alliteration
10
growl, grumble let, learn
long, learn smile, song
3. Rhyming Scheme : aabb
Rhyming Words : this, amiss cold, scold

The Grumble Family


humble, gumble meet, sheet

B. Answer the following question in about 80-120 words.  B.P.No. 48

1. Write a paragraph on ‘The Grumble Family and their attitude towards other folks.

Poem The Grumble Family


Poet L.M. Montgomery
Theme Discontentment begets despair

In the city of Never-Are-satisfied


The River of Discontentment beside

series of novels
beginning in 1908 with Anne Green Gables. A prolific writer, She published over 100 stories between
1897 and 1907.
‘The Grumble Family’ has been written by Lucy Maud Montgomery. The poet gives a vivid
picture of neighbourhood scenes. She says that there is a family which nobody wants to meet. They
live on ‘Complaining’ street in a city called ‘Never-Are-satisfied’. The river of ‘Discontent’ runs beside it.
They always complain about everything. They will never be satisfied and they are always discontented.
They find fault with everything. They grumble at everything whether they live a high or low life. The
weather is always too hot or cold for them. They scold summer and winter in the same way. Nothing

Poem
will go well with these people. If they are pleased with something they will growl saying that they
have nothing to grumble about. The funniest thing is that no member of the family will be ready to
acknowledge the family. If someone stays with them for long he will also get their way of life. He will
also start grumbling about everything.

The weather is always too hot or cold


Summer and winter alike they scold
UNIT 2

2. If you were to live in the Complaining Street, how would you deal with the people who
grumble?

Poem The Grumble Family


Poet L.M. Montgomery
Theme Discontentment begets despair
59
ENGLISH

If I were to live in the complaining street I may find it difficult to deal with such people.
I am with a lot of positive ideas. Only those who have negative ideas will complain about people and
situations. It is very clear that such people will never have a feeling of satisfaction. So I will be careful

10 not to get involved in their way of life. If they come forward to grumble over something I will not
accept it immediately. I will try to convince them that it is better to accept things as they come to us
with a smile. We should avoid taking everything negatively. I will slowly tell them that life has ups and
downs and we should be able to balance them carefully.

Attitude shows over altitude.


The Grumble Family

3. From the poem ‘The Grumble Family’ what kind of behaviour does the poet want the
readers to possess?

Poem The Grumble Family


Poet L.M. Montgomery
Theme Discontentment begets despair

They growl at the rain and they growl at the sun;


In fact, their growling is never done.

L.M. Montgomery (1874 – 1942) was a Canadian author best known for a series of novels
beginning in 1908 with Anne Green Gables. A prolific writer, She published over 100 stories between

‘The Grumble Family’ has been written by Lucy Maud Montgomery. She says that there
is a family which nobody wants to meet. They live on ‘Complaining’ street in a city called ‘Never-Are-
satisfied’. The river of ‘Discontent’ runs beside it. They always complain about everything. They will
never be satisfied and they are always discontented. The poet does not want the readers to follow this
kind of life. She wants them to lead a life with contentment. The poet accepts that even in the grumble
family the members do not want to attach themselves with the family. Let us learn to live with a smile
even if something goes wrong. We should never be the part of the grumble family whether our life is
high or low.
Poem

Let us learn to walk with a smile and a song,


No matter if things do sometimes go wrong:

ADDITIONAL PARAGRAPH

1. What does the poet L.M. Montgomery say in the poem ‘The Grumble Family’?
UNIT 2

Poem The Grumble Family


Poet L.M. Montgomery
Theme Discontentment begets despair

They growl at the rain and they growl at the sun;


60 In fact, their growling is never done.
ENGLISH

‘The Grumble Family’ has been written by Lucy Maud Montgomery. She says that there
is a family which nobody wants to meet. They live on ‘Complaining’ street in a city called ‘Never-Are-
satisfied’. The river of ‘Discontent’ runs beside it. They always complain about everything. They will
never be satisfied and they are always discontented. They grumble at everything whether they live a
high or low life. The weather is always too hot or cold for them. They scold summer and winter in the
10
same way. If they are pleased with something they will growl saying that they have nothing to grumble
about. If someone stays with them for long he will also get their way of life. Let us learn to live with a
smile even if something goes wrong. We should never be the part of the grumble family whether our
life is high or low.

The Grumble Family


Let us learn to walk with a smile and a song,
No matter if things do sometimes go wrong:

Literary Devices
Anaphora
An anaphora is a technique where several phrases (or verses in a poem) begin with the same word or
words.
e.g. They growl at the rain and they growl at the sun.

Epithet
An epithet is an adjective or phrase expressing a quality or attribute regarded as characteristic of the

e.g. grumble family

C. Answer the following. B.P.No. 48

1. And whether their station be high or humble,…


Pick out the alliteration from the above line.
high – humble.

2. Pick out the other examples for alliteration from the poem.
They growl at that and they growl at this;

Poem
they – that
But the queerest thing is that not one of the same
thing – that
Let us learn to walk with a smile and a song,
smile, song
3. The weather is always too hot or cold;
Summer and winter alike they scold.
UNIT 2

Nothing goes right with the folks you meet


Down on that gloomy Complaining Street.
Pick out the rhyming words and identify the rhyme scheme of the above lines.
Rhyming words: cold – sold; meet – street
The rhyme scheme of the above lines : aabb.
61
ENGLISH

UNIT

2
SUPPLEMENTARY
10
Zigzag
- Asha Nehemiah

SUMMARY

The short story ‘Zigzag’ was written by Asha Nehemiah. It is about a pet which makes commotion
Zigzag

at home. Dr. Ashok T. Krishnan was a child specialist. His clinic was always with the noise of children.
One day he informed his wife that his friend Somu was going to Alaska and his pet bird Zigzag would be
left with them for a few days. His daughter Maya and his son Arvind were interested to keep the bird.
Dr. Krishnan explained to them that Zigzag was a lovely bird which was given to Somu by an African
witch doctor. Mrs. Krishnan was not much convinced and she said that it would give a lot of problems
to them. Somu had given them an insect eating plant, a pen knife and a boomerang. Mrs. Krishnan
was not happy with any of the gifts given by Somu.

It could
talk and sing in about twenty one different languages. Mrs. Krishnan was preparing paintings for an
exhibition next week. Within a short time Zigzag was brought in by Visu the old cook. It was a strange
bird with different coloured feathers. It was a foot and a half tall, its head had a crown of pink feathers
and the rest of its plumage had different shades. Its curved beak was sunflower yellow. Visu talked
high abut Zigzag. He also reiterated that the bird could talk well.

Arvind started a conversation with the bird but the bird did not say a single word. When some
Supplementary

nuts and fruits were brought for it, the bird picked some of them and flew to chandelier and fan. It
kept the nuts and fruits on them. Then it perched on a curtain rod and slept. Not only that, it began
to snore. The snoring was so loud that it affected everyone. They tried to wake him up but they failed
to do so. So they went to their bed room and shut them up there. When Lakshmi came she put on the
fan. All the nuts and pieces of fruits fell down. Some of them fell on the master piece of Mrs. Krishna,
sunset at Marina. The painting had orange pulp; and shiny black seeds all over it. By the time some
of the neighbours came in to complain about the snoring of Zigzag.

Mrs. Krishnan phoned to Dr. Krishnan and he came home immediately. He sent an email message
to Somu asking him how to stop Zigzag from snoring. They waited for six days to get the answer. Mrs.
Krishnan had lost interest in painting. Seventh day Somu’s email answer came. He said that Zigzag
UNIT 2

would hardly ever sleep. If they could not manage it, they could give it to Visu. Mrs. Krishnan wanted
to send the Zigzag immediately but it would be difficult to trace Visu. She had asked some experts to
come home to select her paintings for the exhibition. She did not want Zigzag to be there by the time.
So Dr. Krishnan took it to his clinic in his car and made it sit in his car.

Zigzag found its way to the clinic and started controlling the patients. It asked the people to sit
62 in their places. Dr. Krishnan was surprised. The bird also looked happy. It was trained to do this job by
ENGLISH

the witch doctor and Dr. Somu. His clinic was in perfect order. Dr. Krishnan was enjoying the presence
of Zigzag. By the time, Mrs. Krishnan phoned to inform Dr. Krishnan that her sunset at Marina was
bought by the art critic for Rs. 5000. The critic liked the touch of the orange colour. She was laughing
happily. They decided to keep the Zigzag with them.
10
B.P.No. 56
GLOSSARY

aboriginal (adj.) - native, local


sarcasm (n) - use of irony to mock or convey contempt
fringed (v) - bordered
plumage (n) - a bird’s feather collectively
sludgiest (adj.) - wet mud

Zigzag
grumpiness (adj.) - bad tempered

squirt (n) - spray


coyote (n) - a wolf like wild dog native to North America
streaks (n) - line, strap
tantrum (n) - outburst, flare-up
crinkly (adj.) - wrinkly

A.

1. ‘Even though I clearly said no!’ Ans: Dr. Krishnan


2. ‘The one that spits deadly poison straight into its opponent’s eyes.’ Ans: Maya
3. ‘Remember the tiny penknife he gave me last year’. Ans: Arvind
4. ‘It’s Somu’s thoughtless ways that reduce me to tears’ Ans: Mrs. Krishnan
5. ‘Come in, Zigzag, come in dear!’ Ans: Visu

Supplementary
B. Read the story again and write how these characters reacted in these situations:

1. You’re both quite mistaken.


Dr. Krishnan said that Zigzag was a harmless bird and it was a treasure of Somu.
Mrs. Krishnan felt very unhappy to keep Zigzag with them.

2. It’s Somu’s thoughtless ways that reduce me to tears.


Mrs. Krishnan was very unhappy to have Zigzag with them.
Dr. Krishnan wanted to maintain Zigzag because he was asked by another doctor Dr. Somu
UNIT 2

3. Just wait till zigzag settles down in this new home.

Visu said that Zigzag could speak well. Arvind started a conversation with Zigzag but it did not open
its mouth. Visu wanted to pacify the children.

Aravind and Maya were dejected that Zigzag did not speak a word.
63
ENGLISH

4. Zigzag hardly ever sleeps.

Somu in his email message said that Zigzag did not sleep well. He was in Alaska so he asked
Dr. Krishnan to give Zigzag to Visu.
10 Dr. Krishnan could not find Visu so he took it to his clinic.

5. You are an absolute treasure………….

Dr. Krishnan was happy to keep Zigzag in his clinic.

Zigzag was happy to receive the toffee from Dr. Krishnan.

C. Complete the given tabular column.

Somu requested Mrs. Krishnan was not


Zigzag

She was worried about


Arrival of zigzag Dr. Krishnan to take happy to keep the bird
her paintings.
care of his pet. with them.
When their maid
Life of Zigzag at Mrs. Krishnan was
Zigzag perched on the switched on the fan the
annoyed and called Mr.
Dr. Krishnan’s curtain rod and slept nuts and the pieces of
Krishnan to send Zigzag
residence and snored. fruits fell all over the
immediately to Visu
floor.
was Zigzag
The email about Somu’s reply surprised would hardly sleep. They
Dr. Krishnan
Zigzag the Krishnans. could send the bird to
Visu.
When Zigzag entered Gone was Zigzag’s After the family knew that
the clinic he started bored and grumpy zigzag must be kept busy
Zigzag at the clinic
controlling the expression. The bird they wanted to keep it
Supplementary

patients. looked happy and alert. with them.

D. Answer the following question in one or two sentences: B.P.No. 57

1. Why did Dr. Ashok’s cousin call him?


His cousin wanted to go to Alaska. He wanted Dr. Ashok to take care of his pet bird Zigzag.

2. Mention at least two expressions which show that Mrs. Krishnan was not willing to have
Zigzag at home.
Mrs. Krishnan said that whatever was given by Somu were nuisances. She gave a sarcastic comment
when Arvind praised the boomerang given by Somu
UNIT 2

3. What other various pets did Somu have?


Somu had an green and gold fighting beetle and insect eating plant.

4. What was Mrs. Krishnan busy with?


Mrs. Krishnan was busy with her paintings for an exhibition

64
ENGLISH

5. What commotion did the boomerang cause in the neighbourhood?


The boomerang sliced many TV aerials and damaged many cars.

6. What happened when Somu left Zigzag with the Krishnans?


Zigzag started sleeping and snoring when Zigzag was with the Krishnans.
10
7. How did Zigzag communicate with the Krishnans?
Zigzag communicated with the Krishnans with the movement of eyes.

8. What was the e-mail message sent to Somu by Dr. Krishnan?


Dr. Krishnan sent the email message to Somu asking him for clear instruction on how to stop Zigzag
from snoring.

9. What did Aravind confess?

Zigzag
Arvind confessed that for the first time in his life he was actually looking forward to going to school to
avoid the snoring of Zigzag.

10. Why did Mrs. Jhunjhunwalla buy the painting?


Mrs. Jhunjhunwalla was the art critic and she liked the orange stroke in the painting, sunset at Marina.

E. Answer the following questions in about 100-150 words:


1. Write a passage in your own words on various commotions caused by Zigzag at Dr.

about a pet – Zigzag – Visu the cook – strange bird – talk well – Arvind’s conversation
– failed – nuts and fruits kept on chandelier and fan – perched on curtain rod – snored –
nuts and fruit pieces fell down – spoilt sunset at Marina – orange pulp and black seeds –
complaints from neighbours

Title Zigzag

Supplementary
Author Asha Nehemiah
Theme Comic Commotions of a pet

The short story ‘Zigzag’ was written by Asha Nehemiah. It is about a pet which made
commotion at home. Zigzag was brought in by Visu the old cook. It was a strange bird with different
coloured feathers. Visu talked high about Zigzag. He also reiterated that the bird could talk well. Arvind
started a conversation with the bird but the bird did not say a single word. When some nuts and fruits
were brought for it, the bird picked some of them and flew to chandelier and fan. It kept the nuts
and fruits on them. Then it perched on a curtain rod and slept. Not only that, it began to snore. The
snoring was so loud that it affected everyone. They tried to wake him up but they failed to do so. So
UNIT 2

they went to their bed room and shut them up there. When Lakshmi, the house maid came she put
on the fan. All the nuts and pieces fell down. Some of them fell on the master piece of Mrs. Krishna,
sunset at Marina. The painting had orange pulp and shiny black seeds all over it. By the time some of
the neighbours came in to complain about that snoring of Zigzag.

65
ENGLISH

2. What happened when Zigzag was taken to the clinic.


about pet – Zigzag – Visu the cook – strange bird – could talk well – Arvind conversation
failed – nuts and fruits kept on chandelier and fan – nuts and fruit pieces fell down – spoilt
10 sunrise at Marina – Dr. Krishnan took to clinic – controlled the patients – talked well –
phone from Mrs. Krishnan – sunrise at Marina sold for Rs. 5000 – orange colour and black
seed
Title Zigzag
Author Asha Nehemiah
Theme Comic Commotions of a pet

The short story ‘Zigzag’ was written by Asha Nehemiah. It is about a pet which made
commotion at home. Zigzag was brought in by Visu the old cook. It was a strange bird with different
coloured feathers. He reiterated that the bird could talk well. Arvind started a conversation with the
Zigzag

bird but the bird did not say a single word. When some nuts and fruits were brought for it, the bird
picked some of them and flew to chandelier and fan.When Lakshmi, the house maid came she put on
the fan. All the nuts and pieces fell down. Some of them fell on the master piece of Mrs. Krishna,
sunset at Marina. The painting had orange pulp and shiny black seeds all over it. She could not
tolerate it anymore. R. Krishnan took the bird to his clinic and left it in his car. It went to the clinic and
controlled the patients. He was happy with the bird. Mrs. Krishnan phoned to Dr. Krishnan to come
immediately. When he came home he could not convince his wife. So he took the bird to his clinic.

about pet – Zigzag – Visu the cook – strange bird – could talk well – Arvind conversation
failed – nuts and fruits kept on chandelier and fan – nuts and fruit pieces fell down – spoilt
sunrise at Marina – Dr. Krishnan took to clinic – controlled the patients – talked well –
phone from Mrs. Krishnan – sunrise at Marina sold for Rs. 5000 – orange colour and black
seed
Supplementary

Title Zigzag
Author Asha Nehemiah
Theme Comic Commotions of a pet

The short story ‘Zigzag’ was written by Asha Nehemiah. It is about a pet which made
commotion at home. Dr. Ashok T. Krishnan was a child specialist. One day his friend Somu asked him
to keep his pet Zigzag for some days. Zigzag was brought in by Visu the old cook. It was a strange
bird with different coloured feathers. He also reiterated that the bird could talk well. Arvind started
a conversation with the bird but the bird did not say a single word. When some nuts and fruits were
brought for it, the bird picked some of them and kept them on the chandelier and fan. Then it perched
UNIT 2

on a curtain rod and slept and snored. The snoring was so loud that it affected everyone. When
Lakshmi, the house maid put on the fan. All the nuts and pieces fell down. Some of them fell on the
master piece of Mrs. Krishna, sunset at Marina. The painting had orange pulp and shiny black seeds all
over it. . She could not tolerate it anymore. Dr. Krishnan had to come home. He could not convince
his wife. So he took the bird to his clinic. He kept it in his car but Zigzag found its way to the clinic
and started controlling the patients. Dr. Krishnan was surprised. The bird also looked happy. It was
66 trained to do this job by the witch doctor and Dr. Somu. His clinic was in perfect order. By the time,
ENGLISH

Mrs. Krishnan phoned to inform Dr. Krishnan that her sunset at Marina was bought by the art critic for
Rs. 5000. The critic liked the touch of the orange colour. She was laughing happily. They decided to
keep the Zigzag with them.
10
ADDITIONAL PARAGRAPH
1. Describe the bird Zigzag.
about pet – Zigzag – Dr. Krishnan a child specialist – Somu his friend – left Zigzag – a
lovely bird – given by African witch doctor -Visu the cook – strange bird – different colour
- could talk well – foot and a half tall – crown of pink curved beak sunflower yellow

Title Zigzag

Zigzag
Author Asha Nehemiah
Theme Comic Commotions of a pet

The short story ‘Zigzag’ was written by Asha Nehemiah. It is about a pet which made
commotion at home. Dr. Ashok T. Krishnan was a child specialist. His clinic was always with the noise
of children. One day he informed his wife that his friend Somu was going to Alaska and his pet bird
Zigzag would be left with them for a few days. His daughter Maya and his son Arvind were interested
to keep the bird. Dr. Krishnan explained to them that Zigzag was a lovely bird which was given to Somu
d cook. It was a
strange bird with different coloured feathers. It was a foot and a half tall, its head had a crown of pink
feathers and the rest of its plumage had different shades. Its curved beak was sunflower yellow. Visu
talked high abut Zigzag. He also reiterated that the bird could talk well in more than twenty languages.

ADDITIONAL EXERCISES

Supplementary
I. Rearrange the following sentences in coherent order.

I. 1. Zigzag was a special bird.


2. Dr. Krishnan was a child specialist.
3. Zigzag had colourful feathers.
4. Somu decided to keep his pet bird Zigzag with Dr. Krishnan.
5. It could speak well. Answer: 2, 4, 1, 3, 5

II. 1. Zigzag kept the nuts and fruits on chandelier and fan.
UNIT 2

2. Arvind started a conversation but the bird did not speak.


3. Visu, the cook brought Zigzag.
4. They gave nuts and fruits to Zigzag.
5. Maya and Arvind were happy to receive the bird. Answer: 3, 5, 2, 4, 1

67
ENGLISH

III 1. The painting fetched Rs. 5000 because of the touch up given by the mistake of Zigzag.
2. Dr. Krishnan’s family did not like it.
3. Mrs. Krishnan was happy with the act of Zigzag.
10 4. Because of Zigzag the painting of Mrs. Krishnan was spoilt.
5. Zigzag was brought to Dr. Krishnan’s house. Answer: 5, 2, 4, 1, 3

II. Identify the character.


1. ‘Don’t worry, children.’ – Visu
2. ‘I thought it was scientific fact that birds couldn’t snore.’ – Maya
3. ‘May I take Zigzag to school, Papa?’ – Arvind
4. ‘Great hit indeed!’ – Mrs. Krishnan
5. ‘African witch doctor’s birds don’t obey scientific rules.’ – Aravind
Zigzag

6. ‘It’s all because my patients were making much noise and crying.’ – Dr. Krishnan
7. ‘Isn’t that Uncle Somu’s prized giant green-and-gold fighting beetle?’ – Maya
8. ‘The beetle is called Spitefire.’ – Aravind
9. ‘She’s bought it for herself, for Rs. 5000!’ – Mrs. Krishnan

III. Comprehension.
Read the following passage and answer the questions given below.

1. They froze in horror. Lakshmi had apparently switched on the fan on which Zigzag had left
some fruit and nuts. Half-pecked fruit streamed of the fan, dampening even Lakshmi’s enthusiasm as
a guava landed on her cheek with a soft squish and one walnut hit her forehead with a loud smack.
One slice of over-ripe papaya came whizzing off the fan and, as they watched it helplessly, it oh horrors
splattered all over Mrs. Krishnan’s unfinished masterpiece, sunset at Marina, spreading streaks of
gooey orange pulp and shiny black seeds all over it. Mrs Krishnan groaned tragically and looked ready
to shoot Zigzag, but he was saved by the bell.
Supplementary

Questions :
1. Who switched on the fan?
2. What landed on Lakshmi’s cheek?
3. What is Mrs. Krishnan’s unfinished masterpiece?
4. What splattered on the unfinished masterpiece?
5. Who saved zigzag?

Answers :
1. Lakshmi switched on the fan.
2. A guava landed on her cheek.
3. Sunset at Marina is the unfinished masterpiece.
UNIT 2

4. One slice of over-ripe papaya splattered on the unfinished masterpiece.


5. The bell saved Zigzag.

IV. Passage for Comprehension.


1. Six days passed. Six frantic days of checking their e-mail day and night. Six torturous days of
68 having the deafeningly loud KNGRRDRRWHEEZE resound in their home, most nerve wrackingly. Maya
ENGLISH

complained that she heard a permanent rumbling sound in her ears even when she was miles away
from home and that her ears ached all the time. Aravind confessed that, for the first time in his life, he
was actually looking forward to going to school considering it was as calm as a monastery compared to
their house. Mrs. Krishnan had lost interest in painting. Zigzag would sometimes wake up briefly when
he wanted to eat some fruit.
10
Questions :
1. How many days did they check the e-mail?
2. What did Maya complain?
3. What did Aravind confess?
4. What had Mrs. Krishnan lost?
5. Why did Zigzag wake up?

Zigzag
Answers :
1. They checked the e-mail for six days.
2.  aya complained that she heard a permanent rumbling sound in her ears even when she was
M
miles away from home and that her ears ached all the time.
3.  ravind confessed that, for the first time in his life, he was actually looking forward to going to
A
school considering it was as calm as a monastery compared to their house.
4. Mrs. Krishnan had lost interest in painting.
5. Zigzag

2. Never had a morning passed so quietly and peacefully for Dr. Krishnan. When the last patient
had left, he called Zigzag to his room. Zigzag flew in and sat on the table. Scratching the bird under
its beak, Dr. Krishnan sighed and said, ‘Somu was right, after all. You are an absolute treasure. I never
realized what he meant when he called you a great help. Why didn’t you tell me you’d prefer to be at
my clinic instead of snoring like that to show you were bored? What do we do now? No one wants you

Supplementary
back at home now; they want me to leave you with Visu.’
Questions :
1. What did Dr. Krishnan do after the last patient had left?
2. How did Zigzag go to Dr. Krishnan?
3. What did Dr. Krishnan realize?
4. What did Dr. Krishnan say to Zigzag?
5. Who was an absolute treasure?

Answers :
1. Dr. Krishnan called Zigzag to his room.
UNIT 2

2. Zigzag flew in and sat on the table


3. Dr. Krishnan realized that Zigzag was a great help.
4. Dr. Krishnan said that Zigzag was an absolute treasure.
5. Zigzag was an absolute treasure.

69
ENGLISH

UNIT

3
PROSE
10 Empowered Women
Navigating The World
Empowered Women Navigating The World

B.P.No.59
WARM UP

Read the statements given below and match the issues accordingly.
1. The marriage of a young girl below 18.
Child Marriage.

2. A transfer of durable goods / property that the bride’s family gives to the bridegroom as a
condition of the marriage.
Dowry System.

3. The intentional killing of a baby girl due to the preference for male babies in the family.

4. Repeated and unwelcome sexual comments, looks, or physical contact at work place made
by men that could offend women.
Sexual Harassment.

5. Women deciding to remarry despite opposition from relatives /society.


Remarriage.

6. Women excelling in many fields overcoming many hurdles.


Women Empowerment.
Prose

Discuss with your friend the role of women in building a family. What are the roles played
by her?
Women play a variety of significant roles in the family from their birth till the end of life.
Woman’s life is more complicated than a man. She has to take care of herself and family members as
daughter, grand daughter, sister, daughter-in-law, wife, mother, mother-in-law, grandmother, etc. By
following such a big responsibility in the family, they come out and do job for the bright future of their
family and country.
UNIT 3

What do you think of the status of women in the modern society?


In the modern society women are enjoying perfect equality with the men. They are no longer
a slave of man’s passion. They are not living at the mercy of man. They are asserting their rights and
position in shaping the destiny of country. They outshine men in certain fields. They enjoy the right to
ownership of property. They actively participate in politics. They hold highest positions in political and
70 administrative domains. Some of them are CEO’s of top-notch banks and companies.
ENGLISH

Compare the status of women in the past with the present.


The women in the past were denied education and employment. The evil practice like
downy system., child marriage, widowhood, sati etc hindered their progress. In fact they were confined
to the four walls of home. But now, they enjoy equal rights with men. They perform their best as CEO’s, 10
astronauts, scientists and so on. They can participate in politics and occupy the highest position unlike
in the past.

What is the role of women in the modern society? Discuss.

Empowered Women Navigating The World


Women are important in our society. Every woman has her own job or duty in the modern
society. We can´t forget that women´s life is a lot more complicated than a man´s life. A woman has
to take care of her own personal life and if she is a mother, she has to take care of her children´s life
too. Married women have lots of worries and believe it or not, they carry out a more stressful life than
married men. She is the one that cares about the order, the matter and the health of the family.

Divide the class into small groups, choose any one eminent woman personality of the
world. List out her characters, achievements, etc., and speak a minute about her.

Indra Nooyi is an Indian-born American business executive currently serving as the Chairperson
and Chief Executive Officer of PepsiCo, one of the largest food and beverage businesses in the world.

Childhood & Early Life


• She was born as Indra Krishnamurthy on 28 October 1955, in Madras (now Chennai), Tamil

• She was educated at Holy Angels Anglo Indian Higher Secondary School. She graduated in 1974
from Madras Christian College.
• She completed a Post Graduate Diploma in Management (MBA), in 1976.

Career
• Indra Nooyi joined PepsiCo in 1994 as senior vice president of corporate strategy and development.

Awards & Achievements

Prose
• She has been consistently ranked on ‘Forbes’ magazine’s list of The World’s 100 Most Powerful
Women. ’Fortune’ magazine named Indra Nooyi No.1 on its annual ranking of Most Powerful
Women in business for five consecutive years from 2006 to 2010.
• She was honored with the Padma Bhushan by the President of India in 2007.
• She was named 2009 CEO of the Year by Global Supply Chain Leaders Group.

SUMMARY
UNIT 3

The lesson ‘Empowered Women Navigating the World’ is about the power of women which is
in no way inferior to that of man. This lesson describes the expedition undertaken by four women from
Indian Navy to circumnavigate the world. In the past women were not permitted to go out without
an escort. Now in the modern world the real power of women is projected in all walks of life. These
women undertook the expedition in the Indian Naval Ship, Tara-Traini. It has got many special features.
One important feature is that it uses non-conventional renewable energy such as the wind. 71
ENGLISH

The all women crew members were Captain Lt. Commander Vartika Joshi, Lt. Cdr. Pratibh
Jawal, Lt. Cdr. Swathi Patarapalli, Lt. Aiswarya Boddapatti, Lt. Sh. Vijaya Devi and Payap Gupta. The
expedition was flagged off on 10 September 2017 from Goa by the Defence Minister. After going

10 through the whole world they reached Goa in April 2018 spending 254 days. They shared their
experience in an interview.

None of them was acquainted with a sail boat. They had three years training to know
Empowered Women Navigating The World

how to use the sail boat. They were given hands-on training like, how to repair things, how to deal
with emergencies and tactical aspects. Before taking the expedition they sailed on INSV Mhadei
to Mauritius and Cape Town. They had good experience in managing repair work, water and food.
These six personnel were shortlisted from thirty women who had applied for this expedition. Their
family members had apprehensions but when they knew that they could manage themselves well they
supported them.

They wanted to complete the journey honestly following all the rules and regulations. They
did not switch on the engine at all. Everyone had her own way of dealing with situations. So they could
discuss among themselves to find out the best way to solve a problem. Team work was very important.
Everyone had to do her job. When the winds were strong it was difficult to navigate. They encountered
a severe storm in the South Pacific but they managed it well. When something broke down they fixed
it together.

w the
lights emitted by living organisms, swimming dolphins and variety of sea creatures. They saw a dead
sperm whale which looked like an island. During their journey they used to post pictures of their food,
cake etc. They had time to read books and to do some craft work. They watched movies and listened
to music. They knew that the entire country wanted them to complete the journey successfully. It was
a great honour to complete the journey. They felt that it was a great achievement for all women of
India.

B.P.No. 65
GLOSSARY
Prose

circumnavigate (v) – to travel all the way around something, especially the Earth
indigenously (adv.) – naturally; innately; inherently
consonance (n) – agreement or compatibility between opinions or actions
skippered (v) – acted as a master or captain of a vessel especially a small boat
expedition (n) – a journey or voyage made for some specific purpose, such as war or
exploration
UNIT 3

replenishment (n) – restoration of a stock or supply to a former level or condition


apprehensive (adj.) – anxious or fearful that something bad or unpleasant will happen
contention (n) – strenuous effort; struggling together in opposition
auxiliary (adj.) – additional; used as a reserve or substitute in case of need
anticipate (v) – to foresee; to realize beforehand; to expect; be sure of
72
ENGLISH

bio-luminescence (n) – the production of light by living organisms


golgappas (n) – the other term for pani puri
morale (n) – emotional or mental condition with respect to confidence especially in
the face of hardships 10
ADDITIONAL

Empowered Women Navigating The World


escort – bodyguard crisis – disaster
set – kept nightmare – frightening
currently – at present monitoring – supervising
advancement – progress expedition – journey undertaken
accomplishments – achievements by a group of
people for a
tremendous – wonderful
particular purpose
patron – supporter
acquainted – familiar
deity – god/goddess
extensive – wide ranging combat – fight

attain – achieve apprehensive – anxious


ultimate – final
potential – capacity
ping
essential – necessary
repulsion – refusal
depicting – describing
thrust – drive grasped – grabbed

initiative – push poked – thrust


collaborate – cooperate
meteorological – atmospheric
crisis – disaster
accurate – precise, correct
nightmare – frightening
marine – sea
anticipate – foresee

Prose
globally – worldwide
stereotypes – conventional encountered – faced
blissful – delightful
hardship – difficulties
absolutely – definitely
acquainted – informed, familiar
awestruck – fascinated
expedition – voyage
prediction – forecast entire – fully

emergencies – dangers spotted – noticed


UNIT 3

indulged – involved
tactical – strategic rustled – rushed
aspects – features discriminate – differentiate
various – several morale – confidence
showcase – project
collaborate – cooperate
73
ENGLISH

EXERCISE WITH ANSWER

10 Choose the appropriate synonyms of the word underlined.

1. Women in India were permitted to go out with an escort.


a) bodyguard b) father c) brother d) husband
Empowered Women Navigating The World

2. The real power of women is currently being projected by the advancement of technology
a) past b) always c) presently d) never

3. The real power of women is currently being projected by the advancement of technology.
a) sight b) right c) implement d) progress

4. Tara – Tarini is the patron deity for sailors.


a) person b) supporter c) player d) powerful

5. After undergoing extensive sea trials, she was commissioned to the Indian Naval Service.
a) distant b) severe c) playful d) wide-ranging

6. The project is depicting Government of India’s thrust of Nari Shakti.


a) drive b) feeling c) wish d) keeping

a.
a) water b) river c) sea d) people

8. How well were you acquainted with the sail boat before you took up the task?
a) accepted b) rejected c) tried d) familiar

9. Training was needed in tactical aspects as well.


a) strategies b) features c) information d) exclusion

10. We spottted a dead sperm whale once.


a) threw b) touched c) jumped d) noticed
Prose

Answers
1. a 2. c 3. d 4. b 5. d 6. a 7. c 8. d 9. b 10. d

EXERCISE for SELF EVALUATION

Choose the appropriate synonyms of the word underlined.


UNIT 3

1. The sea does not discriminate between genders.


a) accept b) refused c) distinguish d) lessen Ans:

2. I’d say it was easier for us to collaborate and work together.


a) cooperate b) co-join c) co-authorise d) comingle Ans:
74
ENGLISH

3. We were absolutely awestruck as we were not expecting it to see the entire sky lit up in
green light.
a) moved b) laughing c) cursed d) fascinated Ans:

4. Team work is the most important in the middle of the crisis. 10


a) enjoyment b) disaster c) work d) journey Ans:

5. Training was needed in tactical aspects as well.

Empowered Women Navigating The World


a) strategies b) features c) improvement d) encouragement Ans:

6. It covered the expedition in five legs with stop-over for replenishment of ration and repair.
a) exhaustion b) removal c) renewal d) giving Ans:

7. The Project is essential towards promoting ocean sailing activities.


a) aiming b) necessary c) emotional d) feeling Ans:

8. Currently women’s accomplishments are tremendous in many fields.


a) importance b) places c) work d) achievements Ans:

9. Currently women’s accomplishments are tremendous in many fields.


a) wonderful b) important c) accepted d) refused Ans:

10. It is a 55 foot sailing vessel built indigenously in India.


a) foreigners b) workers c) artisans d) domestically Ans:

National Policy 11.


to empower women.
a) detention b) agreement c) connection d) improvement Ans:

12. It is depicting Government of India’s thrust for Nari Shakti.


a) accepting b) throwing c) describing d) involving Ans:

13. The six member women crew broke many stereotypes during their record-setting sail.
a) conventional b) importance c) connections d) ideas Ans:

Prose
14. So my contention was to make sure that we go by the rules of circumnavigation.
a) idea b) thinking c) expansion d) argument Ans:

15. Six of us were shortlisted based on the little survival skills we showcased.
a) informed b) projected c) cleared d) accepted Ans:

16. They were apprehensive and supportive too.


a) anxious b) written c) spoken d) left Ans:

17. It was a blissful experience.


UNIT 3

a) difficult b) interesting c) delightful d) comic Ans:

18. You don’t have to use any auxiliary means of repulsion.


a) negative b) positive c) clear d) helping Ans:

19. You don’t have to use any auxiliary means of repulsion.


a) refusal b) attraction c) collection d) repetition Ans: 75
ENGLISH

20. We encountered a storm where the seas were almost nine to ten meters high.
a) encouraged b) discouraged c) surprised d) faced Ans:

10 ANTONYMS

remained × left
Empowered Women Navigating The World

currently × past
safety × danger
success × failure
indigenous × foreign

extensive × limited
consonance × discord
attain × failed
essential × needless
encouraged × discouraged

collected × scattered
accurate × incorrect

acquainted × unfamiliar
emergencies × calmness

rough × soft
apprehensive × composed
repulsion × attraction
grasped × released
different × same
Prose

brilliant × dull
rare × common
absolutely × partially
entire × partly
appreciate × blame
UNIT 3

EXERCISE WITH ANSWER

Choose the appropriate antonyms of the word underlined.

1. Women in India remained indoors unless permitted to go out with an escort.


76 a) remind b) stayed c) accepted d) left Ans : d)
ENGLISH

2. After undergoing extensive sea trials she was commissioned to the Indian Navy service.
a) limited b) wide-ranging c) important d) unimportant Ans : a)

3. It encouraged use of environment friendly non-conventional renewable energy.


a) miscouraged b) uncouraged c) discouraged d) imcouraged Ans : c) 10
4. We were given training to deal with when the weather gets rough.
a) tight b) soft c) loose d) low Ans : b)

Empowered Women Navigating The World


5. The project was undertaken to empower women to attain their full potential.
a) accept b) understand c) improve d) failure Ans : d)

6. That was a good thing because we could discuss different ways of solving a problem.
a) same b) intolerant c) imperfect d) inadequate Ans : a)

7. We were absolutely awestruck as we were not expecting it to see the entire sky lit up in
green light.
a) full b) complete c) short d) part Ans : d)

8. You don’t have to take anybody else’s assistance grasped that completely.
a) gripped b) released c) attract d) attached Ans : b)

9. We witnessed the brilliant Southern Lights from sea.


a) bright b) dull c) simple d) ruminant Ans : b)

10.
a) common b) uncommon c) acceptable d) informed Ans : a)

EXERCISE for SELF EVALUATION

Choose the appropriate antonyms of the word underlined.

1. I love it when people appreciate the food that I cook.

Prose
a) praise b) blame c) appropriate d) rise Ans :

2. You don’t have to use any auxiliary means of repulsion.


a) refusal b) disgust c) dislike d) attraction Ans :

3. They were apprehensive and supportive too.


a) composed b) anxious c) doubtful d) intolerant Ans :

4. We were absolutely awestruck to see the entire sky lit up in green light.
UNIT 3

a) wholly b) completely c) partly d) partially Ans :

5. It collected wave data on regular basis for accurate weather forecast.


a) scattered b) gathered c) noted d) rejected Ans :

6. It collected wave data on regular basis for accurate weather forecast.


a) true b) correct c) incorrect d) unfulfilled Ans :
77
ENGLISH

7. It covered the expedition in five legs with stop-over for replenishment of ration and repair.
a) renewal b) depletion c) restoration d) rebirth Ans :

8. How well were you acquainted with the sail boat?


10 a) familiar b) accustomed c) impossible d) unfamiliar Ans :

9. We were given hands on training to deal with emergencies.


a) calmness b) anxiety c) disaster d) trauma Ans :
Empowered Women Navigating The World

10. The Project is considered essential towards promoting ocean sailing activities in the Navy.
a) necessary b) needful c) needless d) crucial Ans :

11. Navika Sakar Parikarma was a project undertaken in consonance with the National Policy
to empower women.
a) agreement b) discord c) arrangement d) treaty Ans :

12. It is a 55 foot sailing vessel built indigenously in India.


a) skillful b) domestic c) internal d) foreign Ans :

13. Tara-Tarani is worshipped for safety and success at sea.


a) danger b) protection c) involvement d) remaining Ans :

14. Tara-Tarani is worshipped for safety and success at sea.

15. The real power of women is currently being projected to the world.
a) presently b) next c) latter d) past Ans :

TEXTUAL QUESTIONS and ANSWERS

a. What does INSV stand for? B.P.No.60


INSV stands for Indian Naval Ship Vessel.
Prose

b. When was INSV Tarini commissioned to Indian Navy service?


INSV Tarini was commissioned to Indian Navy service on 18 February 2017.

c. Who is Tara - Tarini?


Tara-Tarini is the patron deity for the sailors.

d. Where did the crew undergo their basic training?  B.P.No.63


The crew underwent their basic training in Mumbai at the Indian Naval Waterman ship Training Centre
and at various schools in the naval base in Kochi.
UNIT 3

e. How long were they trained to undertake this voyage?


They were trained for three years to undertake this voyage.

f. Which skill was considered important in the selection process?


Survival skill was considered important in the selection process.
78
ENGLISH

g. Who mentored the crew?


Commander Dilip Donde mentored the crew.

B.P.No.65
h. Which quality of the skipper helped to bring out a successful expedition?
Honesty of the skipper helped to bring out the successful expedition.
10
i. Who among the crew mentioned about teamwork?
Payal Gupta mentioned about teamwork.

Empowered Women Navigating The World


j. When did they witness the brilliant southern lights from the sea? How did the sky appear
there?
When they crossed the Tasman Sea, they witnessed the brilliant Southern lights from the sea. The sky
appeared green.

k. What festival did they celebrate during their expedition?


They celebrated Diwali during their expedition.

A. Read the statements given below and state whether they are true or false. If false,
then write the correct answer in the space given. B.P.No.65

1. Indian Navy’s all-women crew was the first-ever to circumnavigate the globe. Ans: True

2. The crew consists of six members of men and women Indian Navy service. Ans: False

3. Vartika Joshi skippered the crew to circumnavigate the globe. Ans: True

4. The crew started their expedition on 10 July 2017 from Mumbai. Ans: False
10 September from Goa

5. Dilip Donde was the first person to go on a non-stop solo circumnavigation. Ans: False
Dilip Donde was the first Indian to go on a non-stop solo circumnavigation.

B. Answer the following questions briefly.  B.P.No.66

Prose
1. Mention the special features of INSV Tarini.
One of the special features of INSV Tarini is that it encouraged use of environment friendly non-
conventional renewable energy resources such as the wind. It collected and updated meteorological,
ocean and wave data on regular basis for accurate weather forecast by India meteorological Department
(IMD) and also collected data for monitoring marine pollution on high seas.

2. What does the term circumnavigation mean?


Circumnavigation means to travel all the way around something, especially the earth.
UNIT 3

3. How did the all-women Indian Navy crew go about their voyage?
The crew started their voyage on 10 September from Goa, flagged off by the Defence Minister of India.
It covered the expedition in five legs with stopovers at four ports for refilling of ration and repair as
necessary, before returning to Goa in April 2018.

79
ENGLISH

4. When did the crew start their voyage? When did they return to India? How many days did
it take to complete the expedition?
The crew started their voyage on 10 September 2017 from Goa. They returned to India in April 2018.

10 It took 254 days to complete the expedition.

5. What sort of training did the crew undergo before their expedition?
The crew underwent some theoretical courses on navigation, communication and weather prediction.
Empowered Women Navigating The World

They were also given hands-on training as to how to repair things and deal with emergencies, when
the weather gets tough.

6. How did the crew members work as a team to make their expedition successful?
The crew members really worked as a team, with each one doing different tasks. Vartika Joshi, as the
head of the crew, would discuss different ways of solving a problem and chose the best one. Teamwork
was the most important in the middle of the crisis. When some people were out to watch, others were
inside doing work like heating water and heating the gloves.

7. What challenging tasks did the team face during their voyage?
In the South Pacific, the team encountered a storm where the seas were almost nine to ten metres
high and winds were picking up to 60-70 knots. The voyage was adventurous when they were crossing
the Tasman Sea.

8. What sort of activities did the crew engage in during their long voyage?
ks and did quilling
and craft work. While Joshi read comics and the Ramayana, she loved cooking and baking. Vartika
Joshi said they watched movies, listened to music and baked cakes, breads, halwa, etc. They even
celebrated Diwali and first birthday of the boat.

9. Mention the celebrations which the crew enjoyed during their expeditions.
The crew celebrated Diwali, three birthdays including the first birthday of the boat and specific occasions
like crossing the equator, the International Date Line and so on.

10. Which factor motivated the crew to undertake this expedition?


Prose

As the sea does not discriminate between genders, the crew undertook this expedition to boost the
morale in the country and for more women to take to adventures like sailing. It was this factor that
motivated the crew to undergo this expedition.

ADDITIONAL

Short Answer Questions.


UNIT 3

1. What was so special about Indian Navy’s all-women crew?


Indian Navy’s all-women crew was the first-ever to circumnavigate the globe skippered by Lt.
Commander Vartika Joshi.

2. How long did the circumnavigation of the globe take? Where and when did they return?
80 The circumnavigation of the globe took 254-days. They reached Goa port on 21 May 2018.
ENGLISH

3. What was the training given to them besides the theoretical courses?
The team was given hands-on training like how to repair things and how to deal with emergencies,
when the weather gets rough.

4. Why were out of the thirty women who had applied, six of them shortlisted?
10
Six of them were shortlisted because they showcased their survival skills.

5. What hobbies did they pick up during circumnavigation?

Empowered Women Navigating The World


They picked up hobbies like posting pictures of delicacies like golgappas and cakes, reading books and
some quilling and craft work.

6. What hobby did Vartika Joshi cultivate during her journey?


Vartika Joshi read comics and the Ramayana during her journey. 00000

C. Answer the following in about 100-150 words:

1. Highlight the factors responsible for all-women Indian Navy crew to carry out their
expedition.

Prose Empowered Women Navigating the World


Theme Adventure is synonymous with woman

Women of the past stayed indoors unless allowed to go out with an escort. But today
things have changed in such a way that the real power of women is being projected to the world
by the advancement of technology and media. Currently women’s achievements are tremendous in
many fields. One such achievement is the All-women Indian Navy crew who sailed around the world
for 254 days, in a sailboat called INSV Tarini. As the sea does not discriminate between genders, the
crew undertook this expedition to boost the morale in the country and for more women to take to
adventures like sailing. The crew members worked as a team, with each one doing different tasks.
Vartika Joshi, as the head of the crew, would discuss different ways of solving a problem and chose

Prose
the best one. Payal Gupta added that teamwork is the most important in the middle of the crisis. This
team effort helped in navigating through the 20 hours long storm. These factors motivated the crew to
undergo this expedition.

The boat was named after the famous ‘Tara-Tarini’ temple in Ganjam district in Odisha

2. Write in detail about the selection and training process which the crew underwent.

Prose Empowered Women Navigating the World


UNIT 3

Theme Adventure is synonymous with woman

We were given hands-on training, like how to repairs things and


how to deal with emergencies.

As regards the selection process, out of the thirty women who had applied, six of them
were shortlisted, based on the survival skills they showcased. The crew was mentored by Commander 81
ENGLISH

Dilip Donde. Then they got acquainted themselves with sail boat. They slowly got used to it through
three years of training. Before the expedition, they started with some theoretical courses on navigation,
communication and weather prediction. They were also given hands-on training as to how to repair

10 things and deal with emergencies, when the weather got tough. In addition, Aishwarya said that they
underwent their basic training courses in Mumbai at the Indian Naval Waterman ship Training Centre
(INWTC), and at various schools in the southern naval base in Kochi. They even sailed on INSV Mhadei
to Mauritius and back and also to Cape Town in December 2016. This trip was on how to manage food,
Empowered Women Navigating The World

water and even electricity during the big voyage.

The sea, once it casts its spell, holds one in its net of wonder forever.

ADDITIONAL PARAGRAPHS

Answer the following in about 100-150 words:

1. Describe INSV Tarini.

Prose Empowered Women Navigating the World


Theme Adventure is synonymous with woman

h the
National policy to empower women to attain their full potential

The lesson ‘Empowered Women Navigating the World’ is about the power of women which
is in no way inferior to that of man. This lesson describes the expedition undertaken by four women
from Indian Navy to circumnavigate the world. In the past women were not permitted to go out without
an escort. Now in the modern world the real power of women is projected in all walks of life. These
personnel undertook the expedition in the Indian Naval Ship, Tara-Traini. INSV Tarini is a sailboat.
INSV stands for Indian Naval Ship Vessel. Tara-Tarini is the patron deity for sailors and is worshipped
for safety and success at sea. It is the second sailboat of the Indian Navy. It is a 55 foot sailing vessel
Prose

built indigenously in India by M/s Aquarius Shipyard Pvt. Ltd, in Goa. After undergoing extensive sea
trials, she was commissioned to the Indian Navy service on 18 February 2017. The boat was named
after the famous ‘Tara-Tarini’ temple in Ganjam district of Odisha. The word ‘Tarini’ means boat and in
Sanksrit it means ‘Saviour’. The boat has advance Raymarine navigation suite and an array of satellite
communication systems for perfect navigation anywhere in the world.

Blessed are the curious; for they shall have adventures.

2. What was the challenging task? How did they spend time deep in the sea?
UNIT 3

Prose Empowered Women Navigating the World


Theme Adventure is synonymous with woman

It encouraged use of environmental friendly non-conventional


renewable energy resources such as the wind.
82
ENGLISH

The expedition was undertaken by the all-women naval personnel to empower women to
attain their full potential. The challenging task while sailing was that the sea could get really tough
when the winds were picking up. In the South Pacific, they encountered a storm where the seas were
almost nine to ten metres high and winds were picking up to 60-70 knots. The voyage was adventurous
when they were crossing the Tasman Sea. They witnessed the brilliant Southern Lights from sea. The
10
entire sky was lit up in green light. They spent time deep in the sea by posting the pictures of delicacies
like golgappas and cakes, reading books and doing quilling and craft work. While Joshi was reading

Empowered Women Navigating The World


comics and the Ramayana, she was cooking and baking. Vartika Joshi said they watched movies,
listened to music and baked cakes, breads, halwa, etc. They even celebrated Diwali and three birthdays
including the first birthday of the boat.

Adventure is the best way to learn.

B.P.No. 67
VOCABULARY

Idioms and Phrases


D. Pick out the idioms and phrases from the box and write them in the blanks equivalent
to their meanings. One is done for you.
ll,
get along, hang on, over the moon, work out, sharp as a tack

IDIOMS PHRASES
1. Competent - on the ball 1. To meet or find by chance - come across
2. Become more confident in expressing oneself
2. To exercise - work out
- find one’s voice
3. To accept a challenge with confidence -
3. Extremely happy - over the moon
hang on

Prose
4. Mentally agile - sharp as a tack 4. To have a friendly relationship - get along
5. Listen - lend an ear 5. To keep something - bring it on

E. Read the given sentences carefully and fill in with appropriate phrasal verbs. Choose
them from the help box.

get along with, take off, shut down, look after, warm up

1. The air hostess instructed the passengers to wear the seat belts during the take off.
UNIT 3

2. Venkat felt happy to get along with the neighbours in the new locality.
3. There will be a shut down next week in the office.
4. Doing warm up every day in the morning keeps one healthy.
5. The mother instructed the maid to look after the child carefully.
83
ENGLISH

F. Read the given passage carefully and fill in the blanks with suitable phrasal verbs
from the help box.

burn off keep up build up tire out


10 warms up put on work out stretch out

Riya is a young dancer who feels contented and satisfied with herself. Let’s hear from her.
Empowered Women Navigating The World

Hi, everyone! I am Riya. I suppose I’m really lucky because I don’t put on weight easily. I never
work out in the gym and the only time I stretch out is when I need something from the top shelf. I tried
aerobics several times but I couldn’t keep up with the others. I take my pet for a walk thrice a day and
that helps to burn off the calories. I usually watch what I eat but I sometimes binge on icecream.

My sister Diya, is a real fitness fanatic. Before she works out she warms up every day with push
ups, sit ups, stretches and a jog around the park. She says it’s important to build up good levels of
strength and stamina. I don’t want to overdo it though. A fitness regime like hers would tire out me!

B.P.No. 68
LISTENING

G. Listen to the passage read by the teacher and say whether the given statement is true
or false.

2. At the age of seventeen, she captained the Indian Women’s cricket team. Ans: False
3. Preethi Srinivasan was not only a cricketer but also a runner. Ans: False
4.  reethi’s own trauma inspired her to create Soul Free, a foundation for
P
those suffering from mental illness. Ans: False
5. Preethi received the Kalpana Chawla Award for Courage and Daring Enterprise. Ans: True

B.P.No. 69
SPEAKING
Prose

H. Read the clues given below and develop your story. Narrate your story to the class.
Robert Bruce - King - lying on the ground in a dejected mood - failed to defeat his enemies - was
thinking of giving up the attempt - saw a spider falling down from the ceiling - the ceiling far away -
wondered how it would get there - the spider fell back again - again it tried - again it fell - it made nine
such attempts - no success - climbed up once more - at last succeeded in reaching the roof - Bruce
imitated its example - he too tried once again - was successful.
UNIT 3

The Lesson of a Spider


Once there was a king called Robert Bruce.
One rainy day, he was lying on the ground in a dejected mood. He was tired and felt sick at heart,
ready to give up all hope. It seemed to him that there was no use for him to try to do anything more.
As he lay thinking, he noticed a spider over his head, which was falling down from the ceiling.
The ceiling was far away. The king wondered how it would be reach there. It tried and fell back again
84
and again. It made nine such attempts, yet it failed. Then it made another attempt to climb up once
ENGLISH

more. At last it succeeded in reaching the roof. Bruce got inspired and he imitated what the spider did.
He too tried again and he was successful.

Moral : Try, try till you succeed.


10
I. Develop a story with the given pictures and narrate it to your class. Your story must
have a plot and vivid details.

Empowered Women Navigating The World


Once, a group of adventurous sailors went on an expedition around the world. The sail in the
beginning was joyful with a lot of fun. After sailing a few nautical miles, the ship hit an iceberg and
started to sink in the sea. The crew in the ship dived into the sea. When they went down a few hundred
metres, they were startled to see the sea organisms which were fascinating to the eyes. Further they
re joyful was the
box which contained gold coins. It had fallen into the sea some years ago. They brought it out after an
adventurous exploration into the sea.

B.P.No. 70
READING

I. Read the data below and answer the following questions.

Prose
UNIT 3

Choose the correct answer.

1. What is the data about?


a) women empowerment b) women power
85
c) women at work d) women at home Ans : c)
ENGLISH

2. Identify the three jobs where the same percentage of women work.
a) b)

10 c) d) Ans : a)

3. In which field of work is women’s involvement the second highest?


a) Logistics b) Home maker
c) Medicine d) Administration/Human resource Ans : d)
Empowered Women Navigating The World

4. Percentage of women working in finance is the same as _________.


a) Home maker b) Information Technology
c) Technical Field d) Administration/Human Resources Ans : d)

5. What is the difference between the percentage of women working in logistics and
Medicine?
a) 8 b) 11 c) 13 d) 5 Ans : d)

B.P.No. 70
WRITING

Slogan
A slogan is usually a short phrase that is easy and catchy to remember. They are often used in
advertisements and by political parties or organizations who expect people to remember what they are

J. Read the given slogans and match them appropriately with their theme.
Slogans Answer
1. One for all and all for one. Junk food Unity
2. Limit your fast food otherwise it would be your last food. Save water Junk food
3. Restricting a woman restricts the growth of the family. Cleanliness Women
empowerment
4. Clean and green makes perfect scenet. Woman Cleanliness
Prose

empowerment
5. It takes a lot of blue to stay green. Unity Save water

K. Look at the images of familiar advertisements given below. Identify the products and
try to frame your own slogans for each one of them.
UNIT 3

Cell Phone Tooth Brush

Help to clean
Enjoy the chill teeth and gums,
86 You’ll scream make them gems

ENGLISH

L. Look at the pictures given below and frame your own slogans.

10

Empowered Women Navigating The World


Work out

Letter Writing.
• A letter is an important means of communication which could be personal or official. Letters are
ave studied both
the types – formal and informal.
• Now let us learn to write a letter ordering goods and how to write a complaint for any damage
or any other problem noticed after receiving the goods.

Format of the letter

Sender’s address : (Include email and phone number, if required).


Date : (leave one space line and write the date as, 10 July 2019)
Receiver’s address : (correct address)

Prose
Subject of the letter :
Salutation (Sir / Respected Sir / Madam),
Body of the letter
(Introduce yourself and write the purpose of the letter
Mention the details of the enquiry
Conclusion)
Closure (Yours,)
UNIT 3

Sender’s signature and designation (if any)

Model of the letter:

Letter ordering things.


Ms. Deepa an NGO writes a letter to the wholesale book shop dealer, placing order for 100 copies
of medium size English Oxford Dictionary. 87
ENGLISH

Ms. Deepa,
NGO (Nallam Trust),
Kalapet village,
10 Nagappattinam District.
bdeepa04@gmail.com

13 May 2019
Empowered Women Navigating The World

The Proprietor,
NIZHAL BOOK SHOP,
Chennai – 600 001.
Sir / Madam,
Subject: Order for English Oxford Dictionaries – Reg.
On reading the discount provided by your shop in the advertisement of yesterday’s
newspaper, I would like to place an order for 100 copies of medium sized English Oxford Dictionary in
your shop. I need the copies within a week. So, let me know the estimation for the bulk order placed,
as early as possible.

Thank you,

Deepa. B

Letter of complaint
After receiving the order, Deepa finds that some of the dictionaries are damaged. So, she writes
the following letter of complaint.
Ms. Deepa,
NGO (Nallam Trust),
Kalapet village,
Prose

Nagappattinam District.
bdeepa04@gmail.com

20 May 2019

The Proprietor,
NIZHAL BOOK SHOP,
Chennai – 600 001.
UNIT 3

Sir / Madam,
Subject: Complaint about damaged dictionaries – Reg.
On receiving the order of 100 copies of English Oxford Dictionary from your shop, I found
that around 25 copies of them were damaged. In some copies the pages are missing and in some
more copies the pages are not in order. So, duly accept my complaint and replace the damaged copies.
Kindly, check the returned copies and replace them accordingly.
88
ENGLISH

Thank you,

Yours faithfully,
Deepa.B
10
Letter of Enquiry
Mr. Srinath lost his bag in an over crowed train. The following is the letter of enquiry at the
railway police force.

Empowered Women Navigating The World


Srinath B,
No.24, I cross, Pon Nagar,
Chengalpettu-10.
bsrinath16@gmail.com

25 July 2019

The Commissioner of Railway Police,


A-2 Police station, D-Nagar,
Chengalpettu-02.

Respected Madam,
Subject: Enquiry about status of the lost certificates – Reg.
ddress. I lost my
certificates on 15th July 2019, while I was returning home in a local train from Chengalpattu. As the
train was over crowded, I placed my bag on the rack above. When I was about to get off, I noticed
that my bag was missing. I was helpless and filed a complaint with the Railway Police. I have given all
the details in the complaint letter on the same day. So far I have not received any further response. I
request you to take immediate action in this regard.

Thank you,

Yours faithfully,
Srinath B.

Prose
M. Exercise : B.P.No. 74

1. Imagine that you have parked your two-wheeler / bicycle inside the school premises. You
find it missing in the evening. Write a complaint to the head of the school regarding this
issue.
From
Srikanth. D,
UNIT 3

Class X-B,
ABC High School,
Chidambaram.
10th July, 2020

89
ENGLISH

To
The Headmaster,
ABC High School,
10 Chidambaram.

Sir,
Subject: Complaint about Missing Two-wheeler –Reg.
Empowered Women Navigating The World

I would like to bring to your kind attention that I parked my two-wheeler in the parking shed around
8.30 am on 10th July 2020. When I returned in the evening, it was missing. I searched everywhere and
even enquired my friends in the school. Nobody has any idea. So I request you to restore my two-wheeler
as early as possible.

Thank you,

Yours obediently,
Srikant. D
XB

2. Write a complaint to the officer of the PWD department to take immediate action of
maintaining cleanliness in the Children’s Park in your locality.

Rakesh. S,

Natesan Nagar,
Trichy.

13th July 2020

The Officer,
PWD,
Trichy.

Dear Sir/Madam,
Prose

Subject: Lack of Cleanliness in Children’s Park - Reg.


I am a resident of Natesan Nagar, Trichy and I reside very close to the Children’s Park. I would
like to draw your attention to lack of cleanliness in the children’s park.

The garbage in the children’s park has not been cleaned for weeks. As a result the vicinity is
stinking. And every day hundreds of children go to the park and play in the evening. If the garbage is
not cleaned, the children will contract diseases like cholera, malaria, dengue etc. There is a possibility
of these disease spreading in the neighbouring areas too.
UNIT 3

Hence I request you to take immediate action in this regard.

Thank you,

Yours faithfully,
Rakesh. S
90
ENGLISH

3. Write a letter to the manager of a famous daily, ordering subscription for your school
library.

Jay Shree. L,
Librarian, 10
VET High School,
Thanjavur.

Empowered Women Navigating The World


3rd August 2020

The Manager,
M/s Kasthuribai,
Chennai.

Sir,
Subject: Subscription of Sports Star magazine for School Library - Reg.

With reference to the advertisement in the Hindu dated 13th January 2020 regarding discount, I
am writing this letter to subscribe to the Sports Star of your publications. I am placing order for3 copies
of the Magazine for one year. I have enclosed a copy of the DD towards the subscription. I request you
to send the copies by courier on the day of publication.

Thank you,

Jay Shree. L.

B.P.No. 77
GRAMMAR

Tenses:
• The term, ‘Tense’ denotes the time of action. They show when the work is done.
• The English Tenses are : 1. Past 2. Present 3. Future

Prose
A. Complete the sentences in present tense forms.
1. Saravanan always goes (go) for a walk in the morning.
2. We have gathered (gather) here for a meeting and the chair person is yet to arrive.
3. Aruna has eagerly been waiting (wait) to meet her friend since morning.
4. Sheeba is moving (move) to a new house next week.
5. Naseera attends (attend) music classes regularly.
UNIT 3

6. Ilakiya and Adhira enjoy (enjoy) each other’s company very much.
7. Mani has been working (work) in this school for five years.
8. It is pouring (pour) outside now.

B. Complete the sentences in past tense forms.  B.P.No. 80

1. I went (go) to her place on foot.


2. The children were playing (play) in the ground when the teacher arrived. 91
ENGLISH

3. They requested (request) him when the manager arrived.


4. If you had worked (work) hard, you would have won the relay match
5. Joanna and Joy had already left (leave) for Ooty, when the others reached the station.
10 6. We all sang (sing) in the choir last week.
7. Nancy asked (ask) for help.
Empowered Women Navigating The World

8. The people were waiting (wait) for the train.

C. Fill in the blanks using the verbs in the brackets in the future form.  B.P.No. 82

1. We will not go to the market, in case it rains. (go)


2. Keerthi will have done his work by next week. (do)
3. The peon will have rung the bell by the time I reach the school. (ring)
4. I will visit my sister’s house next April if I go to Uttarkhand. (visit)
5. If you listen carefully, you will understand my point. (understand)
6. By next year, I will have lived in Chennai for fifteen years. (live)
7. The new edition of this book will come out shortly. (come)

D. Underline the verbs and identify the tense forms.


1. I am working hard day and night. – present continuous tense
2. The Moon revolves around the Earth. – simple present tense
3. Were the milk men milking the cow? – past continuous tense
4. He received yours messages last night. – simple past tense
5. I have been ill for a couple of days. – present perfect tense

E. In the following passage, some words are missing. Choose the correct words from the
Prose

given options to complete the passage.


Raghav (a) was born in a middle class family. He is a (b) school-going boy of 8. His mother
(c) works as a software engineer in an MNC. (d) Drawing is his favourite hobby. He (e) won the first
prize in school level competition for drawing last week. He (f) started drawing at the age of 3. His
mother (g) hopes he (h) will become a great painter in future.

F. The following passage has not been edited. There is one error in the tense of the verb in each
UNIT 3

line. Write the wrong word as well as the correct word in the given place. One is done for you.

Incorrect words Correct words


When Anand reach Arun’s place, his reach reached
friends have arrived already. Arun have had
introduces Anand to them. Arun’s brother introduces introduced
92
ENGLISH

buy some snacks from the market. buy bought


Arun serving it to all his friends. Then serving served
they all sat together to planning their holidays. planning plan 10
Arun have a cottage in Ooty, so have had
they all plan to go to Ooty during the holidays. plan planned

Empowered Women Navigating The World


“Would we have a good time?, asked Arun. would can / will
They all cheerfully say, “Yes!” say said

G. Read the story and rewrite it using the past tense.


Juno the elephant was lonely and tried to make friends with the other animals in the forest. But,
the other animals refused to play with Juno because of his size. One day, all the animals were running
away from Dera the tiger who was eating everyone he found. Juno went and gave Dera a swift kick.
Dera immediately ran away. Juno was now everyone’s friend.

H. Read the situations given below and frame two suitable sentences in the appropriate
form of the tenses.
Give two instructions to your classmates.

• Do not throw the chocolate wrappers on the floor.

Make any two requests to your classmates or friends.


• Can you lend me your pen?
• Will you come with me to temple today?

Mention any two of your discontinued habits in the correct tense form.
• I used to eat ice-cream.

Prose
• I used to play chess in my school days.

Mention any of your two dreams in the correct tense form.


• I want to become an engineer.
• I will strive for 100% attendance this year.
UNIT 3

93
ENGLISH

UNIT

3
POEM
10
I am Every Woman
- Rakhi Nariani Shirke
I am Every Woman

SUMMARY

This poem ‘I am Every Woman’ is written by Rakhi Nariani Shirke. It talks about the multifaceted
nature of women. Today’s women are empowered, brave, strong and resolute. They are always ready
to take up new ventures. Women have to be treated respectfully for the growth of a nation.

Every woman has innate beauty in her. She is a symbol of power and strength. She is empowered,
brave and strong. She is prone to put her life at stake and does not fear in putting her life at stake.
She is ready to take risk in her life courageously. She is a down-to-earth-real human being. She is not
fake.

The poet says that the woman is an embodiment of sacrifice. She is ready to sacrifice her whims
is more important
than hers. She is hopeful of enjoying happiness even in the next cycle. This way she proves that she
is the woman who does not have fear about anything. She is hopeful even in times of adversity.

The woman is strongly and inseparably rooted in her faith and beliefs. Nothing can shake her up.
She motivates men so that one should not easily give up, rather one should keep on persevering and
hoping for better results. Though she undergoes untellable sufferings and hardships in the family, she
will not compromise on her faith and beliefs.

The poet goes on to say that she is a lioness. Never take her for granted. If you are a prankster,
she will not spare you. She will fight for equality in all spheres. We should be careful. We should not
Poem

try to outrage her pride, her self-respect and dignity. If we do so, she knows how to insult or thrash
us or outrage our dignity. Moreover, she is today’s woman, not like yester years’ woman of lesser
expressive nature. She is outspoken and candid. She expects equality in everything. So we should love
her, respect her and keep her near.

B.P.No. 85
GLOSSARY
UNIT 3

innate (adj) - inborn and natural


stake (n) - risk
persistence (n) - determination
sigh (v) - emit a long, deep audible breath expressing sadness, relief or tiredness

groans (v) - complaints and grumbles


94
moans (v) - grieves
ENGLISH

mess with (ph.v) - meddle or interfere with.


prankster (n) - a person who acts mischievously

Poetry Appreciation Questions.


A. Read the following lines and answer the questions.
10
1. The summer of life she’s ready to see in spring
She says, ”Spring will come again, my dear
Let me care for the ones who’re near.”
a) What does the word summer mean here?

I am Every Woman
Summer means adversity, pain or agony.

b) How does she take life?


She takes life positively and hopes for the better.

c) What does she mean by ”Spring will come again”?


She means that difficult times will pass by and new hope and brighter life will come up again.
2. Strong is she in her faith and beliefs.
“Persistence is the key to everything,” says she.
a) What is she strong about?
She is strong about her faith and belief.


She is persistent to find a solution for adversities in life.
3. Despite the sighs and groans and moans,
She’s strong in her faith, firm in her belief!
a) Is she complaining about the problems of life?
No, she is not complaining about the problems of life. She is highly hopeful of change of
adversities.

b) Pick out the words that show her grit.

Poem
strong, firm
4. Don’t ever try to saw her pride, her self-respect.
She knows how to thaw you, saw you –beware!
a) What do the words thaw and saw mean here?
Literal meaning of ‘thaw’ is defrosting and ‘saw’ means to cut something. Here these words mean
that she knows how to crush those who oppress her.

b) What is the tone of the author?


UNIT 3

The tone of the author is aggressive.


5. She’s today’s woman. Today’s woman dear.
Love her, respect her, keep her near…
a) Describe today’s woman according to the poet.
Today’s woman according to the poet is that she is ferocious like a lioness. She cannot be
95
threatened.
ENGLISH

b) How should a woman be treated?


A woman should be treated with love, respect and dignity.

10 ADDITIONAL

Read the following lines from the poem and answer the questions given below.
1. A woman is beauty innate,
A symbol of power and strength,
I am Every Woman

a) What is meant by ‘innate’?


‘Innate’ means inborn and natural.

b) Who is a symbol of power?


Woman is a symbol of power.
2. She’s a lioness; don’t mess with her,
She’ll not spare you if your’re prankster.
a) Identify the figure of speech employed here.
Metaphor.

b) What is meant by “Mess with”?

3. She’s today’s woman – Today’s woman, dear.


Love her, respect her, keep her near...
a) Pick out the rhyming words in these lines.
dear, near.

b) How should a woman be treated?


A woman should be treated respectfully.
4. She’s strong in her faith, firm in her belief!
Poem

a) Identify the words in alliteration.


she, strong ; faith, firm.

B. Read the lines and identify the figure of speech. B.P.No. 86

1. A woman is beauty innate,


A symbol of power and strength.
She puts her life and stake,
She’s real, she’s not fake!
UNIT 3

a) Pick out the rhyming words from the given lines.


stake, fake.

b) Add another word that rhymes with ‘strength’.


health, wealth.

c) Give the rhyme scheme for the given lines.


96 abcc.
ENGLISH

2. She’s a lioness; don’t mess with her.


She’ll not spare you if you’re a prankster.
a) Pick out the line that has a metaphor in it.
She’s a lioness. 10
b) Give your example of metaphor to describe the qualities of a woman.
She is an angel.
3. She’s strong in her faith, firm in her belief!
a) Pick out the alliterated words from the given lines.

I am Every Woman
faith, firm.
b) Pick out the alliterated words from the poem.
symbol, strength ; summer, spring

C. Fill in with a word in each blank to complete the summary of the poem. Use the help
box given below.
dignified, healthier, today’s, persistent, care, symbol, innate, fake, adversity, hope, life, disgrace, prankster,
woman, near, faith, optimistic, quitter, thaw, respect, lioness, fear, beliefs, self-respect, saw, strength
Every woman is beautiful innate. She is the symbol of power and strength. She is prone to put her
life at risk. Every woman is true in expressing her love and she is never fake. She is very dignified in
her approach even at times of adversity, she finds a ray of hope and she continues to care for her
near ones. She is the woman and she has no fear. She is forceful in her faith and beliefs. She is never
ster to stay away
from her. Never should one try to bring disgrace to her pride and self-respect, for she knows how to
thaw and saw them. She is today’s woman. It is healthier to love her, respect her and to keep her near.

D. Answer the following in a paragraph of about 80 – 100 words.


1. How are today’s women portrayed by the poet?
Poem I am Every Woman
Poet Rakhi Nariani Shirke
Theme Multi-faceted nature of modern woman

Poem
A woman is beauty innate,
A symbol of poser and strength.

Rakhi Nariani Shirke is an academician with a passion for writing poems as a medium of self
expression. Her poem talks about the multi-faced nature of women.
She portrays that every woman is beautiful innate. She is the symbol of power and strength. She
is willing to put her life at risk. Every woman is true in expressing her love and she is never fake. She
is very dignified in her approach. Even at times of adversity, she finds a ray of hope and she continues
UNIT 3

to care for her near ones. She is very courageous and she has no fear. She is strong in her faith and
beliefs. She is never a quitter and she is optimistic. She is ferocious like a lioness. It is better for the
prankster to stay away from her. Never should one try to bring disgrace to her pride and self-respect,
for she knows how to thaw and saw them. She is today’s woman. It is healthier to love her, respect her
and to keep her near.

She’s today’s woman. Today’s woman dear,


Love her, respect her, keep her near ….. 97
ENGLISH

2. What qualities have made women powerful?

Poem I am Every Woman


Poet Rakhi Nariani Shirke
10 Theme Multi-faceted nature of modern woman

Strong is she in her faith and beliefs


“Persistence is the key to everything,”

Rakhi Nariani Shirke is an academician with a passion for writing poems as a medium of self
I am Every Woman

expression. Her poem talks about the multi-faced nature of women.


The poet Rakhi Nariani Shirke describes the qualities of today’s women. They are empowered,
brave, strong and resolute. They are always ready to take up new ventures. They are persistent and
work tirelessly to prove what they are capable of. Women have to be treated respectfully for the growth
of a nation. Every woman has innate beauty in her. She is a symbol of power and strength. She is
willing to put her life at stake. She is ready to take risk in her life courageously. She is a down-to-earth-
real human being. She is not fake. She is ready to sacrifice her whims and fancy for the happiness of
the loved ones. According to her, others’ happiness is more important than hers. The woman is strongly
and inseparably rooted in her faith and beliefs. Nothing can shake up. The poet goes on to say that she
is a lioness. She should not be taken for granted. She will not spare anyone who plays pranks on her.
She will fight for equality in all spheres.

She knows how to thaw you, saw you – so beware!

ADDITIONAL

Answer the following in a paragraph of about 80 – 100 words.


1. How is today’s woman a warning to prankster?

Poem I am Every Woman


Poem

Poet Rakhi Nariani Shirke


Theme Multi-faceted nature of modern woman

She’s a lioness: don’t mess with her

Every woman is a symbol of power and strength. She is prone to put her life at risk. Every
woman is true in expressing her love and she is never fake. She is very dignified in her approach. Even
at times of adversity, she finds a ray of hope and she continues to care for her near ones. She is not
UNIT 3

fake and she has no fear. She is firm in her faith and beliefs. She is never a quitter and she is optimistic.
She is ferocious like a lioness. It is better for the prankster to stay away from her. Never should one
try to bring disgrace to her pride and self-respect, for she knows how to thaw and saw them. She is
today’s woman. So we should love her, respect her and keep her near.

She is strong in her faith, firm in her belief!

98
ENGLISH

UNIT

3
SUPPLEMENTARY
10
The Story of Mulan

Mulan
EveryofWoman
SUMMARY

Many years ago, China was in the middle of a great war. The Emperor said that one man
from each Chinese family must leave his family to join the army. Mulan, a teenaged girl who lived in a
far away village of China, ran into the house and asked her father whether he knew what the Emperor

I amStory
said. He said that he had heard it in the town itself.
Since her father was not feeling well, Mulan volunteered to join the army in her father’s
place. At once she ran into her room and cut off her long black hair and told her father that she was

The
his son now. She would do her part for China. But the father refused, as women are not allowed in the
army. Mulan replied that since he had trained her in Kung Fu and showed her how to use a sword, she
o know that she
was a woman, they would put her to death.
Mulan kissed her father and said goodbye to him. In the army Mulan proved to be a brave
soldier. In a short time she was put in charge of other soldiers. Her battles went so well that she was
put in charge of more soldiers. After a few years Mulan was given the top job. She became the General
of the entire army.
Sometime later, a very bad fever swept through the army. Many soldiers were sick. And

Supplementary
Mulan, the General of the Army also became sick.
When the doctor came to Mulan’s tent, he knew the truth. The soldiers shouted that the

Poem
General was the woman. Since she tricked them, she must be put to death. But other soldiers raised
their voices saying that they won every battle with Mulan.
Just then a soldier ran up to them saying a surprise attack was coming. Mulan heard this
from inside her tent. She got dressed and went inside. Though she was not strong, she told the soldiers
they must hide themselves so that they could attack when the enemy came inside. It worked as she
had directed them. The battle was won. It was such a victory that the enemy gave up, at last. The war
was over.
UNIT 3

The Emperor was so glad that Mulan had ended the long war. He did not consider the rule
about being a woman. He asked her to be his royal adviser. She said that she must return home and
join her family. The Emperor gave her fine gifts-six fine horses and six fine swords. Everyone cheered
that she was safe.

99
ENGLISH

B.P.No. 90
GLOSSARY

10 carving (v) - an act of cutting a shape or pattern into wood or stone.


robe (n) - a long, loose outer garment reaching the ankles
kung fu (n) - a chinese method of fighting that involves using your hands and feet
and not using weapons
might (n) - great and impressive power or strength, especially of a nation, large
The Story of Mulan

organization, or natural force


bowed (v) - bending the body forwards from the waist, especially to show respect
for someone

A. Choose the best answers. B.P.No. 90

1. Mulan goes to the battle instead of her father because _______.


a) she wants to be a soldier. b) she was asked to join the army.
c) her father is old. d) her brother is sick. Ans: c)

2. What did Mulan do before leaving the house?


a) took leave from her mother b) cut off her hair
Ans: b)

3. What is the story about?


a) winning b) friendship
c) women empowerment d) patriotism Ans: c)

4. The Emperor asked Mulan to stay with him in the palace as his _______.
a) wife b) royal advisor c) army general d) friend Ans: b)

5. The Emperor gave Mulan _______.


Supplementary

a) six horses and six swords b) a death sentence


c) gold d) six camels Ans: a)

6. How did people of the village react to Mulan after her return from the battle?
a) cheered her b) mocked her c) punished her d) scolded her Ans: a)

B. Identify the Character or speaker of the following lines.


1. I heard about it in town Ans: Mulan’s father
2. I am your son now. Ans: Mulan
3. The General is a woman? Ans: Soldier
UNIT 3

4. Mulan, stay with me in the palace. Ans: The Emperor


5. You are too kind sire. Ans: Mulan

C. Answer the following questions in a sentence or two.

1. What was the Emperor’s order?


100 The Emperor’s order was one man from each Chinese family must leave his family to join the army.
ENGLISH

2. Where did Mulan’s father hear about the Emperor’s order?


Mulan’s father heard about the Emperor’s order in the town.

3. Why couldn’t Mulan’s brother go to war?


Mulan’s brother couldn’t go to war because he was a little boy.
10
4. Why did Mulan disguise herself as a man?
Mulan disguised herself as a man because women were not allowed in the army.

5. How did the soldiers become sick?

The Story of Mulan


As a very bad fever swept through the army, the soldiers became sick.

6. How would she be punished if found guilty?


If she were found guilty, she would be put to death.

7. Why did the Emperor give her fine gifts?


The Emperor gave her fine gifts because Mulan had ended the long war successfully.

8. How did the soldiers come to know about Mulan’s real identity?
When she fell sick, the doctor went to her tent to attend to her. At that time he came to know about
Mulan’s real identity. Thus the soldiers came to know about Mulan’s identify.

D. Answer the following questions in a paragraph.

1.
Mulan courageous and patriotic – Emperor’s order – decided to join the army – disguised
herself – strong soldier – powerful leader – fought well – brought victory – ended the war
– chance to become royal advisor – returned to her family

Title The Story of Mulan


Author Robert Daniel San Souci

Supplementary
Theme A Woman can win a war

‘The Story of Mulan’ talks about a girl who was courageous and patriotic. Once she heard
the order of the Emperor to every family to send one member to the army to fight against the enemy.
She decided to volunteer herself to join the army despite being a woman. Her father could not send
his son because he was a little boy so he decided to go to the army. But Mulan put on the dress of her
father and told him that she could disguise herself as a man and join the army. Though she knew that
women were not allowed, she joined the army in disguise, simply ignoring the risks involved. She had
been trained by her father in Kung Fu and use of sword. She knew that if the soldiers came to know
that she was a woman she would be put to death. Ignoring all the risks, she joined the army. She
UNIT 3

proved to be a strong soldier and powerful leader. She led the entire troop to victory and thus ended
the war in China forever. amazed by her bravery, the Emperor asked her to become the royal adviser.
But she reverently refused and sought permission from the emperor to return to her family. This shows
her love towards her family, besides her patriotic spirit.

2. Do you agree with Mulan’s decision to go to war? Justify.


agree – China faced war – Emperor’s announcement – Mulan’s decision – little brother –
101
father not well – disguised – Kung Fu – use of sword – cut the hair – wore father’s dress
ENGLISH

– big risk – put to death – fought well – leader of soldiers – general – ended war – decision
is justified

Title The Story of Mulan


10 Author Robert Daniel San Souci
Theme A Woman can win a war

Yes, I do agree with Mulan’s decision to go to war. Her country China was in the middle
of a great war. The Emperor said that one man from each Chinese family must leave his family to join
the army. Mulan, a teenage girl who lived in a far away village of China told her father that she would
The Story of Mulan

join the army from their family. She had a brother but he was a little boy. She did not want her father
who was not well to go to the army. So in this situation her decision to join the army is agreeable.
Moreover she had been trained in Kung Fu and use of sword. So she cut her hair, put on her father’s
dress and disguised herself as a man. It was a big risk because if she were found to be a woman she
would be put to death. She joined the army and fought for China. She proved to be a strong soldier.
So she was made a leader of soldiers. Finally she became the General. She finally brought an end to
the long war. This clearly shows that she was the right person to join the army. Strong will power, and
determination will make woman succeed. Hence Mulan’s going to war is justified.

ADDITIONAL PARAGRAPH

Answer the following questions in a paragraph.

1. Narrate how Mulan won the fine gifts from the Emperor.
Mulan courageous and patriotic – Emperor’s order – decided to join the army – disguised
herself – strong soldier – powerful leader – fought well – brought victory – ended the war
– chance to become royal advisor – she refused – emperor let her go home with gifts –
returned to her family
Supplementary

Title The Story of Mulan


Author Robert Daniel San Souci
Theme A Woman can win a war

Mulan heard the order of the Emperor to every family to send one member to the army to fight
against the enemy. China was in the middle of a war. She decided to volunteer herself to join the army
despite being a woman. Her father could not send his son because he was a little boy so he decided to
go to the army. But Mulan put on the dress of her father and told him that she could disguise herself
as a man and join the army. She had been trained by her father in Kung Fu and use of sword. She
UNIT 3

knew that if the soldiers came to know that she was a woman she would be put to death. Ignoring
all the risks, she joined the army. She proved to be a strong soldier and powerful leader. She led the
entire troop to victory and thus ended the war in China forever. The Emperor was so glad that Mulan
had ended the long war. He did not consider the rule about being a woman. He asked her to be his
royal adviser. But she said that she must return home. The Emperor gave her fine gifts-six fine horses
102 and six fine swords. Everyone cheered that she was safe.
ENGLISH

ADDITIONAL EXERCISES

I. Rearrange the following sentences in coherent order. 10


I 1. Mulan’s father was sick and her brother was a child.
2. Mulan decided to put on man’s dress and go to the army.
3. The Emperor ordered the families to send one man to the army
4. China was in the middle of war.

The Story of Mulan


5. Mulan’s being a lady could not go. Answer: 4, 3, 1, 5, 2

II 1. After somedays many soldiers fell sick.


2. Mulan was put in charge of many soldiers.
3. Mulan proved to be a brave soldier.
4. Mulan, the General of the Army also became sick.
5. Mulan became the General of the Army. Answer: 3, 1, 5, 1, 4

III 1. The news reached other soldiers.


2. The soldiers did not know their general was a lady.
3. Hey shouted that they would not work under a lady.
4. But when the war came they all fought together.

 Answer: 2, 5, 1, 3, 4
II. Identify the character.
1. Your brother is a child. – Mulan’s father
2. You showed me how to use a sword. – Mulan
3. Then at least take these fine gifts. – Emperor

Supplementary
4. You are too kind, sire. – Mulan
5. The cost is for her to die! – Soldiers

III. Reading Comprehension.

Read the following passage and answer the questions given below.
1.  Many years ago, China was in the middle of a great war. The Emperor said that one man from
each Chinese family must leave his family to join the army. Mulan, a teenage girl who lived in a far
away village of China, heard the news when she was outside, washing clothes. Mulan ran into the
house. Her father was sitting in a chair, carving a piece of wood.
Questions:
UNIT 3

1. What happened to China long ago?


2. What was the order of the Emperor?
3. Who heard the news?
4. What was she doing?
5. What was her father doing? 103
ENGLISH

Answers:
1. China was in the middle of a great war long ago.
2. One man from each Chinese family must leave his family to join the army.
10 3.
4.
Mulan heard the news.
She was washing clothes.
5. Her father was sitting in a chair, carving a piece of wood.

IV. Passage for Comprehension.


1.  Mulan went into her room. With her sword, she cut off her long, black hair. She put on her
of Mulan

father’s robe. Going back to her father, Mulan said, “Look at me. I am your son now. I will go in your
place. I will do my part for China.” “NO, my daughter!” said the old man. “You cannot do this!” “Father,
listen please,” said Mulan. “For years, you trained me in Kung Fu. You showed me how to use a sword.”
The Attic

Mulan swung the sword back and forth with might. “Only so that you could stay safe!” said her father.
“I never meant for you to go to war. If they find out you are a woman, you know as well I do that you
will die!”
The Story

Questions :
1. What did Mulan do in her room? 4. How did she defend herself?
2. What did she say to her father? 5. What was the warning given by her father?
3. How did Mulan’s father react to her decision?
Answers :

2. She said that she would go in his place for China.


3. Mulan’s father was shocked and prevented her from joining.
4. She defended herself by saying that he had already trained her in Kung Fu and she was very
good at that.
5. If they found out that she was a woman, they would put her to death.
2.  In the army, Mulan proved to be a brave soldier. In time, she was put in charge of other soldiers.
Her battles went so well that she was put in charge of more soldiers. Her battles kept on going well.
Supplementary

After a few years Mulan was given the top job-she would be General of the entire army. Not long after
that, a very bad fever swept through the army. Many soldiers were sick. And Mulan, the General of the
Prose

army, became sick, too. When the doctor came out of Mulan’s tent, he knew the truth.
Questions :
1. Who proved to be a brave soldier?
2. What was Mulan in charge of?
3. What was the top job Mulan was given?
4. What happened to Mulan and other soldiers when a fever swept through?
5. Who knew the truth and when?
Answers :
UNIT 3
4

1. Mulan proved to be a brave soldier.


2. Mulan was in charge of more soldiers.
3. Mulan was given the job of the General of the army.
4. Mulan and many soldiers fell ill.
5. The doctor knew the truth when he attended the sick Mulan.
104
ENGLISH

UNIT

4
PROSE
10
The Attic
- Satyajit Ray

B.P.No.93
WARM UP

The Attic
Look at the image given below.

1. Pick out the qualities that you possess and qualities that you expect from your siblings /
friends.
• I possess the qualities such as

Prose
compassion, tolerance, perseverance, respect.

• The qualities I expect from my friends.


sharing, caring, good conduct, commitment, forgiveness.

2. Working in pairs, find out the mutual qualities that you and your friends share. Justify
your top priority with his / hers.
• The mutual qualities are :
UNIT 4

compassion, commitment, forgiveness, respect. Top priority : commitment.

3. Discuss in groups the need for human values.


Human values are the base for harmonious society. Without human values, the fabrics of the
society will collapse. Everywhere there will be tension, anger, quarrel, ill-will and violence. For progress,
peace and prosperity, everyone should imbibe human values.
105
ENGLISH

SUMMARY

10 Ancestral Home at Bramhapur


‘The Attic’ is a short story written by Satyajit Ray. The writer with his friend and business
partner Aditya was driving back from their factory at Deodarganj. As they reached a point of bifurcation,
the author asked his friend about his willingness to take the road,that deviated to the right. That road
led to the ancestral home of Aditya, at Bramhapur. His father was a landlord there and later shifted to
business at Calcutta. He was born and brought up at Bramhapur. After passing matriculation from the
local school, he had left for Calcutta to continue his further studies.
Aditya decided to go to the place. He tried to recollect old memories of his two-hundred-
The Attic

year-old big ancestral house, school, schoolmates, and the tea stall of Nagen uncle. But he also feared,
they might have changed with time and sweet memories of the past might disappoint him.
The sun was on the western horizon, about to set in. As harvest was over, raw paddy fields
were visible on either side of the road. Soon within 10 minutes, they reached the school. Beyond the
iron-gates were the playing field and the two-storeyed school building.
Aditya explained the change- the old building had another floor now and a new building
had come up. The author asked Aditya whether he was good in studies. He said that he always stood
second. They had a cup of tea with two nankhatai biscuits at Nagen Uncle’s tea stall. They met a
n. He was neither
eating, nor drinking tea. Instead he was sitting with head bent over the tea-table, as if daydreaming.
He was hard of hearing, and his sight was not that good as explained by Nagen Uncle, but being a poor
man, couldn’t afford to buy spectacles. Nagen Babu requested him to leave the shop as the afternoon
customers would be coming in soon. The man suddenly stood up and began to recite Rabindranath
Tagore’s “ Panraksha”. Having recited, he left the place.
Nagen Uncle explained that Sasanka Sanyal was living with his friend Jogesh Kabiraj. He
lost his wife and only son last year. He had sold all his lands to get his daughter married. He almost
became a pauper and became somewhat crazy since then, due to mental stress. But he didn’t forget
any memories of the past and possessed a high self-esteem and respect. He never failed to pay his bill
Prose

at the tea stall.


Aditya said that he wished to meet the man in person and drove to the place. Aditya’s
house was surrounded by high walls. The building was in ruins. They reached the attic, on the second
floor of the building. It was in a dilapidated condition and a portion of the wall had crumbled down
creating a window-hole. The floor was not cleaned for long and scattered with straw, twigs and pigeon
droppings. Also there was a broken cricket bat, remains of a former armchair and a wooden packing
case.
UNIT 4

Aditya stood on the packing case and pushed his hand inside the ventilator. A sparrow’s
nest was disturbed, a part of which fell on the ground, and he could find the thing that he was looking
for- that was an antique inside a jeweler’s box. Then he urged the author to go to Jogesh Kabiraj’s
house, where Sasanka Sanyal was living. Sasanka was busy reciting verses from Tagore. He allowed
them to come in. Except for a lone charpoy, there was nothing to sit on. When Aditya asked him
whether he remembered him, he said that he remembered him clearly. He was the spoilt child of rich
106 parents, a good student who used to tell lies and was extremely jealous of him.
ENGLISH

Aditya agreed with him and told him that he wished to give him one hundred fifty rupees. Sasanka
laughed at his sudden generosity and refused to accept money. He wanted the medal, which he took
from him to show and impress his father.
He also explained, the medal cost was not more than 5 rupees. And, he had recognized 10
Aditya by the mole on his right cheek, at the tea stall. So he recited purposely the same poem that he
had recited on the prize-giving day so that he might recall him. He wanted to teach him a lesson for
his act of betrayal. Hence he was furious with Aditya. He didn’t need any charity in cash or kind, but
only needed his lost glory back. Aditya admitted his fault and accepted Sasanka’s rightful grievances
towards him.He handed over the medal to Sasanka, which had been kept inside the ventilator of the
attic, as a hidden treasure for long 29 years. Thus it was restored to its true owner.

B.P.No. 99
GLOSSARY

The Attic
bifurcated (v) – divided into two
revive (v) – to bring something back to life
soothing (v) – making someone feel calm
rustic (adj.) – typical of the countryside
dilated (v) – widened than usual

overwrought (adj.) – state of being upset


attic (n) – the space or room at the top of a building, under the roof
crumbled (v) – broken
spire (n) –  tall, pointed structure on top of a building, especially on top of a
a
church tower
unperturbed (adj.) – undisturbed
affluent (adj.) – wealthy
smacks (v) – drive or put forcefully into or on to something

Prose
ADDITIONAL

site – place essential – important


ancestral – family favourite – beloved
existed – be present strewn – scattered
dedicated – devoted antique – ancient
UNIT 4

savouries – snacks extremely – highly

wink – flash amazed – astonished


reciting – narrating stared – gazed
gesture – sign peering – looking
crazy – mad intently – carefully
107
acute – sharp venting – expressing
ENGLISH

absolutely – completely restored – returned


eventually – finally

10 EXERCISE WITH ANSWER

Choose the appropriate synonym of the word underlined.

1. When I left our ancestral house, it was almost two hundred years old.
a) big b) bungalow c) family d) small Ans : c)

2. He wished to visit the tea shop of Nagen Uncle, if it existed.


a) be present b) be excited c) be old d) be good Ans : a)
The Attic

3. This owner of the tea shop, now over sixty, was a little rustic in appearance.
a) dirty b) trim c) old d) rural Ans : d)

4. With his eyes dilated he began to recite a poem by Tagore – Panraksha.


a) write b) narrate c) show d) hide Ans : b)

5. He left the place, making the gesture of Namaste with his hands.
a) sign b) uttered c) conduct d) life Ans : a)

6. It has become essential to do so,’ Aditya replied.

a) shown b) crushed c) made d) lifted Ans : b)

8. ‘Of course,’ said the gentleman. ‘The spoilt child of affluent parents!’
a) poor b) fashionable c) good d) rich Ans : d)

9. Peering intently at Aditya, Sasanka Sanyal Smiled and said, ‘I had recognised you.’
a) looking b) smiling c) jumping d) running Ans : a)

10. Your grievances are absolutely justified.


Prose

a) wonderfully b) lovingly c) coolly d) completely Ans : d)

EXERCISE for SELF EVALUATION

Choose the appropriate synonym of the word underlined.

1. The medal was eventually restored to its owner.


UNIT 4

a) never b) finally c) beginning d) midst of Ans :

2. When you came to visit me I couldn’t help venting my anger on you.


a) going b) controlling c) calming d) expressing Ans :

3. Peering intently at Aditya, Sasanka Sanyal said, “I had recognised you at Nagen Uncle’s
tea shop.
108
a) carefully b) casually c) happily d) sadly Ans :
ENGLISH

4. Sasanka Samyal was amazed.


a) controlled b) involved c) astonished d) casual Ans :

5. ‘No one visits me’ he said in an unperturbed manner.


a) calm b) disturbed c) quick d) slow Ans : 10
6. The jeweller remarked that it was an antique.
a) new b) fresh c) broken d) ancient Ans :

7. His nerves seemed overwrought for some reason.


a) strong b) stressed c) calm d) overthrown Ans :

8. The floor was strewn with twigs and straw and pigeon droppings.
a) decorated b) withdrawn c) scattered d) broken Ans :

The Attic
9. Having ascertained the location of Jogesh Kabiraj’s house we got into the car.
a) confirmed b) disregard c) confused d) veiled Ans :

10. Sasanka always remembers to pay – having an acute sense of self-respect.


a) dull b) mild c) confused d) sharp Ans :

11. With eyes dilated he began to recite a poem by Tagore – Panraksha.


a) contract b) curtain c) widened d) smiling Ans :

12.
a) joined b) divided c) ended d) started Ans :

13. Trying to revive old childhood memories may prove disappointing.


a) recover b) close c) tell d) cover Ans :

14. The son was soothing.


a) shining b) setting c) comforting d) dull Ans :

15. Nagen Uncle’s tea shop stood next to a grocery shop and opposite a temple dedicated to
Lord Shiva.

Prose
a) disloyal b) different c) indifferent d) devoted Ans :

ANTONYMS

doubt × certainty probably × uncertainly


prove × disprove antique × new
soothing × agitating
affluent × poor
UNIT 4

rustic × sophisticated
extremely × mildly
certainly × doubtfully
absolutely × doubtfully
dilated × contracted
cursed × blessed
favourite × dislike
109
ENGLISH

EXERCISE WITH ANSWER

10 Choose the appropriate antonym of the word underlined.

1. I doubt if even the school building will be recognisable any more.


a) certainty b) confusion c) ambiguity d) uncertainty Ans : a)

2. Trying to revive old childhood memories may prove disappointing.


a) misprove b) disprove c) confirm d) test Ans : b)

3. He leads a cursed life – gone little crazy.


a) wicked b) helping c) cruel d) blessed Ans : d)
The Attic

4. The jeweller remarked that it was an antique.


a) old b) known c) new d) cheap Ans : c)

5. ‘Of course’ said the gentleman, ‘The spoilt child of affluent parents.
a) rich b) well-known c) helping d) poor Ans : d)

1. The sun was soothing.


a) comforting b) agitated c) calming d) restful Ans :

2. The owner of the tea shop was a little rustic in appearance.


a) sophisticated b) old c) new d) rural Ans :

3. ‘Certainly, besides tea, I have biscuits and savouries.’


a) surely b) positively c) definitely d) doubtfully Ans :
Prose

4. With eyes dilated he began to recite a poem by Tagore – Panraksha.


a) widened b) softened c) contracted d) shining Ans :

5. ‘This was my favourite room.’ said Aditya.


a) dislike b) like c) resting d) playing Ans :

6. The attic had probably been the worst hit by wind and weather.
a) certainly b) possibly c) uncertainly d) surely Ans :
UNIT 4

7. He was extremely jealous of me.


a) severely b) positively c) surely d) mildly Ans :

8. Your grievances are absolutely justified.


a) doubtfully b) unconditionally c) fully d) easily Ans :

110
ENGLISH

TEXTUAL QUESTIONS and ANSWERS

a. When did Aditya leave the local school? B.P.No. 94 10


Aditya left the local school after passing matriculation examination to continue his studies in Calcutta.
It was twenty-nine years ago.

b. Why did Aditya think that the school would not be recognisable?
Aditya thought the school might have undergone many changes. So it would not be recognisable.

c. Who were Aditya’s ancestors? B.P.No. 95

Aditya’s acestors were once the zamindars.

The Attic
d. How was the landscape through which they travelled?
As it was the month of Magha-January – February the middle of winter, the sun was soothing. On either
side of the road were paddy fields, as far as the eye could see.

e. What did Aditya visit?


Aditya visited the local school where he had studied up to matriculation.

f. Where was Nagen Uncle’s shop?


Lord Shiva.

g. Besides tea, what did Nagen Uncle have in his shop?


Nagen Uncle had biscuits and savouries.

h. What did Nagen Uncle tell about Sanyal?  B.P.No. 96

Nagen Uncle said that Sanyal was hard of hearing and he could not see well. Being a poor man, he
couldn’t afford to buy spectacles.

i. In what way was Mr. Sanyal’s behaviour strange?

Prose
Mr. Sanyal was sitting in the shop without eating or drinking anything, keeping his head bent. He
stood up and stretched himself. Raising his lean right arm, and with eyes enlarged, he began to recite
a poem by Tagore.

j. What did Nagen Uncle tell about Sanyal’s past life?


Nagen Uncle told that Sasanka Sanyal was living with his friend Jogesh Kabiraj because he lost his wife
and only son last year. He had sold all his lands to get his daughter married. He led a miserable life and
became somewhat crazy since then, due to mental stress.
UNIT 4

k. How did Sanyal show that he had a sense of self-respect?


Though Sanyal was poor, he never failed to pay his bill at the tea stall. Thus he possessed high self-
esteem and respect.

l. Why was the attic ‘a favourite place’ for the children? B.P.No. 97

It was in the attic that the child seems to be in a world of its own.
111
ENGLISH

m. What did Aditya do on reaching the attic?


On reaching the attic, Aditya got on the top of the packing case and pushed his hand inside the
ventilator, upsetting a sparrow’s nest, searching for something.

10 n. What did the jeweller say about the article?


The jeweller said that it was an antique.

o. Was Sanyal happy about his visitors?


Yes, he was happy about his visitors.

p. Why did Sanyal recite the poem in the tea shop earlier?
Sanyal recited the poem in the tea shop earlier because he recognized Aditya by the mole on his right
cheek. So he recited the same poem that he had recited on the prize-giving day so that he might
The Attic

remember him. He wanted to teach him a lesson for his act of betrayal.

q. What was engraved on the medal?


‘Sriman Sasanka Sanyal-Special Prize for Recitation -1948’ was engraved on the medal.

A. Answer the following questions in two or three sentences. B.P.No. 99

1. Write a few lines about the owner of the shop.


The owner of the tea shop was now over sixty. He was a little rustic in appearance with his white
that could be seen
from under a green shawl.

2. What was the daily routine of Sanyal?


The daily routine of Sanyal was that he would visit Nagen Uncle’s tea shop, have tea and biscuits and
would always remember to pay.

3. Why was there a sudden change in Aditya’s expression?


A sudden change in Aditya’s expression was because of his knowledge of the stranger’s plight. He
realised that he was partly responsible for the present situation.
Prose

4. Why did Aditya decide to visit his ancestral home?


Aditya decided to visit the ancestral house because he remembered the medal that had wrongfully kept
with himself for twenty-nine years. It belonged to Sanyal. So he decided to visit to get the medal to
return to him.

5. What was the condition of the attic?


The attic was in a dilapidated condition and a portion of the wall had crumbled down creating a
window-hole.
UNIT 4

6. When did Aditya heave a sigh of relief? Why?


When Aditya got the medal which he had wrongfully kept with him, he heaved a sigh of relief. The
reason was that he had a doubt whether it was still there.

7. Why did Aditya and his friend go to the jeweller?


112 Aditya and his friend went to the jeweller to find out the weight of the article.
ENGLISH

8. What did Aditya offer Sanyal?


Aditya offered Sanyal one hundred and fifty rupees. It was the price of the medal he had been keeping
with him for twenty-nine years.

9. “Your grievances are absolutely justified.” Who says this to whom? Why?
10
Aditya says this to Sasanka Sanyal. The reason is that Sanyal received the medal for his recitation of
Tagore’s verses ‘Panraksha’- Keeping of a Promise. Aditya got it from him to show and impress his
father. But he never returned it to him for twenty-nine years. Instead he said he lost it because of a
hole in his pocket.

ADDITIONAL

The Attic
Answer the following questions in two or three sentences.

1. Why did Aditya wish to visit his birth place?


Aditya wished to visit his birth place to revive old childhood memories though they might prove
disappointing.

2. What change did Aditya notice in his school?


Aditya’s school used to be a one-storeyed building. Now a new building came up, which was not there

3.
Nagen Uncle was wearing a dhoti and a blue striped shirt that could be seen from under green shawl.

4. How did Sanyal respond to Adiya’s offering him money?


Sanyal said that he preferred medal to money. He would have forgotten that unpleasant incident of his
childhood if he had got the medal back.

B. Answer in detail the following questions in about 100-150 words.

Prose
1. Give a detailed account of all thoughts and questions in the narrator’s mind while
accompanying Aditya from the tea shop to Sanyal’s house.

Prose The Attic


Author Satyajit Ray
Theme Mistakes should be atoned and rectified

Give up what appears to be doubtful of what is certain;


UNIT 4

truth brings peace of mind and deception doubt

‘The Attic’ is a short story written by Satyajit Ray. The writer with his friend and business
partner Aditya was driving back from their factory at Deodarganj. As they reached a point of bifurcation,
the author asked his friend about his willingness to take the road,that deviated to the right. That road
led to the ancestral home of Aditya, at Bramhapur. His father was a landlord there and later shifted to
business at Calcutta. He was born and brought up at Bramhapur. After passing matriculation from the 113
ENGLISH

local school, he had left for Calcutta to continue his further studies. The question in the mind of the
narrator was whether Aditya would get back his old memories. When they met Sasanka Sanyal, Aditya
was a little stressed to meet him. When he heard the details about Sasnska Sanyal the narrator saw

10 a change in the expression of Aditya. This brought many questions in his mind like why Aditya was
interested to meet Sasanka Sanyal. When he asked him what it was, he replied that he would get to
know very soon. On reaching the house of Jogesh Kabiraj, he was a little curious, but he didn’t ask him
anything. But at the end he got answers for all his questions.

His nerves seemed overwrought for some reason.

2. ‘Man does change with time’-What were the various changes that came about in Aditya?

Prose The Attic


The Attic

Author Satyajit Ray


Theme Mistakes should be atoned and rectified

Mistakes are fact of life. It is the response to error that counts.

When Aditya went to meet Sanyal, he revealed the incident in the tea shop. In the beginning
Aditya was not interested in going to his ancestral house after twenty-nine years. But when Nagen
Uncle revealed who the stranger was, he remembered the past wrongful incident. Sasanka Sanyal
m him to show and
impress his father. But he never returned it to him. Instead he told him a lie that he lost it. Immediately
he realized his mistake. He wanted to rectify his mistakes. He went to the attic of his ancestral house
and searched for something. He heaved a sigh of relief when he got what he had been looking for. With
that he went to a jeweller’s shop to find out the weight of the article to repay the price of the article
to Sanyal. When Sanyal complained about him, he patiently listened to him and admitted his mistake.
He gave him a packet containing one hundred and fifty rupees. But Sanyal was not willing to get the
money and he wanted the medal back. So Aditya took it from the attic and returned it to Sasanka.

The medal that had been hidden in the attic for twenty-nine years was
Prose

eventually returned to its owner

3. Give a brief character sketch of Sasanka Sanyal.

Prose The Attic


Author Satyajit Ray
Theme Mistakes should be atoned and rectified
UNIT 4

People are interested by talent; God is impressed by character.

Sasanka Sanyal was a talented boy in the school days. Aditya always stood second next to
Sasanka. He received Special Prize for Recitation of Tagore’s Panraksha. He was a generous student.
His friend Aditya asked him to lend the medal to be shown to the father to impress him. He did not
have a second thought. He blindly lent him though he knew that Aditya was jealous of him. When
114
ENGLISH

Aditya did not return the medal he would have felt bad. Another interesting aspect of his character
was when he recognised Aditya by the mole on his right cheek, he recited the same verses of Tagore
to make him realize and repent for the betrayal of trust. Moreover, though he lost his wife and son and
sold the lands, he did pay the bill everyday at the tea shop. He was a man of self-respect. When Aditya 10
came to meet him and gave him one hundred and fifty rupees for the medal, he refused to accept it,
although the price of the medal was only five rupees. This aspect of his character really makes us love
and adore him, because he was not interested in money. Thus right from the tea shop to the end, he
is found to be a lovable character in the story.

He had an acute sense of self-respect.

ADDITIONAL PARAGRAPH

The Attic
Answer in detail the following questions in about 100-150 words.
1. Give a character sketch of Nagen Uncle.

Prose The Attic


Author Satyajit Ray
Mistakes should be atoned and rectified

He was wearing a dhoti and a blue striped shirt.

The owner of the tea shop, now over sixty, a little rustic in appearance with his white
neatlty-combed hair and clean look, was the same as before. It was Nagen Uncle whose revelation
about Sanyal that brings about a change in Aditya. He repents and rectifies his mistakes by going to
Sanyal and returning the medal even after twenty-nine years. He runs a good tea shop and hence lot of
people come there to have tea and savouries. To this shop Aditya comes even after twenty-nine years
simply remembering the location. Even Sanyal comes daily to have tea and biscuits. Besides, he is a
gentle and caring human being. Since Sanyal’s vision and hearing ability are weak, he tells him to go
home at about four o’ clock before other customers come. This may be taken that it would be difficult

Prose
for him to get back home after four. So he tells him to do so. We are greatly touched by his hospitality.

The owner of the tea shop was now over sixty and
little rustic in appearance

B.P.No. 100
VOCABULARY

Compound Words.
UNIT 4

C. Complete the following table with two more compound words.

Noun + Noun kitchen garden, time table, snowball


Noun + Verb mouthwash, rainfall, cat walk
Verb + Noun watchman, call taxi, bath room
Preposition + Noun overcoat, after life, by pass 115
ENGLISH

Gerund + Noun bleaching powder, drinking water, driving school


Noun + Gerund housekeeping, horse riding, bird watching
Adjective + Preposition + Noun good for nothing, free–for–all
10 Noun + Preposition + Noun mother-in-law, father–in–law, lady in love

D. Combine the words in column A with those in column in B to form compound words as
many as you can.
Column A Column B Answer
rain light rainfall
snow thing snowball
star fall starlight
The Attic

draw ball drawback


play back plaything
lottery ticket lottery ticket
under walk underworld
man note manhole
side world sidewalk
foot hole footnote

ks in the
sentences that follow with the appropriate compound words.
waiting out income green sun room tax
alarm dry traffic wall house clock jam
glasses hair cleaning cut put paper
1. Siva visited the hair stylist to have a clean haircut.
2. Tharani had given the sarees for dry cleaning.
3. The green house effect is a natural process that warms the earth’s surface.
Prose

4. Never wait for an alarm clock to wake you up.


5. The children were late to school as there was a traffic jam near the toll plaza.
6. The government expects every individual to promptly pay the income tax.
7. People usually wear sunglasses during summer.
8. The patients were asked to sit in the waiting room until the doctor arrived.
9. With teamwork we are able to multiply our output.
10. The room was looking bright with the colourful wallpaper.
UNIT 4

E. (i) Form new words by adding appropriate prefix/suffix. B.P.No. 101

1. accurate – inaccurate
2. understand – misunderstand
3. practice – malpractice
4. technology – technological
116
5. fashion – fashionable
ENGLISH

6. different – indifferent
7. child – childhood
8. national – nationality
9. origin – original 10
10. enjoy – enjoyment

(ii) Frame sentences of your own using any five newly-formed words.
1. If I don’t invite my friends, they may misunderstand me.
2. Ravi had an unpleasant childhood.
3. Malpractice in the exam will not be entertained under any circumstance.
4. The figure quoted is inaccurate.

The Attic
5. He did not first tell us his original name.

G. Fill in the blanks by adding appropriate prefix /suffix to the words given in brackets.
1. He was sleeping comfortably in his couch. (comfort)
2. Kavya rides a bicycle to school. (cycle)
3. There were only a handful of people in the theatre. (hand)
4. It is illegal to cut sandalwood trees. (legal)
5. The arrival of the President has been expected for the last half an hour. (arrive)

7. Swathy had no intention of visiting the doctor. (intend)


8. The bacteria are so small that you need a telescope to see them. (scope)

B.P.No.102
GRAMMAR

Conjunctions

Prose
• Conjunctions are also known as connectors or linkers or link words.
• We use Conjunctions to join words, a group of words or sentences.
• There are three types of Conjunctions.

Coordinating Conjunctions
• Coordinating Conjunctions link two groups of words that independently make sense.
Examples: and, or, for, otherwise, so, but, yet, still, as well as, etc.
UNIT 4

Subordinating Conjunctions
• These Conjunctions help us to introduce Subordinate Clauses. They are also used to join
Subordinate or dependent Clauses to Main clauses.`
Examples: when, though, although, since, until, till, after, as, before, if, unless, whereas,
while, in case, as long as, as soon as, as much as, therefore, so that, because, as
if, however, etc. 117
ENGLISH

Correlative Conjunctions
• These Conjunctions are always used in pairs. The two Connectors in each sentence that are
related to each other are known as ‘Correlative Conjunctions’.
10 Examples: neither.......nor, either.........or, not only.......but also, scarcely.......when, both........
and, no sooner.........than , such......that, etc.

Conjunctions And Their Functions

Conjunctions Functions
and, not only, but also, as well as, moreover, furthermore, besides, to add information
in addition to
Since, as, for, because, since then, before that, after that to indicate cause/reason
The Attic

but, yet, still, nevertheless, on the other hand, though, although, to express contrast
even though, however, on the contrary
therefore, consequently, then, so, so that, hence, thus to show result or purpose
when, while, after, before, till, until, as soon as, as long as to indicate time
if, unless, whether, in case, provided that to add condition
or, either.........or, neither........nor, otherwise, or else to express choice
Likewise, similarly to denote comparison
ace

EXERCISE

A. Complete the sentences given below choosing the right connectors given in brackets.
1. Call me in case you need money. (so that, in order that, in case)
2. I forgot that I had to meet the Principal. (whether, that, if)
3. Though he is ninety years old, he is in the pink of health. (when, since, though)
4. It is raining. Take an umbrella or else you will get drenched. (or else, and, but)
Prose

5.  hey faced many hardships nevertheless they are always cheerful. (although, nevertheless,
T
otherwise)

B. Fill in the blanks with the connector that goes with the underlined words.
1. Both the minister and the officers visited the affected areas.
2. Jaya teaches not only English but also Science.
3. Either Raghu or Bala will have to buy vegetables from the market.
4. No sooner did I enter the house than it started drizzling.
UNIT 4

C. Combine the pairs of sentences using appropriate connectors.


1. We came late. We did not miss the train.
Though we came late we did not miss the train. / We came late but we did not miss the train

2. They checked the packet twice. Then they sealed it.


118
After they had checked the packet twice, they sealed it.
ENGLISH

3. Sita saw a snake. At once she ran away.


As soon as Sita saw a snake, she ran away. / No sooner did Sita see the snake than she ran away.

4. Robert completed the project. He submitted it to the teacher.


After Robert had completed the project, he submitted it to his teacher.
10
5. Yusuf was running high temperature. He could not take part in the competition.
As / Since Yusuf was running high temperature he could not take part in the competition.

D. Tick the correct linker.

1. _________ he was honest, he was punished. though  but 


2. Walk carefully _________ you will fall down. unless  otherwise 

The Attic
3. My mother called me _________ I was playing football. or  while 
4. My salary is low _________ I find the work interesting. nevertheless  similarly 
5. The passengers rushed to board the bus _________ it arrived. as soon as  as long as 

E. Supply suitable linkers.

1. “Though I was alive and had a human heart, ” answered the statue, “I did not know what tears were,
when I lived in the palace and sorrow was not allowed to enter. My courtiers called me the Happy
Prince because Happy Indeed I was. So I lived and soon I died.

ey may make 2.
grammatical errors and leave out important punctuation marks. Making such mistakes is quite common
at the time of preparing the first draft . So he must carefully edit his final draft.

3. In most large cities and towns of our country, there are special schools for girls. But, there are many
co-educational schools where girls and boys study together. Most parents allow their daughters to
attend these schools, though there are some parents who are against such schools for girls in the age
of 14 or15.

F. Rearrange the words in the correct order to make meaningful sentences.

Prose
1. as / I / healthy / are / you / am / as
I am as healthy as you are.

2. your / today / put on / new / since / is / birthday /dress / the


Since today is your birthday, put on the new dress.

3. allergic / dogs / Rani / though / is / to / of / six / she / them / has


UNIT 4

Though Rani is allergic to dogs, she has six of them.

4. speaks / Ruben / besides / German /languages / two


Ruben speaks two languages besides German.

5. loan / apply / you / if / for / you / a / get / will / immediately / it /


If you apply for a loan, you will get it immediately.
119
ENGLISH

Nominalisation
• The term “nominalisation” refers to the process of producing a noun from another part of speech
by adding a derivational affix.
10 • A grammatical expression is turned into a noun phrase when we nominalise a sentence. For
example,

A) After 1885, trade with Europe grew. (Verb)

B) After 1885, there was a growth in trade with Europe. (Noun)


• In sentence B, we have used the word ‘growth’ which is the noun form of the verb ‘grow’ by
adding the suffix ‘th’.

Examples
The Attic

admire – admiration arrive – arrival


careless – carelessness fail – failure
include – inclusion intense – intensity
punish – punishment

G. Write the noun forms of the following words.


1. beautiful – beauty 6. zealous – zeal

3. enter – entry 8. accept – acceptance


4. know – knowledge 9. dangerous – danger
5. deafen – deaf

H. Complete the following sentences using the noun form of the words given in brackets.
1. The boy had to give a proper explanation for being late. (explain)
2. They could make prediction about the future. (predict)
3. At one point in life, he had no choice but to trust his friend. (choose)
Prose

4. The monuments are to be preserved because of their historical significance. (significant)


5. It is very difficult to work with so many distractions. (distract)

I. Rewrite the sentences nominalising the underlined words. The first one has been done
for you.
1. We succeeded in our attempt.
UNIT 4

We got success in our attempt.

2. Nalini leads a happy life.


Nalini leads a life of happiness.

3. She failed and it disappointed her.

120 She met with failure and it gave her disappointment.


ENGLISH

4. India became an independent country in the year 1947.


India got independence in the year 1947.

5. The child resembles her father.


10
The child has resemblance to her father.

J. Combine the pairs of sentences given below into a single sentence using the noun
form of the highlighted words.
1. He is an honest person. Everyone likes him.
Because of his honesty, everyone likes him.

2. Sathya gave an explanation. The police wanted her to prove it.

The Attic
Though Sathya gave an explanation, the police wanted a proof for it.

3. He speaks well. It attracts all.


His good speech is an attraction for all.

4. Suresh is always punctual and regular. It has earned him a good job.
Suresh’s punctuality and regularity has earned him a good job.

The policeman’s quick arrival made us happy.

K. Complete the sentences in the paragraph using the appropriate form of words given
in brackets.
1. My sister wanted to go to Mumbai last week. She made a decision (decide) to buy a ticket at once.
As reservation (reserve) could be done online, she gave preference (prefer) to book a ticket that way.
First, she collected information (inform) about the arrival (arrive) and departure (depart) of trains and
airplanes.

Prose
2. A few days later, Androcles was captured by his master. He had to suffer all kinds of punishment
(punish). At last, he was thrown to a lion which was in great hunger (hungry). It had been kept in an
enclosure (enclose) and had not been fed for several days. His friends stood there with teary (tear)
eyes as the lion rushed towards him. The lion stopped near him and stood for a while looking (look)
at him. Then it lay down by his side like a pet dog. Obviously (obvious), the lion recognized Androcles
and the help (help) he had given it.

Phrases and Clauses.


UNIT 4

Finite And Non-Finite Verbs:


Words which denote an action are known as verbs. We classify verbs into two types. They are:

1. Finite verbs:

a. My brother goes to temple daily.

b. We have already finished the project. 121


ENGLISH

The words printed in bold letters are finite verbs.


1. Finite verbs indicate the tense and time of actions.
2. Finite verbs undergo a change as and when the Subject (number or person) changes.
10 2. Non-Finite Verbs:
1. Non-finite verbs do not indicate the tense and time of actions.
2. Non-finite verbs do not change even when the Subject (number or person) changes.
There are three kinds of non-finite verbs.
1. An infinitive (to + verb)
2. A gerund (verb + ing)
3. A participle
The Attic

Example:
a. My son likes to watch cricket matches. (Infinitive)
b. Playing chess is my hobby. (Gerund)
c. Driven out of the kingdom, the king hid himself in a forest. (Participle)

PHRASE:
• A Phrase is a group of words without a finite verb.

an intelligent boy, a costly pen, an interesting story


The above group of words are known as phrases. It doesn’t contain a finite verb.

CLAUSE:
• A Clause is a group of words which consists of a finite verb.

Example :
a boy who is intelligent, a pen which is costly, a story which is interesting
Prose

The groups of words given above are clauses

More Examples:
• Having completed the work, the boy went out to play.
 he underlined part of the sentence, doesn’t contain a finite verb. This group of words is a
T
Phrase.

• After the boy had completed the work, he went out to play.
The underlined part of the sentence contains a finite verb. Hence, we call it a clause.
UNIT 4

Kinds of Phrases :
We have three kinds of phrases according to their functions in sentences.

1. Adjective Phrase:
It is a group of words that does the work of an adjective. It describes the noun.
122
Example : We bought chairs made of wood for our auditorium.
ENGLISH

2. Adverb Phrase:
It is a phrase which functions as an adverb. This Phrase supplies some information about the action.
Example : When the patient was taken to the emergency ward, the doctors rushed there in a
hurried manner. 10
3 Noun Phrase:
This is a phrase which acts as a noun.
Example : A boy of class X became the house captain.

L. Identify the phrases in the following sentences and classify them as Adjective, Adverb
or Noun phrase.
1. The girl in blue saree is my sister. (Adjective phrase)

The Attic
2. Kohli hopes to win the trophy. (Noun phrase)
3. The train halts at every junction. (Adverb)
4. I have never seen such a picture. (Noun)
5. She worked in an enthusiastic manner. (Adverb)

Kinds Of Clauses
1. Adverb Clause :
It modifies the verb, that is, it tells something about the action. This Clause gives details about the

arrived.
(The highlighted part of the sentence speaks about the time of the action)

2. Noun Clause :
This clause functions as a noun.
Example: Whoever wins the contest will get a prize.
(The highlighted portion acts as a noun here)

3. Adjective Clause : 

Prose
It acts as an adjective and describes a noun.
Example: I went to the place where I was born.
(The highlighted words describes the place)

M. Identify the clauses and classify them accordingly.


1. Ramu bought a pen that doesn’t write well. (Adjective Clause)
2. Come back as soon as possible. (Adverb Clause)
UNIT 4

3. Most of her friends whom she had invited attended her wedding. (Adjective Clause)
4. My brother visits my father whenever he comes to Chennai. (Adverb Clause)
5. Call me in case there is an emergency. (Adverb Clause)
6. Until the sun sets, the old woman cannot step out of her house. (Adverb Clause)
7. She knows where I go. (Noun Clause)
123
8. You can go wherever you want. (Adverb Clause)
ENGLISH

B.P.No. 109
LISTENING

10 N. Fill in the blanks:


1. Login into your IRCTC account.
2. Fill in the information asked to you in Booking section.
3. The starting point and destination of your journey must also be selected.
4. List of available trains will appear.
5. You must check on the availability and fare for the train of your choice.
6.  our personal details like name, date of birth, berth preference, mobile number, any valid ID
Y
The Attic

proof and email are a must.


7. After filling information and captcha click on continue booking.
8. You can make the payment either by credit / debit cards or e-wallets.

B.P.No. 110
SPEAKING

O. Given below are the various personalities from different fields. The topic of discussions is

e press. Take
turns to be the director and media persons. Conduct a perfect discussion.

Reporter A : Congrats Sir! To whom would you like to dedicate the award?
Tony : To my loving dad. But for his blessing, I would not have completed the
movie.
Reporter B : Who is the brain – child of your project?
Tony : It’s my sister who has given the one line story. I was simply stunned the
Prose

instant I heard it.


Reporter C : Usually award – winning movies wouldn’t be box – office hits. But yours is
a different story. Your comments, please.
Tony : When I was directing the movie, I was pretty sure it would ring the cash
boxes.
Reporter D : How did you feel when you received the award?
Tony : I felt as if I were in seventh heaven. But in a jiffy, I felt humbled.
UNIT 4

Reporter E : What’s your next project?


Tony : Certainly not Poo- Part II. It is going to be a Sci-fi like ‘Gravity’
Reporters : Wish you all the best, Sir.

ii. Ms. Pavithra Rao, the squash player who won the gold medal at the recent Asian Games ,
is holding a press conference . Let the discussion focus more on the strategies that helped
124 her to win.
ENGLISH

Reporter A : Congrats madam!


Ms Rao : Thank you. I never thought I would be in the limelight.
Reporter B : A sports person of your caliber will be certain to receive acclaim and
accolade.
10
Reporter C : Would you please tell us the strategies you adopted to win a gold in the
Asiad?
Ms Rao : It’s sheer single – track mind. I turned my weakness into strength and
turned the opponent’s strength into weakness.
Reporter D : How did you get back when you had a set back?
Ms Rao : My focus was not on the previous set or point. ‘Now or Never’ is always
my mantra. During breaks, a minute mediation gives me immense

The Attic
concentration.
Reporter E : Did you feel jittery in the last few minutes of the game?
Ms Rao : Certainly not. I remain calm, collected and composed against all oddities
and adversities. Just like my hero, Mahendra Singh Dhoni I remain unfazed
and unagitated. That’s the secret of my success.
Reporters : Congrats once again. Wish you a stunning success in the Olympic Games.

iii. GL Home Appliances have introduced a product to purify salt water. The CEO of the

CEO of GL : I am happy to meet you all. Thank you so much for respecting my invitation
to be here.
Reporters : We are also happy to be here
CEO of GL : I have called you all here to introduce our new product to purify salt water.
You know well that nowadays clean water is a big problem.
Reporter A : We have many other things to do the same work. Do you need one more?
CEO of GL : People get mostly the bore water which is very salty. As you said there are
many other products but ours is something different.

Prose
Reporter B : The complaint about these products is that water is wasted.
CEO of GL : Yes, this is the problem of many people make.
Reporter C : Do you have any extra facility to reduce the wastage of water?
CEO of GL : Yes, we have added some chips which would reduce the water wastage.
Moreover the waste water can be used for washing purpose and it has a
facility to send the water to the garden pipes.
Reporter D : I think it sounds well. If your product helps the people surely people will
UNIT 4

go for your product.


CEO of GL : Thank you. I thank you all for your presence.
Reporters : Thank you sir.

125
ENGLISH

B.P.No. 111
READING

10 Read the following letter from a parent to her son’s coach and answer the questions given
below:
Dear coach,
Thanks for the special gifts that you have given to my child. You learned his name and spoke it
often. You taught him the basics of the sport as well as special ways to improve and excel. Although
you had a whole team of kids to mentor, you took time for individual instruction where needed.
Under your care, I have watched him transform from a timid, doubting child to a strong, happy
The Attic

player willing to give all for the team. Throughout the season when he gave his best, even though it
was not quite enough to gain that extra point, you recognised his contribution with a pat on the back
and encouraging words.
Your wise approach showed him that, although winning is a goal, there are other goals just as
worthy. He learned the value of finishing what he started and joy of personal accomplishment. These
attributes carried him through a season that was full of hard work and fun, discouragement and
resolve, defeat and victory.
And at the very end, at the championship meet when he brought home his first place medal, you
to all of us. What
amazes me is you’ve taught them skills that will last a lifetime. You’ve kindled in them a desire to excel.
The medals, trophies and ribbons are all symbols of real gifts. These most certainly have had to come
straight from your heart.
With appreciation,
A parent.

P. Answer the following questions.


Prose

1. What did the coach teach the child?


The coach taught the child the basics of sport.

2. What values did the child learn?


The child learnt the value of finishing what he had started and joy of personal accomplishment.

3. The parents noticed some changes in the child. What were they?
Some changes the parents noticed in the child were that he went through a season of hard work and
fun, discouragement and resolve, defeat and victory.
UNIT 4

4. Read the letter again and write a few lines on each of the following:

a) things that the coach taught...


 he coach taught him the basics of the sport as well as special ways to improve and excel.
T
Although there was a team of kids to mentor, he found time for individual instruction where
126 needed.
ENGLISH

b) transformation in the child....


 nder the coach’s care the child transformed from a timid, doubting child to a strong, happy player
U
willing to give all for the team. When the child did not gain extra point, the coach recognized his
contribution with a pat on the back and encouraging words. 10
c) things that amazed the writer...
 he coach has taught the players the skills that will last a lifetime. He has kindled in them a
T
desire to excel. The medals, trophies and ribbons are all symbols of real gifts.
5. Find sentences/words from the text which express the following:
a) timid and doubting child...
b) winning is a goal, there are other goals just as worthy...

The Attic
c) the medals, trophies and ribbons...

B.P.No. 112
WRITING
Q. Prepare notice for the following.

i. You are the school monitor, of Modern Matriculation School, Villupuram. Your school
Principal has requested you to inform the students about a trip to Yercaud for 3 days.
ortation,
amount, dress code etc.

NOTICE
MODERN MATRICULATION SCHOOL, VILLUPURAM.
Trip to Yercaud
10 September 2020
All the students are informed that the school is organising a three-day trip to

Prose
Yercaud from 3rd – 5th of October 2020. The mode of transportation is bus. The amount for the
trip is Rs. 2000 which includes stay, breakfast, lunch, supper and tea and snacks for three days.
Students should bring three sets of uniform, bed sheet, towel and toiletries and required items
for their personal use. Students are kindly asked to give the following contact numbers to their
parents for contact in case of any emergency: 97517XXXXX/85523XXXXX. Interested students are
asked to enrol their names with their class teachers.

Ramesh/Ranjani
UNIT 4

School Monitor.

ii. You are the Secretary of Park Circus Residents Welfare Association. Write a notice to
inform the residents of your colony of a Meditation program under the guidance of
Dr. P. Ranjit with a view to understanding the self better. The program is exclusively for
the residents. It will be conducted on the second Saturday of the following month from
7.00 a.m. to 9.00 a.m. at the children’s park nearby . 127
ENGLISH

NOTICE
PARK CIRCUS RESIDENTS WELFARE ASSOCIATION

10 Meditation Programme
15 July 2020
This is to inform the residents of Netaji Colony about the Meditation programme
to be conducted under the guidance of Dr. P. Ranjit with a view to understanding the self better.
The programme is exclusively for the residents of Netaji Colony only. It will be conducted on the
second Saturday the 14th of August 2020 from 7.00 a.m. to 9.00 a.m. at the Children’s Park near
the Electricity Board. Interested persons can register their names with the undersigned.
Note: No Entry Fee. Yoga mat will be provided free of cost for every participant
The Attic

Mano
Secretary
Park Circus Resident’s Welfare Association

iii. You are Ganesh/Gayathri Head boy/Head girl, of your school. Write a notice for your school
notice board informing the students about the ‘Fancy Fete’ that is going to be organised in

NOTICE
CHAMPIONS MATRICULATION SCHOOL, SIRKAZHI.
Fancy Fete
11 November 2020
This is to inform the students that Fancy Fete will be organised for Classes LKG to
XII on the 10th of December 2020 on the school premises between 9.00 a.m. and 1.00 p.m. The
rubrics for the competition are: theme, make-up, costume, dialogues for one minute. Students
may select national leaders, great personalities and social issues. Interested students may enrol
Prose

their names with their respective class teachers on or before 25 November 2020.
Sushant/Sushmitha
Head boy/Head girl

R. Write an article for the following.

i You are Jansi/Avinash of Class X studying in GHSS, Chengalpet. You believe that physical
activities improve our health and reduce the risk of sickness. It has got immediate and long
UNIT 4

term benefits. Write an article in not more than 150-200 words for your school magazine
activities in a student’s day to day life.

Importance of Physical Activity in School


- By Jansi/ Avinash

128
ENGLISH

Engaging in acts of physical activity from a young age is very important for both mental and
physical health of a child. The habits learnt in youth tend to continue onto adulthood and reduce the
effect of obesity and related illnesses.

The benefits of physical activity are vast. Physical activity is connected with a lowered risk of 10
cardiovascular disease and also an increased life expectancy. So its importance is obvious but its value
in children sometimes needs to be justified.

“Young people will carry established healthy behaviours throughout their lives such as better
eating habits and decreased likelihood of smoking” say Shilton, T. and Naughton. Through physical
activity children of middle childhood develop healthy muscles, bones and joints. They develop healthy
heart and lungs. They also develop a higher neuromuscular awareness which influences their co-
ordination and movement and the development of motor skills. Most obviously it enables them to

The Attic
maintain a healthy body weight. Physical activity has also been linked to psychological benefits by
improving the control children have over the symptoms of anxiety and depression. Studies have also
shown that the participation in physical activity can assist in the social development of children aged
6-10 by providing the opportunity for expression, building confidence and social interaction. Physically
active young people demonstrate higher academic performance at school.

ii. The service provided by the conservancy workers in your city is very poor. You find all the
street corners dumped with garbage thrown by the residents of the locality. It causes a
menace for the public at large. You are Ramya/Rajan of Class X, studying in TM Model
of The Indian
Express, about this and suggest ways by which the situation could be improved.

Garbage Menace –a Serious Health Hazard


- By Ramya/Ranjan

Garbage that is not properly managed is a serious health hazard and leads to the spread of
infectious diseases. Unattended garbage lying around attracts flies, rats, and other creatures that in
turn spread diseases. Normally it is the wet waste that decomposes and releases a bad odour. This
leads to unhygienic conditions and thereby to a rise in health hazards.

Prose
There are many forms of waste such as solid, gas or liquid and each has different process of
disposal and management. Waste management manages different types of waste created by industries,
household, commercial activities or natural waste. Large segment of waste management relates to
municipal solid waste i.e. the waste created by industries, housing and commercial establishments.

Skin and blood infections resulting from direct contact with waste, and from infected wounds,
are a serious concern to be addressed.
UNIT 4

The general concepts of waste management are: Reduce, Reuse and Recycle. Inefficient waste
management has several negative effects on health of living beings, environment and economy for e.g.
air pollution, soil contamination, spread of hazardous diseases, etc.

Proper methods of waste disposal have to be undertaken to ensure that it does not affect the
environment around the area or cause health hazards to the people living there.

129
ENGLISH

iii. Recently while returning home from school you were knocked down by a speeding
motorcycle. You escaped with minor injuries. You are Kishore/Kavitha of class XI, studying
in GHSS, Coimbatore. Write an article to The Hindu, in about 150-200 words expressing

10 your concern about the increasing number of road accidents due to reckless driving. Also
stress the importance of following traffic rules.

Hazards Of Reckless Driving


- By Kishore/ Kavitha

Reckless driving is a serious problem in India. Many people become victims everyday due to this
menace. According to Road Accident Statistics in India, 1214 road crashes occur every day in India.
One serious road accident in the country occurs every minute and 16 people die on Indian roads every
The Attic

hour.

In most of the accidents, it has been found that the people don’t have road safety sense. Nor
do they follow safety rules and regulations. In a majority of cases and incidents people don’t wear
helmets. They fall prey to over-speeding. It is the duty of the Government to provide safe environment
to the people who use road to reach their destinations.

Keeping in mind the shocking statistics, it becomes imperative to introduce learning road safety
measures at schools. Schools must make road safety measures an integral part of curriculum. The

schools
road safety weeks and days should be observed. Road safety seminars, workshops, and other similar
programmes should be conducted. They must be taught that driving on roads is not fun or a child’s
play. One must be totally prepared and trained to go on roads.
The traffic police must organize vehicle training sessions for the public. People must be given
demonstrations on safe driving skills. The traffic police must also be more vigilant. Strict action must
be taken against those people who break the law.
Prose
UNIT 4

130
ENGLISH

UNIT

4
POEM
10
The Ant and the Cricket
-Adapted from Aesop’s fables

the Cricket
SUMMARY

andAttic
The poem tells us the story of a careless cricket who was busy in singing only and did nothing
for his future. However, an ant who was very sincere in his work did everything for the future. The ant
lived with self-respect even in harsh time while the cricket had to beg and could not survive during

The AntThe
winter.

The poem “The Ant and the Cricket” is a fable –that is a story consisting of animals as characters
that convey a moral – written in a poetic format. It is about a silly young cricket and a prudent ant.
The cricket used to sing all day long and enjoy his good times during summer season. He lacked
farsightedness, for he never had plans for his future. When winter arrived, the cricket could not find
from the ant, he
would die of starvation and sorrow. So, the cricket went to the ant to borrow food and get shelter.

The ant gave an important life lesson during its conversation with the cricket — ants neither
borrow from somebody nor lend to somebody. Instead they work hard and save for the future. When
the ant questioned the cricket what it was doing during summer, the cricket confessed his indulgence
in merrymaking.

The poem ends with the ant asking the cricket to try dancing and singing again during its hard
times-in winter too. The ant chased him away from his place. This fable drives home the message even
for the human beings that we should indeed enjoy our happy moments but must not ignore our future.

Prose
Poem
Carefully planning our future is essential and significant for everyone, in order to ensure security.

B.P.No. 117
GLOSSARY

accustomed to (v) – be used to


gay (adj.) – glad, joyful
crumb (n) – piece of bread
UNIT 4

famine (n) – extreme scarcity of food


miserly (adj.) – hesitant to spend money
quoth (v) – said (old English usage, used only in first and third person singular
before the subject)
hastily (adv.) – hurriedly
warrant (v) – guarantee, promise 131
ENGLISH

ADDITIONAL

10 a crumb - a piece starvation - hunger


shelter - home / habitat wicket - gate/ stump

A. Based on your understanding of the poem, read the following lines and answer the
questions given below.
The Ant and the Cricket

1. A silly young cricket, accustomed to sing


Through the warm, sunny months of gay summer and spring.
a) What was the routine of the cricket?
The routine of the cricket was that he got accustomed to singing in summer and spring.

b) Name the seasons mentioned here.


Summer and Winter are the seasons mentioned here.
2. Began to complain when he found that,
at home,
His cupboard was empty, and winter
was come.
a) Who does ‘he’ refer to?
‘He’ refers to the cricket.

His cupboard was empty because he did not collect and preserve any grains for the winter.
3. Not a crumb to be found
On the snow-covered ground;
a) What couldn’t he find on the ground?
He couldn’t find a single crumb –piece of bread on the ground.

b) Why was the ground covered with snow?


The ground was covered with snow because it was winter season.
Poem

4. At last by starvation and famine made bold,


All dripping with wet, and all trembling with cold,
a) What made the cricket bold?
Starvation and famine made the cricket bold.

b) Why did the cricket drip and tremble?


It was winter and he had not made his shelter. So he dripped with wet and trembled with cold.
UNIT 4

5. Away he set off to a miserly ant,


To see if, to keep him alive, he would
grant
Him shelter from rain,
And a mouthful of grain.
a) Whom did the cricket want to meet? Why?
132
The cricket wanted to meet the ant, because he wanted to get shelter and borrow some grain.
ENGLISH

b) What would keep him alive?


Shelter from rain and a mouthful of grain would keep him alive.
6. But we ants never borrow;
we ants never lend. 10
a) Why do you think ants neither borrow nor lend?
Since ants have the habit of saving something for the future, they are boldly saying they neither
borrow nor lend, for borrowing or lending makes one a slave for the other.

The Ant and the Cricket


b) Who says these lines to whom?
The ant says these lines to the cricket.
7. “Not I!
My heart was so light
That I sang day and night,
For all nature looked gay.”
a) Who does ‘I’ refer to?
‘I’ refers to the cricket.

b) What was the nature of the cricket? How do you know?


The nature of the cricket was that it indulged in merrymaking in summer. This we can know from
the way it spent the summer.
8.
And out of the door turned the poor little cricket.
a) The ant refused to help the cricket. Why?
 he ant refused to help the cricket because it spent the whole summer singing all day long and
T
enjoying his good times during summer season. He lacked farsightedness, for he never had
plans for his future.

b) Explain the second line.


The ant chased the little careless and lazy cricket out of the door to fend for himself.

Poem
9. He wished only to borrow;
He’d repay it tomorrow;
a) Pick out the rhyming words in the above lines.
borrow – tomorrow

b) Give more examples of rhyming words.


s ing, spring; home, come; found, ground; see, tree; bold, cold; ant, grant; rain, grain; light,
night; gay, say; wicket, cricket; true, two are other rhyming words in the poem.
UNIT 4

10. My heart was so light


that I sang day and night,
For all nature looked gay.”
“You sang, Sir, you say?
a) Mention the rhyme scheme employed in the above lines.
aabb. 133
ENGLISH

ADDITIONAL

10 1. Not a flower could he see,


Not a leaf on a tree,
oh! what will become, “says cricket,” of me?
a) Who does ‘he’ refer to?
The Ant and the Cricket

‘He’ refers to the cricket.

b) Why is he sad?
There was no food and shelter for him. So he is sad.
2. ”You sang, Sir, you say?”
Go then, “says the ant,” and dance the winter away,
a) To whom does the ant say this?
The ant says this to the cricket.

b) What does the phrase ‘dance the winter away’ mean?


‘Dance the winter away’ means spending the winter season in a joyful manner.
3. Says the ant to the
cricket, “I’m your servant

But we ants never


borrow; we ant’s never
land.
a) What rule do the ants follow?
Ants follow the rule that never lend or borrow.

b) Pick out the rhyming words.


friend, lend.
Poem

4. Folks call this a fable. I‘ll warrant it true:


Some crickets have four legs, and some have two
a) What is the fable about?
The fable is about the ant and the cricket.

b) What is the moral of this fable?


Prevention is better than cure.
UNIT 4

B. Based on your understanding of the poem, complete the summary using the phrases
given below.

the pleasant nature human beings doesn’t save warm place


kitchen cupboard just a fable saving for future some grains
never borrow or lend an ant and a cricket sings and dances
134
ENGLISH

In this narrative poem, the poet brings out the idea that is essential for every creature. He
conveys this message to the readers through a story of an ant and a cricket. The ant spends all
its summer saving for future. The cricket sings and dances happily in the summer. He doesn’t save
anything for the winter. When winter comes, he is worried that his kitchen cupboard is empty. So, he 10
seeks the help of the ant to have some grains and a warm place to stay. The cricket was even prepared
to repay it in the future. The ant made it clear that ants never borrow or lend. He also enquired cricket
if it had saved anything when the weather was fine. The cricket answered that it had sung day and

The Ant and the Cricket


night enjoying the pleasant nature. The ant threw the cricket out and stated in a stern voice it should
dance in the winter season too. In his concluding lines, the poet affirms that this is not just a fable but
it is true and applicable to human beings also.

C. Answer each of the following questions in a paragraph about 100 words.


1. ‘Some crickets have four legs and some have two.’ Elucidate this statement from the poet’s
point of view.
Poem The Ant and the Cricket
Poet Aesop
Theme Forewarned is forearmed

Not a crumb to be found

‘Aesop’s fables’ is a collection of fables credited to Aesop, a slave and a story teller believed to
have lived in Greece between 620 and 564 B.C.E. These fables became popular when they emerged in
print.
Through the poem ‘The Ant and the Cricket’, the poet brings out the idea that it is essential for
every creature to work hard for the future. In the poem, the ant spent all its summer saving for future,
whereas the cricket sang and danced happily in the summer. He did not save anything for the winter.
When winter came, he was worried that his kitchen cupboard was empty. So, he sought the help of the
ant to have some grains and a warm place to stay. The ant made it clear that ants would never borrow
or lend. He also enquired the cricket if it had saved anything when the weather was fine. The cricket

Poem
answered that it had sung day and night enjoying the pleasant nature. Some crickets work as if they
have only two legs. In his concluding lines, the poet affirms that this is not just a fable but it is true
and applicable to human beings also.

And out of the door turned the poor little cricket


Folks call this a fable.

2. Compare and contrast the attitude of the ant and the cricket.
UNIT 4

Poem The Ant and the Cricket


Poet Aesop
Theme Forewarned is forearmed

A silly young cricket, accustomed to sing


Through the warm, sunny months of gay summer and spring 135
ENGLISH

‘Aesop’s fables’ is a collection of fables credited to Aesop, a slave and a story teller believed to
have lived in Greece between 620 and 564 B.C.E. These fables became popular when they emerged in
print.

10 Through the story of the lazy cricket and the hardworking ant, the poet teaches us the virtue of
hard work. The cricket was foolish enough not to see the future. He made no plans or provisions for
the future. He lived in the present moment. The ant, on the other hand, was wise and hardworking.
He worked hard so that he could enjoy the future. The poet wants us to learn an important lesson in
The Ant and the Cricket

life. One must always save for the future. One should not be foolish enough to just enjoy the present
moment. ‘As you sow, so shall you reap.’ is a popular proverb that fits this story. We must be fare
sighted enough to see our future and make plans accordingly. One should be ready for the good as
well as the bad times ahead. This attitude of the ant is not only an eye-opener for the cricket but also
for the human beings. We should not be like the cricket and ruin our future.

But we ants never


Borrow; we ants never lend

3. If given a chance, who would you want to be - the ant or the cricket? Justify your answer.

Poem The Ant and the Cricket


Poet Aesop
Theme Forewarned is forearmed

Began to complain when he found that , at home,


His cupboard was empty, and winter was come.

I would obviously want to be the ant because the cricket is lazy whereas the ant is hard-working.
The cricket was foolish enough not to see the future. He made no plans or provisions for the future.
He lived in the present moment. He indulged in merrymaking and daydreaming and gave in to desires
and passions. The ant, on the other hand, was wise and hard-working. He worked hard so that he
could enjoy the future. The poet wants us to learn an important lesson in life. One must always save
something for the future. One should not be foolish enough to just enjoy the present moment. ‘As you
Poem

sow, so shall you reap.’ is a popular proverb that fits this story. We must be fare sighted enough to see
our future and make plans accordingly. One should be ready for the good as well as the bad times
ahead. This act of the ant is not only an eye-opener for the cricket but also for the human beings. We
should not be like the cricket and ruin our future. So I would prefer to be the prudent miserly ant.

But tell me, dear cricket,


Did you lay anything by
When the weather was warm?
UNIT 4

ADDITIONAL PARAGRAPHS

Answer each of the following questions in a paragraph of 120-150 words.

136
ENGLISH

1. What led to the doom of the Cricket in the poem?

Poem The Ant and the Cricket


Poet Aesop
Theme Forewarned is forearmed
10
“Oh! What will become,” says cricket, “of me?”
At last by starvation and famine made bold

The Ant and the Cricket


The cricket himself is the cause for his own doom. Since he loved to sing, he sang all summer
and spring with no worries in the world. He began to complain as the winter arrived. He found that his
cupboard was empty and he had no food to eat. Neither a leaf, nor a flower could he find. Everything
was covered under the snow. He cried as he imagined his bad situation. Finally, out of hunger and
starvation, it being all wet and cold, he set off to the house of a miserly ant. He wanted to know if the
ant would lend him a mouthful of grain and a shelter from the rain. He promised that he would repay
the ant. He knew that if he didn’t do this, he would die of hunger and pain. The ant said to the cricket
that he was his servant and friend. But he made one point clear in friendship that there should be no
borrowing or lending. Having said that, the ant chased him out of his house. Thus he ended up in a
miserable condition.
He wished only to borrow;

2. How does the poet apply the fable to human beings?

Poem The Ant and the Cricket


Poet Aesop
Theme Forewarned is forearmed

Away he set off to a miserly ant,


To see if, to keep him alive, he would grant

Poem
The poet through the poem admonishes us that we should enjoy happy moments but should not
ignore our future. Careful planning is essential for everyone and it ensures secure future. The ant
spends all its summer saving for future. The cricket sings and dances happily in the summer. He
doesn’t save anything for the winter. When winter comes, he is worried that his kitchen cupboard is
empty. So, he seeks the help of the ant to have some grains and a warm place to stay. The cricket is
even prepared to repay it in the future. The ant makes it clear that ants never borrow or lend. He also
enquires the cricket if it saved anything when the weather was fine. The cricket answers that it has
sung day and night enjoying the pleasant nature. The ant throws the cricket out and states in a stern
UNIT 4

voice it should dance in the winter season too. In his concluding lines, the poet affirms that this is not
just a fable but it is true and applicable to human beings also.

And out of the door turned the poor little cricket


Folks call this fable. I’ll warrant it true

137
ENGLISH

UNIT

4
SUPPLEMENTARY
10
The Aged Mother
- Matsuo Basho
The Aged Mother

SUMMARY

“The Story of the Aged Mother” by Matsuo Basho is a story about a son and his aged widowed
mother. The son secretly fought against the decree of putting aged people to death.

Once there lived a poor farmer with his aged widowed mother. They owned a bit of land and they
were humble, peaceful and happy.

Their place Shining was governed by a despotic leader who proclaimed the decree to kill all the
aged people of the country. That time, killing aged people was not uncommon. When the poor farmer
heard the cruel decree he felt sad in his heart. Just as his work ended, he cooked food, wrapped it
l and sweet water.
Then, he carried his mother on his back and started walking towards the summit of Obatsayuma, the
mountain of the “abandoning of the aged”.

On their way to the summit, the helpless mother noticed the dangerous paths and thought that
her son might not know the way back and might pass through danger. So she reached for twigs,
snapped them to pieces and quietly dropped them all along the way until they reached the summit.

The poor farmer released his mother and created a comfortable cushion and tenderly lifted his
Supplementary

old mother onto it. With tearful eyes and painful heart, he then bade his farewell.

Before he left her, his unselfish and loving mother told him about the path she had made. She
told him to walk along the path which had the piles of twigs. Looking back over the path, the son was
surprised and he broke down before his mother. Then the farmer firmly told his mother he would not
leave her. Together they would follow the path of twigs and together they would die. So he carried his
mother again on his back and started walking back. When they reached home he hid her in the walled
closet for food beneath the kitchen floor. He supplied her with everything she needed.

As the time passed, the governor sent forth another unreasonable order that his people should
present him a rope of ashes. The farmer was confused and distressed. When he whispered to his
UNIT 4

mother, his mother gave him a clever idea. He must make a rope of twisted straw and then stretch it
upon a row of flat stones and burn it on a windless night. He called the people and did as his mother
said. When the blaze died down, they could see upon the stone a rope of ashes.

The governor was pleased by the intelligence of the youth and wanted to know how he got the
idea. The farmer told him that his old mother whom he had hidden gave the idea. Then the governor
meditated in silence and understood that old people had experience to solve problems and the country
138
needed them. At once he abolished the decree.
ENGLISH

B.P.No. 122
GLOSSARY

despotic (adj.) – tyrannical, cruel 10


proclamation (n) – announcement
barbarous (adj.) – extremely brutal or mercilessly harsh
abandon (v) – desert, give up completely
reverence (n) – deep respect

The Aged Mother


mandate (n) – an official order
summit (n) – the topmost point of a hill or mountain
injunction (n) – an order restraining someone from performing an act
shriveled (adj.) – wrinkled and contracted due to old age or due to strain
gravely (adv.) – seriously

A. Rearrange the sentences given below in the correct sequence.


1. The son made up his mind to take back his mother home. 5

3. The governor realized his mistake and abolished the law. 8

4. Once in Shining, a cruel ruler declared that all old people must be put to death. 1

5. Using the clever idea of his mother, the farmer made a rope of ashes. 7

6. When the farmer bade farewell, she advised him to return home with the aid of twigs. 4

7. Filled with dread, he hid his mother in his home. 6

Supplementary
8. The mother dropped the small twigs as markers on the way to help her son return. 3

B. Answer the following questions in one or two sentences.

1. What was the cruel announcement made by the leader?


The cruel announcement made by the leader was all the old people must be put to death.

2. Why was the farmer filled with sorrow?


The farmer was filled with sorrow because the poor farmer loved his aged mother and as per the
government decree he had to kill his old mother.
UNIT 4

3. What were the things carried by the farmer to the summit of the mountain?
The farmer carried cooked food, and cool and sweet water.

4. Why did the mother become anxious as they climbed up the mountain?
The mother became anxious as they climbed up the mountain because the paths crisscrossed one
another and her son might not know the mountain paths and his return might be dangerous.
139
ENGLISH

5. What did the mother drop along the way?


The mother broke the twigs and quietly dropped them all along the way as markers

6. What was the advice given by his mother for the safe return of her son?
10 When the son bade farewell to his mother, the mother told him about the path she had made with a
handful of broken twigs on the path. She advised him to walk along the path which had the piles of
twigs.

7. Why did the farmer’s burden seem to be light on his way back home?
The guilty conscience of leaving his mother to die was no more. Also his firm attitude to die together
The Aged Mother

did not make him feel his mother was burdensome. So the farmer’s burden seemed to be light on his
way back home.

8. Where did the farmer hide his mother?


The farmer hid his mother in the walled closet for food beneath the kitchen floor.

9. How did the farmer make the rope of ashes? On whose suggestion did he do it?
On his mother’s suggestion, the farmer made a rope with a twisted straw, and then stretched it upon
a row of flat stones and burnt it on a windless night. When the blaze died down, they could see a rope
of ashes on the stones.

10. How did the Governor realize his mistake?


dom to get a rope
of ashes, the farmer informed him that he had done it on the suggestion of his old mother whom he
hid in the closet. The governor was surprised and meditated upon his wrong order and abolished his
decree.

C. Answer each of the following in a paragraph of 120-150 words.


1. Narrate the circumstances that led to the abandoning of the aged in Shining.
Supplementary

The story of the Aged Mother - poor farmer and his aged mother – humble, peaceful life
– despotic leader – decree – felt sorry – took food and water – carried his mother to the
mountain – decided to leave her there – reached Obatsayuma – abandoning the aged

Title The Aged Mother


Author Matsuo Basho
Theme Old is gold

“The Story of the Aged Mother” by Matsuo Basho is a story about a son and his aged widowed
mother. He secretly fought against the decree of putting aged people to death. The story started at
the foot of the mountain where a poor farmer lived with his aged widowed mother. They owned a bit
UNIT 4

of land and they were humble, peaceful and happy. The country was governed by a despotic leader
who proclaimed the decree of killing aged people. That time, killing aged people was not uncommon.
When the poor farmer heard the decree he felt sorrow in his heart, because he loved his aged mother
with so much of reverence. Sorrowfully and painfully, he prepared to put his mother to death. Just as
his work ended, he cooked food, wrapped it in a small piece of cloth and tied it in his neck together
with a gourd filled with cool and sweet water. Then, he carried his mother on his back. The road was
140 narrow crisscrossed many times because of many paths made by the hunters and woodcutters. He
ENGLISH

kept blindly walking towards the summit of Obatsayuma, the mountain of the “abandoning of the
aged”. With tearful eyes and an aching heart, he bade farewell to his aged mother.

2. Describe the farmer’s painful journey up the mountain.


The Story of the Aged Mother – at the foot of mountain – peaceful and happy – governor’s
10
proclamation – farmer felt sad – prepared to put mother to death – carried cooked food
and water – carried his mother –road long and steep – narrow, crossed – paths made by
hunters – lost ways – reached the summit

Title The Aged Mother

The Aged Mother


Author Matsuo Basho
Theme Old is gold

“The Story of the Aged Mother” by Matsuo Basho is a story about a son and his aged widowed
mother. He secretly fought against the decree of putting aged people to death. The story started at
the foot of the mountain where a poor farmer lived with his aged widowed mother. They owned a bit
of land and they were humble, peaceful and happy. When the governor sent a proclamation that aged
people should be put to death, the poor farmer felt sad in his heart because he loved his aged mother
with so much. Shining at the time was ruled by the despotic leader and killing aged people was not
uncommon. Sorrowfully and painfully, he prepared to put his mother to death. Just as his work ended,
he cooked food, wrapped it in a small piece of cloth and tied it in his neck together with a gourd filled
g and steep. The
road was narrow and was crossed and re-crossed many times because of the many paths made by
the hunters and woodcutters. In some places they lost the way. He got confused but he gave no heed.
He kept blindly walking towards the summit of Obatsayuma, the mountain of the “abandoning of the
aged”.

3. ‘The old are wise’. Prove this with reference to the story ‘The Aged Mother’.

Supplementary
The Story of the Aged Mother – old is gold – wisdom – two occasions – mother guided her
son – to find out the way – to prepare the rope of ashes – remove the decree.

Title The Aged Mother


Author Matsuo Basho
Theme Old is gold

“The Story of the Aged Mother” by Matsuo Basho is a story about a son and his aged widowed
mother. He secretly fought against the decree of putting aged people to death.We know the famous
saying ‘Old is gold’. They are really gold because of their wisdom. The foundation for all the new things
in the present world was laid in the olden days. In the story, we see on two occasions, the old mother
UNIT 4

guiding her son the young farmer. First when the farmer carried her to the top of the steep and narrow
mountain, the old mother saw the paths crossed and re-crossed in many ways which might be very
dangerous for the son to get back home. So she broke the twigs to pieces and quietly dropped them all
along the way until they reached the summit. When the son bade farewell to his mother, the unselfish
and loving mother told him about the path she had made with a handful of broken twigs on the path.
She advised him to walk along the path which had the piles of twigs. The other occasion was when the 141
ENGLISH

governor passed the order to make a rope of ashes. She gave her son a clever plan to make to rope
with a twisted straw, and then stretched it upon a row of flat stones and burnt it on a windless night.
When the blaze died down, they could see a rope of ashes. This idea of the old mother abolished the

10 decree that old people should be put to death.

With the crown of snow there cometh wisdom

ADDITIONAL PARAGRAPH
The Aged Mother

Answer the following in a paragraph of 120-150 words.

1. Write the character sketch of the governor.


The antagonist governor – Shining – decree to kill the aged people – farmer’s love for
mother – carried – to mountain –aged mother’s help to find the way back – helped tofind
a way to prepare rope of ashes.

Title The Aged Mother


Author Matsuo Basho
Theme Old is gold

r. He is the central
character. He proclaimed the decree to kill the aged people. As he was a warrior he thought the aged
people were useless and could not be helpful to those who were still strong like him. So, all the people
put the aged people to death. But the farmer carried his mother on his back to the top of the mountain.
On another occasion, he sent out another proclamation that his subjects should make a rope of ashes.
It was very difficult for the people. But the old mother gave the farmer son a clever idea as to how
to make a rope with a twisted straw. The farmer demonstrated before everyone by stretching it upon
a row of flat stones and burning it on a windless night. When the blaze died down, they could see a
Supplementary

rope of ashes. It was this idea that transformed the despotic governor into a good person respecting
old people and considering them an asset to the kingdom. At once he abolished the decree against old
people.

D. Identify the character /speaker.


1. He gave orders for the aged to be put to death. - The governor
2. He considered the order to be the kindest mode of death. - Poor farmer
3. She quietly dropped some twigs on the way. - Aged mother
4. Let not thine eyes be blinded. - Aged mother
UNIT 4

5. Together we will follow the path, together we will die. - Poor farmer
6. I will think, I will think. - Aged mother
7. The truth must be told. - Poor farmer
8. He listened and meditated in silence. - The governor
9. Shining needs more than the strength of the youth. - The governor
142 10. With the crown of snow there cometh wisdom. - The governor
ENGLISH

E. Choose the appropriate answer and fill in the blanks.


1. Shining was governed by a _________ leader.
a) strict b) kind c) cruel d) diplomatic Ans: c)

2. The _________ was the principal food for the poor.


10
a) wheat b) brown rice c) unwhitened rice d) millet Ans: c)

3. The road was crossed and re-crossed by many paths made by the _________.
a) hunters and woodcutters b) robbers and thieves
c) vendors and tradesmen d) wildlife photographers and trekkers Ans: a)

The Aged Mother


4. Gathering _________ he made a soft cushion and tenderly lifted his old mother onto it.
a) dry leaves b) fallen pine c) broken twigs d) flowers Ans: b)

5. The governor demanded that his subjects should present him with a _________.
a) basket of fruits b) rope of ashes
c) flesh of animals d) bag of silverwares Ans: b)

ADDITIONAL EXERCISES

I.
I. 1. The son made up his mind to take back his mother home.
2. The mother dropped the small twigs as markers on the way to help her son return.
3. Once in Shining, a cruel ruler declared that all old people must be put to death.
4. When the farmer bade farewell, she advised him to return home with the aid of twigs.
5. A farmer decided to leave his old mother on top of a mountain. Answer: 3, 5, 2, 4, 1

Supplementary
II. 1.  hen the Governor came to know the wisdom of the old lady he removed his order of
W
killing the old people.
2. The farmer decided not to leave his mother on the mountain.
3. When the Governor ordered the people to make a rope of ashes, the mother helped them.
4. He hid her in the closet to avoid the neighbours’ attention.
5. He brought her back home.  Answer: 2, 5, 4, 3, 1

III. 1. The old mother of the farmer helped them to make a rope of ashes.
2. The Governor ordered the people to make a rope of ashes.
3. When he heard the wisdom of the old lady he removed his order of killing the old people.
4. People were confused how to make a rope of ashes.
UNIT 4

5. The governor was happy to get the rope of ashes. Answer: 2, 4, 1, 5, 3

II. Identify the Character.


1.  he stretched forth her hand and snapping the twigs
S
from bushes as they passed - Farmer’s Mother
2. Oh, honourable your kindness breaks my heart! - Farmer
143
ENGLISH

3. Shining needs more strength than of youth. - Emperor


4. I will think, I will think. - Farmer’s Mother
5. Alas! Alas! the truth must be told. - Farmer
10 III. Reading Comprehension.

Read the following passage and answer the questions below.

1.  The country Shining was governed by a despotic leader who though a warrior, had a great and
cowardly shrinking from anything suggestive of failing health and strength. This caused him to send
out a cruel proclamation. The entire province was given strict orders to immediately put to death all
The Aged Mother

aged people. Those were barbarous days, and the custom of abandoning old people to die was not
uncommon. The poor farmer loved his aged with tender reverence, and the order filled his heart with
sorrow. But no one ever thought twice about obeying the mandate of the governor, so with many deep
and hopeless sighs, the youth prepared for what at that time was considered the kindest mode of
death.

Questions :
1. Who governed Shining?
2. What was the cowardly thinking of the governor?
3. What proclamation did the governor send out?

5. What was the general attitude of the people?

Answers :
1. A despotic leader governed Shining.
2.  he cowardly thinking of the governor was all the people with sickness and failing health should
T
be put to death.
3.  he proclamation that the governor sent out was that all aged people should immediately be put
T
Supplementary

to death.
4. The farmer felt sorrowful because he loved his aged mother more than anything else.
5.  he general attitude of the people was that no one ever thought twice about obeying the
T
mandate of the governor.

IV. Passage for Comprehension.


1.  he eyes of the old mother were not so dim but that they noted the reckless hastening from one path
T
to another, and her loving heart grew anxious. Her son did not know the mountain’s many paths and
his return might be one of danger, so she stretched forth her hand and snapping the twigs from bushes
UNIT 4

as they passed, she quietly dropped a handful every few steps of the way so that they climbed, the
narrow path behind them was dotted at frequent intervals with tiny piles of twigs. At last the summit
was reached. Weary and heart sick, the youth gently released his burden and silently prepared a place
of comfort as his last duty to the loved one. Gathering fallen pine needles, he made a soft cushion and
tenderly lifted his old mother onto it. Her wrapped her padded coat more closely about the stooping
shoulders and with tearful eyes and an aching heart he said farewell.
144
ENGLISH

Questions :
1. How sharp were the eyes of the mother?
2. Why did she grow anxious?
3. What did she do for her son’s safe return? 10
4. How did he prepare the bed?
5. How did he bid farewell?

Answers :
1.  he eyes of the mother were so sharp that they noted the reckless hastening from one path to
T

The Aged Mother


another-crossing and recrossing of the paths.
2.  he grew anxious that her son might not return home safely, as he did not know the mountain’s
S
many paths.
3.  he snapped the twigs from bushes as they passed and quietly dropped a handful every few
S
steps of the way.
4. He gathered fallen pine needles and made a soft cushion.
5. He bade farewell with tearful eyes and an aching heart.

2.  he entire province trembled with dread. The order must be obeyed yet who in all Shining could make
T
a rope of ashes? One night, in great distress, the son whispered the news to his hidden mother. “Wait!”
she said. “I will think. I will think.” On the second day she told him what to do. “Make rope of twisted

“Then stretch it upon a row of flat stones and burn it on a windless night.” He called the people
together and did as she said and when the blaze died down, there upon the stones, with every twist
and fibre showing perfectly, lay a rope of ashes.
Questions :
1. How did the entire province react to the governor’s order?
2. What was the order?

Supplementary
3. What did the farmer discuss with his aged mother?
4. What idea did the mother give her son?
5. How did the farmer carry out the instructions of the aged old mother?

Answers :
1. The entire province trembled with fear.
2. The people of Shining should make a rope of ashes.
3. The farmer discussed with his aged mother how to make a rope of ashes.
4.  he mother told him to make a rope of twisted straw and then stretch it upon a row of flat stones
T
UNIT 4

and burn it on a windless night.


5.  he farmer called the people together and did as his aged mother instructed. When the blaze
T
died down, they could see a rope of ashes.

145
ENGLISH

UNIT

5
PROSE
10
Tech Bloomers

B.P.No. 125
WARM UP
Tech Bloomers

What do you infer from the above picture?


We infer from the above pictures that technologies have made life more comfortable and easier.
Technology has become an inseparable part of human life.
Prose

Look at the above electronic gadgets. If you were given a chance to remodel a device, in
which device you would bring in changes and what would be those changes?
If I were given a chance to remodel a device, I would bring in changes in Digital Dictionary with
separate pages for functional English, archaic words, literary terms and grammar.

SUMMARY
UNIT 5

We have never imagined that our refrigerator can order stuff on its own. Today it is possible
because of the development of technology. Our refrigerator can be connected to the ecommerce
site and it can order anything that is about to exhaust. In addition, managing entertainment and
home appliances by voice commands or using a finger is possible now. If we are bored with our TV
programme we can tell our Smart TV to view our social feed. If we are struck in a traffic jam, we can
146 make the kettle prepare tea for us.
ENGLISH

Technology has not only made a normal person’s life easier but it is also a boon to people
with inabilities. Alisha says that she cannot physically type as she thinks because she has cerebral
palsy. She uses a technology called Dragon Dictate. When she speaks, the words appear on her
screen and then she prints them out. Now she can even do Maths for her General Certificate of
Secondary Education (GCSE). Technology has opened up the world to her. She thanks Kim, the
10
Assistant Technologist of her school for having introduced her to Dragon Dictate. If someone has
limited movement he/she can control a computer screen with Eye Gaze or move from page to page
using the pupils of the eyes.
Twenty one year old David was born with Athetoid Cerebral Palsy. Technology enables
him to communicate with others and be independent. For verbal communication he uses Liberator

Tech Bloomers
Communication Device which he controls with eye movement. He has an ACTV controller in the
headrest of his chair in his bed room. He can control his TV, Bluesky and music players. He uses ECO2
with ECO point. He is also a keen sportsman regularly playing football, hockey and baseball. He now
uses ECO2 with ECO point to access the curriculum, study materials for his GCSE. He can also speak
complete sentences with correct syntax with the help of ECO2. He controls the play station with a
bespoke switch system, drives his electric wheel chair. He has recently been selected to travel to Brazil
to work with the Olympic opening ceremony team as part of the Remix Drama Group.
World renowned physicist Stephen Hawking is the best example to show how technology
can help a talented mind to overcome the physical impairments. The Assistive Technology helped him
to contribute productively to the world. Newer technology allows differently abled learners to learn
ng.

B.P.No. 130
GLOSSARY
grapple (v) - to fight, especially in order to win something
inclusion (n) -
the act of including someone or something
cerebral palsy (n) - permanent tightening of the muscles caused by damage to the
brain
Dragon Dictate (n) - a software which recognizes speech and converts it to text

Prose
assistive technologist (n) - a person who assists with technological gadgets to overcome
disability
gaze (v) - stare at something for a long time

Liberator Communication
Device (n) - a special device used to communicate with eye movements
cloister (adj) - enclosed by
UNIT 5

collaborative process (adj) - produced by or involving two or more parties working together

ADDITIONAL
threshold – entrance appliance – machine
stuff – matter current – present
147
ENGLISH

struck – caught amazing – surprising


aid – help
entire – complete
required – needed
boon – blessing
10 disability – handicapped
mounted – fixed
spare –
extra
barriers – blocks
deprived – denied response – reply
peers – friends
frustrating – annoying confident – sure
huge – large competent – capable
Tech Bloomers

achieve – attain bespoke – modified


passed away – died access – contact
rely on – depend on recently – newly
renowned – famous

EXERCISE WITH ANSWER

Choose the suitable synonym of the word underlined.

1. Anything which is below a certain threshold can be self-ordered by the appliance.


a) edge b) movement c) talent d) word Ans: a)

u.
a) planned b) observed c) seen d) caught Ans: d)

3. Your entire water and energy management can be taken care by automating all the
activities.
a) famous b) complete c) weak d) strong Ans: b)

4. India’s disabled are deprived by attitudinal barriers.


a) carriage b) work c) blocks d) simple Ans: c)
Prose

5. It’s made a huge difference to me.


a) big b) small c) decent d) near Ans: a)

6. He has been using a high tech communication aid since he was eight years old.
a) device b) tech c) help d) strength Ans: c)

7. He has fixed his gaze on the icon that is required.


a) hand b) look c) close d) fame Ans: b)
UNIT 5

8. It is essential that much of his spare time is spent in the performing arts!
a) object b) original c) sudden d) extra Ans: d)

9. He uses the ECO point Eye Gaze system to access the computer.
a) contact b) enter c) close d) support Ans: a)

10. David has recently been selected to travel to Brazil.


148 a) strongly b) happily c) surely d) lately Ans: d)
ENGLISH

EXERCISE for SELF EVALUATION

Choose the suitable synonym of the word underlined. 10


1. Anything which is below a certain threshold can be self-ordered by the appliance.
a) edge b) movement c) machine d) word Ans:

2. Your refrigerator can order for milk if is about to be exhausted.


a) drained b) controlled c) average d) denied Ans:

Tech Bloomers
3. Technology is a boon to citizens with special needs.
a) curse b) notorious c) famous d) blessing Ans:

4. India’s disabled are deprived by attitudinal barriers.


a) accepted b) denied c) depended d) confused Ans:

5. They continue to grapple with the challenges of access.


a) clutch b) reject c) proud c) happy Ans:

6. It would have been frustrating and difficult.


a) joyful b) defeating c) annoying d) easy Ans:

7.
a) accept b) reject c) attain d) need Ans:

8. Kim helps us use technology in different ways. She’s amazing.


a) correct b) sure c) happy d) astonishing Ans:

9. She has fixed his gaze on the icon that is required.


a) supported b) needed c) denied d) confused Ans:

10. It has been mounted on his walker.

Prose
a) fixed b) higher c) controlled d) thrown Ans:

11. His response was, “I like it, it makes me faster, when can I have one?”
a) demand b) reply c) occupation d) denial Ans:

12. He became a confident and competent communicator.


a) doubtful b) hesitant c) cynical d) sure Ans:

13. He became a confident and competent communicator.


UNIT 5

a) open b) slow c) capable d) fast Ans:

14. He controls his Play Station with a bespoke switch system.


a) modified b) fixed c) modern d) old Ans:

15. World renowned physicist Stephen Hawking is probably the best example.
a) spotted b) famous c) loved d) hated Ans: 149
ENGLISH

ANTONYMS

10 certain × doubtful
consumable × inconsumable, useless
frustrating × satisfying
huge × tiny
current × past achieve × abandon
entire × partly
limited × unrestricted
boon × curse
confident × doubtful
deprived × bestowed competent × incapable
Tech Bloomers

inclusion × exclusion recently × later

EXERCISE WITH ANSWER

Choose the suitable antonym of the word underlined.

1. Consumable products such as ink cartridges may be capable of self-ordering.


a) unconsumable b) inconsumable c) disconsumable d) imconcumable Ans: b)

2. Anything which is below certain threshold can be self-ordered by the appliance.


a) doubtful b) sure c) strong d) weak Ans: a)

nclusion.
a) injection b) exclusion c) imclusion d) information Ans: b)

4. It’s made a huge difference to me.


a) large b) big c) steady d) tiny Ans: d)

5. David has recently been selected to travel to Brazil.


a) slowly b) newly c) later d) surely Ans: c)
Prose

EXERCISE for SELF EVALUATION

Choose the suitable antonym of the word underlined.

1. David became a confident and competent communicator.


a) sure b) doubtful c) fast d) slow Ans:

2. David became a confident and competent communicator.


UNIT 5

a) incapable b) capable c) consultant d) fast Ans:

3. If someone has very limited movement they can control a computer screen
with Eye Gaze.
a) controlled b) unaware c) uncontrolled d) slow Ans:

4. It’s made me achieve things I only dreamt of.


150 a) accept b) know c) attain d) abandon Ans:
ENGLISH

5. It would have been frustrating and difficult.


a) annoying b) satisfying c) irritating d) vexing Ans:

6. India’s disabled are deprived by attitudinal barriers.


a) bestowed b) denied c) refused d) depressed Ans: 10
7. It is also a boon to citizens with special needs.
a) blessing b) praise c) curse d) burden Ans:

8. Your entire water and energy management can be taken care by automating
all the activities.
a) complete b) partly c) fully d) heavy Ans:

Tech Bloomers
9. The current level falls below a certain threshold.
a) present b) near c) slow d) past Ans:

10. I had to rely on someone to type everything into a computer for me.
a) independent b) promise c) trust d) depend Ans:

TEXTUAL QUESTIONS and ANSWERS

a. What is the future of technology? B.P.No. 126

ven the disabled


people can live with the help of technology independently.

b. How many people in India suffer with disability?


2.7 crore people suffer with disability.

c. Who is Kim? B.P.No. 127

Kim is the Assistant Technologist at Alisha’s school. She introduced her to Dragon Dictate.

d. How does Kim help Alisha?

Prose
Kim shows her how to train Dragon Dictate to understand her voice. Now she uses it in class and at
home for doing GCSE Maths.

e. Why is technology important according to David? B.P.No. 129

According to David technology is important because it enables him to communicate and be independent.
It gives him freedom.

f. Which instrument does David controls with his eye movement?


UNIT 5

David controls Liberator Communication Device with his eye movement.

g. What devices helps David to move from to one place to other?


AAC and ECO2 help David to move from one place to other.

151
ENGLISH

A. Answer the following questions in two or three sentences. B.P.No. 130

1. What are the benefits of the internet to the common man?

10 The benefits of the internet to the common man are that it gives instant access, provides endless
supply of knowledge and entertainment.

2. Do you think technology has improved communication? How?


Yes, technology has greatly improved communication. Advances in technology have led to the birth
of many new methods of electronic communication, such as social networking, websites and video
conferences. The increase of electronic communication has helped to eliminate time and distance that
Tech Bloomers

are obstacles to communication.

3. How does David operate computers with the Liberator Communication Device?
David controls the Liberator Communication Device with his eye movements.

4. Which devices are controlled using ACTIV controller?


Using ACTIV controller TV, BluRay and music players are controlled.

5. Who says these words: “I want everyone to know the difference technology has made in
my life”?
Alisha says these words to express her feelings about the new technology which has enabled her to

Dragon Dictate software helps Alisha to overcome her difficulty in typing.

7. Name a few Indian innovations which are helpful to the disabled and make their day to
day life easier.
Former President A.P.J. Abdul Kalam developed lightweight prosthetics materials to make artificial
limbs. Lechan Shoes by Krispian Lawrence, Blee Watch by Nupura Kiriloskar and Janhavi Joshi and
IGEST by Anil Prabhakar are some of the Indian innovations useful to the disabled.
Prose

8. Is it possible to control the computer screen with eye gaze?


Yes, it is possible to control the computer screen with eye gaze.

9. Suggest ways of making our society inclusive.


In the past differently abled people were alienated from the main stream of society. New technology
helps these people to learn and move with others. Differently abled people are brought back to the
society. This way our society is made inclusive.

10. How would you help the people with disabilities in your neighbourhood?
UNIT 5

I would help the disabled people by making the new technology accessible to them.

152
ENGLISH

B. Answer in a paragraph of about 100-150 words.

1. How do we use technology in our day to day lives?

Prose Tech Bloomers 10


Theme Empowering the disabled with technology

It has become appallingly obvious that our technology has exceeded our humanity.

We have never thought that our refrigerator can order stuff on its own. Today it is possible
when anything is below predetermined limit or certain threshold. Our refrigerator can directly link

Tech Bloomers
to the e-commerce site and order anything that is about to exhaust. For example, if we are running
out of milk, it can order milk. Consumable products such as ink cartridges may be capable of self-
ordering replacements when the current level falls below a certain threshold. In addition, managing
entertainment and home appliances by voice commands or by swapping a finger is a reality now. Also,
if we get bored by the program, we can tell our smart TV to view our social feed. If we are struck in a
traffic jam, we can make the kettle get some tea for us. Our entire water and energy management can
be taken care of by automating all the activities. Thus technology has made our lives more comfortable
and easier and happier.

Technology has not only made a normal person’s life easier


but it is also a boon to citizens with special needs.

2.

Prose Tech Bloomers


Theme Empowering the disabled with technology

Any sufficiently advanced technology is indistinguishable from magic.

Technology has not only made a normal person’s life easier but it is also a boon to people
with inabilities. Alisha says that she cannot physically type as she thinks because she has cerebral

Prose
palsy. She uses a technology called Dragon Dictate. When she speaks, the words appear on her
screen and then she prints them out. Now she can even do Maths for her General Certificate of
Secondary Education (GCSE). If someone has limited movement he/she can control a computer screen
with Eye Gaze or move from page to page using the pupils of the eyes. Twenty one year old David
was born with Athetoid Cerebral Palsy. Technology enables him to communicate with others and be
independent. For verbal communication he uses Liberator Communication Device which he controls
with eye movement. He has an ACTIV controller in the headrest of his chair in his bed room. He can
control his TV, Blue sky and music players. World renowned physicist Stephen Hawking is the best
UNIT 5

example to show how technology can help a talented mind to overcome the physical impairments. The
Assistive Technology helped him to contributing productively to the world. Newer technology allows
differently abled learners to learn with their peers as well as contributing fruitfully to the collaborative
process of learning.

David will now use his ECO2 to speak in


complete sentences with correct syntax.
153
ENGLISH

ADDITIONAL PARAGRAPHS

10 Answer in a paragraph of about 100-150 words.


1. How has Kim changed the life of Alisha.

Prose Tech Bloomers


Theme Empowering the disabled with technology

The purpose of the technology is not to confuse the brain but to help the body.
Tech Bloomers

Alisha says that she cannot physically type as fast as she thinks, because she has cerebral
palsy. It was Kim, the Assistive Technologist at Alisha’s school, who introduced her to Dragon Dictate
and it has opened up a new world to her. He showed her how to train it to understand her voice. It took
almost a few hours. Now she uses it in class and at home as well. It has made her more independent
and she is now able to study on her own. She can now do Maths GSCE herself without being dependent
on a computer. She never thought she would be able to do one GCSE in her life. She feels like she
can push herself even further. Kim says technology can help her do that. It is opening up the world
for young disabled people like her. There are many different types of technology that can help a
young disabled person become independent. If someone has a limited movement, they can control a
eir eyes. She says
about one person Kim, who works with all 42 students at her school and helps them use technology
in different ways. She feels that if she had not been introduced to technology, she would have lost so
many opportunities. All these things happened because of Kim.

I don’t know what we’d do without her we’d lose out on so many opportunities.

2. How is the twenty-one-year-old David able to do stunning things due to technology?

Prose Tech Bloomers


Theme Empowering the disabled with technology
Prose

The science of today is the technology of tomorrow.

Twenty-one-year-old David was born with Athetoid Cerebral Palsy. He says that technology is
very important because it enables him to communicate and be independent. For verbal communication,
he uses a Liberator Communication Device, which he controls with eye movements. He has an ACTIV
controller also in the headrest of his chair in his bedroom, which means he can control his TV, BluRay
and music players. He now uses an ECO2 with ECO point. He is also a keen sportsman, regularly
UNIT 5

playing football, boccia, hockey and baseball. Besides, he is a sports leader who uses his ECO2 linked
to an interactive white board to teach PE lessons. He has 144 icons on the screen, which he uses with
ECO point Eye Gaze. He now uses ECO2 and ECO point to access the curriculum, study for his GCSE,
order food and communicate. Also, he uses ECO2 to speak in complete sentences with correct syntax.
He controls the PlayStation with a bespoke switch system, drives his electric wheelchair and uses ECO
point Eye Gaze system to communicate, access the computer to check on how the Chelsea football is
154 doing and send and receive text messages.
ENGLISH

David also an advocate to other students who use AAC shows


then how easy it is to communicate using AC method.

B.P.No. 131
10
VOCABULARY

Abbreviations, Acronyms and Contractions


• Abbreviations and acronyms are shortened forms of words or phrases. An abbreviation is typically
a shortened form of words used to represent the whole (such as Dr. or Prof.) while an acronym

Tech Bloomers
contains a set of initial letters from a phrase that usually form another word (such as radar or
scuba).
• Abbreviations and acronyms are often interchanged, yet the two are quite distinct. The main
point of reference is that abbreviations are merely a series of letters while acronyms form new
words.
• We use contractions (I’m, we’re) in everyday speech and informal writing. Contractions, which
are sometimes called ‘short forms’, commonly combine a pronoun or noun and a verb, or a verb
and not, in a shorter form.

Contractions with I, you, he, she, it, we, and they

’re = are (you’re, we’re, they’re)


’s = is and has (he’s, she’s, it’s)
’ve = have (’ve, you’ve, we’ve, they’ve)
’ll = will (I’ll, you’ll, he’ll, she’ll, it’ll, we’ll, they’ll)
’d = had and would (I’d, you’d, he’d, she’d, it’d, we’d, they’d)

Contractions with auxiliary verb and not


The contraction for not is n’t:
aren’t = are not (we aren’t, you aren’t)

Prose
can’t = cannot
couldn’t = could not
didn’t = did not (I didn’t, they didn’t)

C. Pick out the contractions from the lesson and expand them.

Contractions Expansions
It’s It is
UNIT 5

You’re You are


I’m I am
Didn’t Did not
That’s That is
I’d be I would be
They’re They are
She’s She is 155
ENGLISH

D. Expand the following abbreviations or acronyms.

SIM ISRO WHO CCTV HDMI LASER MRI CRY RAM ROM CUP ALU

10 SIM Subscriber Identification Module


ISRO Indian Space Research Organization
WHO World Health Organization
CCTV Closed Circuit Television
HDMI High-Definition Multimedia Interface
Tech Bloomers

LASWER Light Amplification by Stimulated Emission of Radiation


MRI Magnetic Resonance Imaging.
CRY Child Relief and You
RAM Random Access Memory
ROM Read-only Memory
CUP Catalog Updated Process
ALU Arithmetic-Logic Unit

E. Complete the sentences with the correct abbreviations or acronyms from the given

a.m. etc. BCE e.g HD m IQ GPS p.m. vs

1. My dad wakes up very early in the morning because he has to be at work at 6.00 a.m.
2. Socrates, the famous Classical Greek Athenian philosopher, died in 399 BCE.
3. Leonardo Da Vinci was a famous Italian polymath, a painter, a sculptor, an architect, a musician,
a scientist etc.
4. I usually return home from work at 10.30 p.m.
5. John downloaded a clip from YouTube in HD quality.
Prose

6. There are many irregular verbs in the English language, e.g. break, do, make.
7. I’m watching a great football match, Barcelona vs Real Madrid.
8. Humans who dive without protection can survive 300 m under water.
9. A 11-year-old girl just beat Einstein on an IQ test.
10. We used the GPS facility to track the location.
UNIT 5

B.P.No. 134
LISTENING

F. Listen to the passage and state whether the statements are true (T), false (F) or no
information (N)?
1. Santhiya can’t live without her mobile phone. Ans: True
156 2. She got her mobile in January. Ans: False
ENGLISH

3. Her parents bought her the mobile phone one year ago. Ans: False
4. There’s a calculator in her mobile. Ans: True
5. She can connect to the Internet on her mobile. Ans: True
6. She usually listens to music on her mobile. Ans: False
10
7. She can’t read emails on her mobile. Ans: False
8. There are often a lot of problems with mobile phones. Ans: True
9. Santhiya always talks on her mobile to her friends. Ans: True
10. She doesn’t like mobile phones. Ans: False

Tech Bloomers
G. Listen to the passage again and answer the questions.

1. Where does Santhiya keep her mobile?


Santhiya keeps her mobile in her bag or in her pocket.

2. What can she use it for?


It has got a calculator in it so she can use it frequently to calculate. She can connect to the Internet
and look through the news or read emails.

3. When was she cycling?


She was cycling last year.

4.
When she was cycling last year with her friend on a holiday, it started to rain and it got dark. Suddenly
her friend fell off her bike and broke her leg.

5. How did Santhiya solve the problem?


When her friend fell off her bike and broke her leg, immediately Santhiya thought of her phone and
telephoned for help. And fifteen minutes later the doctor arrived.

H. Complete the sentences after reading the passage.

Prose
1. Santhiya’s parents and friends can always call her.
2. Her mobile phone is also a kind of information file.
3. On the cycling holiday after the accident Santhiya phoned for help.
4. Mobile phones often ring at the wrong moment.
5. Children can feel safe when they have their mobile phones with them.

B.P.No. 135
SPEAKING
UNIT 5

PUBLIC SPEAKING SKILLS


Public speaking involves communicating information before a large audience. In public speaking,
the information is purposeful and meant to inform, influence or entertain a group of listeners.

157
ENGLISH

I. Prepare on any one of the topics given below and present before your English teacher.

1. Prepare a welcome address on the occasion of Republic Day Celebration.

10 WELCOME ADDRESS-REPUBLIC DAY CELEBRATION


First of all, I would like to greet our respected Principal, our respected teachers, and my fellow
classmates with warmth of love and respect. We are gathered here to celebrate the 70th Republic Day.
I’d like to take this opportunity to with all of you a very Happy Republic Day. I welcome all the members
of our school management, our chairman, secretary and correspondent. I welcome our principal, our
teachers and my fellow students. I take this opportunity to welcome our parents, people from press
and all those who are present here.
Tech Bloomers

2. Prepare a vote of thanks on the occasion of Independence Day celebration.

VOTE OF THANKS
Honorable guest, Director of our Schools, Ms. Gita Doss, Respected Principal, Ms. Madhu
Chandra, our most beloved teachers, students, parents and well-wishers, I take this privilege to propose
a vote of thanks as the Student President, on the auspicious occasion of Independence Day of our
beloved country, India.
It has been a great day and we have had beautiful performances by the students, who
have displayed their spirit of patriotism. I want to thank all the teachers who have put in a lot of efforts
to make the celebrations a grand success.
days. I thank
every one of you for making this celebration a success. Thank you very much. Happy Independence
Day to everyone here. Jai Hind!

3. Mock anchoring for annual day celebration.


MOCK ANCHORING FOR ANNUAL DAY CELEBRATION
Nithya: I am delighted to welcome you all to our annual day celebration. We welcome all the dignitaries,
parents, invitees, our beloved teachers and our friends.
Arun: Education is not acquisition of knowledge but formation of character. These celebrations and
Prose

functions help us towards the formation of an ideal character. I once again welcome all those who are
present here.
Nithya: Today is a special day for us. We are celebrating our 20th annual day. We are grateful to all
those who have given their time for us.
Nithya: Now we have here our honourable chief guest Mr. Arnold, a scientist from ISRO. We acknowledge
the presence of our guest of honour Mr. Sunder Singh, retired principal of VNK Matriculation Higher
Secondary school, Kanyakumari.
UNIT 5

Arun: Let us begin our function with a prayer. I request Miss Bavana of std X to lead us in prayer.
Nithya: Thank you Miss Bavana. Now I request Master Sundaram of Std XII to give the formal
welcome speech.
Arun: Thank you Master Sundaram. Now I request the dignitaries to light the lamp.
Nithya: Than you all. Now let us all sit comfortably and get ready to enjoy the programmes of the
158 scholars of our school.
ENGLISH

4. Collect images of some electronic gadgets and play a JAM (just a minute) game by picking
one image and talking for a minute about it in your classroom.

10

Mobile phone is an electronic device, which is mainly used for communication. We can say
it is a wireless communication media. Now a days use of mobile is enhanced. People not only use it
for a conversation, but also they use it as an multimedia device. With it a person can click many kinds

Tech Bloomers
of pics, and can be connected to internet. A Sim card helps to communicate, and a memory card is
used to save the multimedia data on the mobile. For conversation, people should have a good mobile
handset, and a valid and activated Sim card in it. Also, we can communicate by using the message
(SMS / LMS) services is available in every mobile phone.

B.P.No. 139
READING
J. Read the comic strip and answer the following questions.
1. What do you mean by cyber safety?
hnology. It is not
only about keeping information safe and secure, but also about being responsible with that information,
being respectful to other people online, and using good ‘netiquette’ (internet etiquette).

2. How do you behave in a virtual platform?


In a virtual platform everybody has their own values according to their tradition and history. We need
to respect their values and sentiments as a part of global network.

3. Can we read and access the information free of cost?


No, we cannot read and access the information free of cost because everything in internet is owned
by someone.

Prose
4. Do you think that all the information online can be used without any permission?
No, all the information online requires permission. Copyrighted content can be used only after permission
and we need to give credits to the true owner.

5. Which website do you often access? Why?


I often access the Wikipedia website, because it has a wide range of information about many topics.

B.P.No. 139
WRITING
UNIT 5

e-mail
Short for electronic mail, e-mail or email is information stored on a computer that is exchanged
between two users over telecommunication. More plainly, e-mail is a message that may contain
text, files, images, or other attachments sent through a network to a specified individual or group of
159
individuals.
ENGLISH

K. Fill in the missing words in this email.

Dear Sir,
In response to your mail, I have prepared a presentation for the Science Fest. Please
10 find in the attachment the presentation for your kind perusal. I look forward to hearing from you.

Sincerely,
David.

L. Write an email to your teacher about the interesting English model that you have
prepared for the literary fest.
Tech Bloomers

To: cecilia2005@gmail.com
Cc: hariharan@gmail.com
Subject: English model for the Literary Fest
Dear Mam,
I would like to inform you that I have prepared an interesting English model for the literary fest
which is going to be held on December 20, 2020 at the school. I am attaching a copy of the same
for your kind perusal. Please make necessary corrections. I look forward to hearing from you.
Love,

MESSAGE WRITING
• A Message is an informal means of communication. The receiver of the message has to sift
through the given message and pick out the most vital bits of information.
• Then, he/she should be able to reproduce that information in order to convey it to the person
for whom it is intended.

M. Practice Exercise B.P.No. 141


Prose

You are the receptionist of your school. Your Headmaster instructs you to send a message
to all the parents of class ten to attend a PTA (Parent Teacher Association) meeting which
is to be held on 22.12.2020.

04 November 2020
3.30 p.m.

Dear Parents,
UNIT 5

Greetings.
PTA meeting will be held on 22nd of December 2020 at the school between 3.30 and
4.30 p.m. for the parents of class X. Kindly attend the meeting to discuss your child’s performance.

Meena,
Receptionist.
160
ENGLISH

CREATIVITY WRITING

Let us Become Blog writers.

• A blog is a type of website that focuses mainly on written content, also known as blog posts. In 10
popular culture we most often hear about news blogs or celebrity blog sites. Bloggers often write
from a personal perspective that allows them to connect directly with their readers.
• In addition, most blogs also have a “comments” section where readers can correspond with the
blogger. Interacting with your readers in the comments section helps to further the connection
between the blogger and the reader.

Tech Bloomers
N. Write about Your Favourite Sports person / Famous personality / Hobby / Recipe by
starting your own blog.
KNOWLEDGE EXPLOSION
Blog Series 2018:
M.S. Dhoni: Captain Cool
By Ramani at Knowledge Explosion Blogger
Mahendra Singh Dhoni was the former captain of the Indian Cricket team. He was born on
7 July 1981 in Ranchi, Jharkhand to Pan Singh and Devaki Davi. Dhoni studied at DAV Jawahar Vidya
Mandir, Shyamli, (now the school is known as JVM, Shyamli, Ranchi) Ranchi, Jharkhand where he
in these sports.
Dhoni was a goalkeeper for his football team and was sent to play cricket for local cricket club by his
football coach. Dhoni focused on cricket after his 10th standard. Dhoni is an aggressive right –handed
batsman and wicket-keeper.
Under his captaincy, India won the 2007 IIC World Twenty 20, CB Series of 2007-08 and
the Border-Gavaskar trophy 2008 in which they beat Australia 2-0. Dhoni has also been the recipient of
many awards including the IIC ODI player of the year award in 2008 (the first Indian player to achieve
this feat), the Rajiv Gandhi Khel Ratna award and the Padma Shri, India’s fourth highest civilian honour
in 2009. In the history of test match India became first time number one in December 2009 under the
leadership of Dhoni. In 2011 India won ICC World Cup cricket.

Prose
If you are interested in reading the full article, I invite you to sign up for the blog series at:
knowledgeexplosion.org
Feel free to share news of this blog series with your friends!

B.P.No. 142
GRAMMAR
UNIT 5

PRONOUNS
• A pronoun is a word or phrase that substitutes a noun or a noun phrase. There are ten types of
pronouns generally used.
Read the stories of Ravi and Rani.
Ravi is an intelligent boy. Ravi lives in a small village. A chill breeze touches the skin, a cool lake
with swans swimming on the lake catches the eyesight. Ravi loves nature a lot. Ravi is studying in class 161
ENGLISH

ten in a government school. Ravi loves helping others. When a woman was crossing the road with
heavy luggage, Ravi asked the woman, “May I help you?” and carried the luggage and dropped the
luggage at home. The woman thanked Ravi for the help.

10 Rani is a brilliant girl. Rani lives in an urban area where huge buildings touch the sky, buzzing
noise of traffic hit the ears and crowds move busily towards work. One day when Rani was on the way
to school, Rani saw a dog hurt by a moving scooter. At once Rani went near the dog, lifted the dog and
rushed to a veterinary doctor. The dog, after recovering, shook the tail to thank Rani.

A. Write the words that can replace Ravi, Rani, woman, luggage and the dog when we use
them for the second and subsequent times in the passage _____, _____, _____.
Answer: He, She, she, it, it.
Tech Bloomers

These words are called pronouns.


B. Fill in the gaps with personal pronouns.
Kumaravel lives in Thiruvannamalai. He is a doctor. All the people like him because of his
helping nature. His hospital is located at Car street and most of his patients are poor so he does not
charge much money. His daughter goes to school. She studies in 5th Standard. Her teachers love her
very much. Her friends are also very good. They always encourage her. They have given her good
advice. Her mother is also a teacher. She always encourages her to keep studying. I also like her as
she often comes to my house. One day I told my mother that she wants to learn cooking. My mother
taught her cooking. Now she cooks well.

1. It is an excellent opportunity.
2. One of these two students can solve this question.
3. These books have been written by a great Indian writer.
4. They have come to know the truth.
5. Some of the students have passed the exam.
6. Some of your friends can guide you.
Prose

7. This is your story based on your real life.


8. All your friends will guide you.
9. Some of his family members would come to visit you.
10. Some of those books will be helpful to you.
11. This is your bag, you can take it anytime.
12. He himself is responsible for the downfall of his life.
UNIT 5

D. Join the sentences using ‘Relative Pronouns’.

1. I have book. It is written by Rabindranath Tagore.

I have the book which is written by Rabindranath Tagore.

2. Kavita is my teacher. She teaches us English.


162 Kavita is my teacher who teaches us English.
ENGLISH

3. This is Varun. His father is an architect.


This is Varun whose father is an architect.

4. She invited most of her friends. They attended the party.


Most of her friends whom she invited attended the party.
10
5. Give me a pen to write a letter. It was gifted to you on your birthday.
To write a letter give me the pen which was gifted to you on your birthday.

6. I have sold the house. It was located at the bank of a river.


I have sold the house which was located at the bank of a river.

Tech Bloomers
7. Here is your watch. It has been found in the garden.
Here is your watch which has been found in the garden.

REPORTED SPEECH
• There are two main types of speech: direct speech and indirect or reported speech.
• Direct speech repeats the exact words the person used, or how we remember their words.
• Reported speech is how we represent the speech of other people or what we ourselves say.

E. Read the different verb forms where they remain the same in the direct and indirect
speech in the following cases. Fill in the blanks with missing indirect speech.
B.P.No. 145 1.

Kavi says that she will never go to work.

2. When we report a universal truth (something that is always true).


Alisha said that people in Africa are starving.

3. With modal verbs would, might, could, should, ought to, used to.
Vinoth said that he would try it.

4. With would rather, had better


Sophia said that they had better go.

Prose
5. In if-clauses and time-clauses
Jaheer said that when I was staying in Madurai I met my best friends.
Jaheer said that when he had been staying in Madurai he had met his best friend.

6. We do not usually change the modal verbs ‘must’ and ‘needn’t’. But ‘must’ can become ‘had to’ or
‘would have to’ and ‘needn’t’ can become ‘didn’t have to’ or ‘wouldn’t have to’ if we want to
express an obligation. ‘Would/wouldn’t have to’ are used to talk about future obligations.
UNIT 5

Ans: He said that they had to done it in June.

F. Read the following dialogue and report it.

Johnson : “What are you doing here, Suganthi? I haven’t seen you since June.”
Suganthi : “I’ve just come back from my holiday in Ooty.”
Johnson : “Did you enjoy it?”
163
ENGLISH

Suganthi : “I love Ooty. And the people were so friendly.”


Johnson : “Did you go to Coakers Walk?”
Suganthi : “It was my first trip. I can show you some pictures. Are you doing anything tomorrow?”
10 Johnson : “I must arrange a couple of things. But I am free tonight.”
Suganthi : “You might come to my place. At what time shall we meet?”
Johnson : “I’ll be there at eight. Is it all right?”
Johnson asked Suganthi what she was doing there. And he said that he had not seen her since
the Machines

June. Suganthi explained that she had just come back from her holiday in Ooty. Johnson wondered
if she had enjoyed it. Suganthi told him that she loved Ooty and that the people had been so friendly.
Bloomers

Johnson wanted to know if she had gone to the Coakers Walk. Suganthi said that it had been her first
trip and that she could show him some pictures. And then she asked him if he was doing anything the
next day. Johnson explained that he must arrange a couple of things. But he added that he was free
at night. Suganthi suggested that he should come to his place and asked him at what time they could
meet. Johnson said he could be there at eight. And finally he asked whether it was all right.
Tech of

G. Tick the right choice (Indirect Speech).


The Secret

Direct Speech Indirect Speech


1. “Who took my English book?” a. took my English
He was curious to know who... b. had taken his English book. 
c. takes his English book.
d. has taken my English book.

2. “Where does Helen live?” a. Helen lived.


Jim wants to know where... b. Helen lives. 
c. Helen had lived.
d. does Helen live?

3. “Why do volcanoes erupt?” a. volcanoes erupt.


Prose
Poem

She wondered why... b. volcanoes had erupted.


c. volcanoes erupted. 
d. did volcanoes erupt?

4. “Do you know why she is unhappy?” a. I know why she is


He asked me if ..... unhappy b. you know why she was
c. did I know why she was
d. I knew why she was
UNIT 5


5. “How many photos have you got?” a. photos I had got.
He wants to know how many.... b. photos you have got.
c. photos had I got?
d. photos I have got. 
164
ENGLISH

UNIT

5
POEM
10
The Secret of the Machines
- Rudyard Kipling

Machines
theMachines
SUMMARY

Bloomers
The poem ‘The Secret of the Machine’ by Rudyard Kipling deals with the problem of modern
technology and automation. In the beginning, the reader is informed about how machines are produced

the
and what kind of treatment they need. Afterwards the machines explain how they can serve humanity.
But machines are not only useful, but also they can lead to big disasters, if they are not used in the

Techofof
right way.

Secret
TheSecret
The first stanza states that machines were taken from the ore-bed and the mine. They were
melted in the furnace and the pit. They were hammered to design. They were cut and filed to fit in
its place. The second stanza explains that all they require are some water, coal and oil. They require
some space to set them work and start working immediately. If they are set properly they will serve

The
The third stanza enumerates the various types of tasks different type of machines do in our
life. They can pull, haul, lift, drive, print, plough, wave, heat, light, run, swim, fly, hear, count, read
and write. These tasks are done by machines in different workplaces faster and accurately. The fourth
stanza reminds us that they are not built to tell lies. They neither love nor pity nor forgive. They do
not have feelings. If human beings make a mistake in handling them, they will be dangerous – they
will kill the people.
The final stanza reminds us the irony of the machines. They help us a lot in our life’s activities.
Smoke may be created by us and it may block the sight of the sky. But it is for only a few minutes

Prose
Poem
and then the sky will be clear. They confess one thing that all their power, weight and size are nothing
compared to human brain. It is the human brain that creates them. They are the children of the
human brain.

B.P.No. 150
GLOSSARY
furnace (n) – an enclosed structure in which material is heated to very high temperatures
UNIT 5

wrought (adj.) – beaten out of shape by hammering


gauge (n) – an instrument that measures and gives a visual display of the amount,
level, or contents of something.
thousandth (adv.) – a fraction of thousand
haul (v) – pull or drag with effort or force
comprehend(v) – grasp, understand
vanish(v) – disappear suddenly and completely 165
ENGLISH

ADDITIONAL

10 melted
cast


having become liquefied by heating
shaped
wrought – beaten out / shaped
hammered – beaten out / hit
The Secret of the Machines

gauged – measured
filed – burnished / sharpened
haul – pull forcefully
comprehend – understand
vanish – disappear

A. Answer the following questions briefly. B.P.No. 150

1. Who does ‘we’ refer to in first stanza?


a) Human beings b) Machines Ans: b)

2. Who are the speakers and listeners of this poem?


The speakers are the machines and the listeners are the human beings.

3. What metals are obtained from ores and mines?


Iron is obtained from ores and mines.

4. Mention a few machines which are hammered to design.


Tractor, bulldozer, crane, truck, printer, etc.

5. Mention the names of a few machines that run on water, coal or oil.
Underwater treadmill, trains, cars, trucks, aeroplane, etc., are some of the machines that run on water,
coal and oil.

6. Mention a few machines used for pulling, pushing, lifting, driving, printing, ploughing,
Poem

reading, and writing, etc.


Tractor, bulldozer, crane, truck, printer, cars, Braille etc, are used for the above-mentioned purposes.

7. Are machines humble to accept the evolution of human brain? Why?


Yes, machines are humble to accept the evolution of human brain because in the last stanza of the
poem they confess that they are the children of human brain.

8. What feelings are evoked in us by the machines in this poem?


UNIT 5

Compassion, sympathy, humility and vanity are the feelings evoked in us by the machines in this poem.

9. ‘And a thousandth of an inch to give us play:’


Which of the following do the machines want to prove from this line?
a. Once Machines are fed with fuel, they take a very long time to start.
b. Once Machines are fed with fuel, they start quickly. Ans: a)
166
ENGLISH

10. And now, if you will set us to our task,


We will serve you four and twenty hours a day!
a. Who does the pronoun ‘you’ refer to here?
‘You’ refers to human beings here. 10
b. Whose task is referred to as ‘our task’ here?
The Machines’ task is referred to here.

The Secret of the Machines


c. Open conditional clause is used in the given line. Why is the future tense ‘will set’
and ‘will serve’ used both in the ‘if clause’ and in the ‘main clause’?
Open conditional clause usually refers to a future event which is conditional on another future
event. The verb of the main clause is in the future tense with “will” and the if-clause also is in
the future tense. Here it is used because if the condition is fulfilled, the consequent action will
automatically happen without fail.

d. Do the machines serve us twenty-four hours a day?


Yes, the machines serve us twenty-four hours a day.

e. Rewrite the given lines with the ending ‘365 days a year’.
We will serve you three hundred and sixty five days a year.

Answer the following questions.

1. We were cast and wrought and hammared to design,


We were cut and filed and troled and guaged to fit.
a. Who does ‘we’ refer to?
‘We’ refers to the modern machines.

b. Who can design the machines?

Poem
Man can design the machines.

2. And now, if you will set us to our task,


We will serve you four and twenty hours a day!
a. Who will serve us for a whole day?
The machine will serve us for a whole day.

b. Will the machine do any task without human being?


UNIT 5

No, it will not do.

3. We can neither love nor pity nor forgive,


If you make a slip in handling us you die!
a. Do the machines have any feeling?
No, the machines don’t have any feelings or emotions. 167
ENGLISH

b. What do you mean by the word ‘slip’ here?


It means handling the machines in a wrong way.

4. We can see, and run and hear and count and read and write.
10 a. Name the figure of speech used here
Personification.
b. Identify the words in alliteration.
The Secret of the Machines

We, write; run, read.

5. We can pull and haul and push and lift and drive
We can print and plough and weave and heat and light.
We can run and race and swim and fly and dive.
We can see and hear and count and read and write!
a. Pick out the rhyming words in these lines.
drive, dive; light, write.
b. Pick out the rhyme scheme in these lines.
abab.

B. Write your favourite stanza from the poem and find the rhyming scheme.
But remember, please, the Law by which we live,
We are not built to comprehend a lie,

If you make a slip in handling us you die!


The rhyme scheme of this stanza is abab.

C. Read the poem and find the lines for the following poetic devices or write your own
example.

Alliteration
pull, push; print, plough; run, race; stars, shine

Assonance
Poem

all weak
a thousand of an inch

Personification
We can run and race and swim and fly and dive
We can see and hear and count and read and write
We will serve you
UNIT 5

Simile
He is as busy as a bee
I am as snug as a bug in a rug
You were as brave as a lion.
They fought like cats and dogs.
168 He is as funny as a barrel of monkeys.
ENGLISH

ADDITIONAL PARAGRAPH
Answer the following question in about 80-120 words. 10
1. How do machines confess they are inferior to human brain?
Poem The Secret of the Machines
Poet Rudyard Kipling

The Secret of the Machines


Theme Machines are the creation of human brain

We were taken from the ore-bed and the mine,


We were melted in the furnace an the pit.

The poem “The secret of the machines” by Rudyard Kipling deals with the problem of
modern technology and automation. If we set them to work, they will serve us twenty four hours a day.
The Machines explain various types of tasks that different kinds of machines do in our life. They can
pull, haul, push, lift, drive, print, plough, wave, heat, light, run, race, swim, fly, dive see, hear, count,
read and write. These tasks are done by the machines in different workplaces faster and accurately in
our life. But they remind us that they are not built to tell a lie. They neither love, nor pity nor forgive.
They don’t have feelings. As they have been created, they work. If human beings make a mistake in
handling them, they will be dangerous and we will die. But they confess that all their power, weight
and size are nothing compared to human brain. It is the human brain which creates them. They are

Because, for all our power and weight and size


We are nothing more than children of your brain!

2. How do machines confess they are inferior to human brain?


Poem The Secret of the Machines
Poet Rudyard Kipling
Theme Machines are the creation of human brain

Some water. Coal and oil is all we ask

Poem
And a thousand of an inch to give us play

The poem ‘The secret of the Machine’ by Rudyard Kipling deals with the problem of
modern technology and automation. In the beginning, the reader is informed about how machines
are produced and what kind of treatment they need. Afterwards the machines explain how they can
serve humanity. But machines are not only useful, but also they can lead to big disasters, if they are
not used in the right way. All they require are some water, coal and oil. They require some space to
set them work and start working immediately. If they are set properly they will serve people twenty
four hours a day. The poet enumerates the various types of tasks different type of machines do in our
UNIT 5

life. They can pull, haul, lift, drive, print, plough, wave, heat, light, run, swim, fly, hear, count, read and
write. These tasks are done by machines in different workplaces faster and accurately. They are not
built to tell lies. They neither love nor pity nor forgive. They do not have feelings. If human beings
make a mistake in handling them, they will be dangerous – they will kill the people. They help us a lot
in our activities.
We can neither love nor pity nor forgive,
If you make a slip in handling us you die! 169
ENGLISH

UNIT

5
SUPPLEMENTARY
10 A day in 2889 of
an American Journalist
A day in 2889 of an American Journalist

- Jules Verne

SUMMARY

‘A Day in 2889 of an American Journalist’ written by Jules Verne is a science fiction which
presents what will happen in 2889 due to development of technology. Here he connects earth and
other planets. On 25th July morning in the year 2889, Francis Bennet, Manager Editor of the Earth
Herald switched on his phonotelephote. His wife went to France eight days ago. He saw his wife
appear in the telephote mirror. His name was spoken by the sweet voice. Immediately he jumped out
of bed and went inside this mechanised room where the machine washed, shaved and dressed him.
He went into the reporters’ room where fifteen hundred reporters were working. Each reporter had a
telephone and a series of communications.
in the stellar
world. He said that they got phototelegrams from Mercury, Venus and Mars but they did not get
anything from the Jupiter and the Moon. Because of the moon’s position they could not get information
from the moon. So they decided to turn the satellite. He was satisfied with the work in his office.
When the clock struck twelve the director of Earth Herald left and sat in a rolling armchair.
In a few minutes he reached the dining hall. The table was laid and he took his place. On the
phonotelephote their dining room appeared. Mr. and Mrs. Bennet had arranged to have lunch at the
same time. Food would come to him through a network of pneumatic tubes. When he finished his
Supplementary

lunch, he went across space by aero-car. After visiting Niagara, he returned to New York by way of
Philadelphia and Boston.
A huge crowd waited to receive him at the waiting room of the Earth Herald. The crowd
came up with a lot of proposals. Among them was a young man with a high degree of intelligence. He
said he was going to make a complete human being without a soul. Francis Bennet assigned him to
the scientific editorial department of his journal. The second visitor came up with the idea of moving
a whole city in a single block – he would transform into a seaside resort.
During his meal, phonotelephotic communication had been set up with Paris. His wife
appeared from France. She said that she would return by tube at eleven-fifty that evening.
UNIT 5

B.P.No. 158
GLOSSARY
phonotelephote (n) - A means of transmitting and receiving both voice and picture for a
personal conversation
170 Champs-Elysees (n) - a prestigious avenue in Paris, famous for the Cafes, cinemas and shops
ENGLISH

valet(n) - a man’s personal male attendant, who is responsible for his clothes and
appearance
phototelegrams (n) - A telegraphic transmission including images
jovians (n) - inhabitants of planet Jupiter 10
inhabitants(n) - a person or animal that lives in or occupies a place.
Centropolis (n) - (in this story) the new name of New York in 2889

A day in 2889 of an American Journalist


A. Answer the following questions in two or three lines.

1. Why did Francis Bennett wake up with a bad temper?


Since his wife went to France eight days ago, Francis Bennett was feeling lonely. So he woke up with
a bad temper.

2. What was a mechanized dressing room?


A mechanized dressing room had a machine for dressing a human being like washing, shaving, shoeing,
dressing and buttoning from top to toe. The machine was in his office.

3. How was food served to him?


Food was served to him through a network of pneumatic tubes.

4. Why was Bennett curious about astronomy?


unning discoveries
and inventions about new planet, distances and intricate details with greater precision.

5. Why did he visit Niagara?


He visited Niagara because he wanted to see how his accumulator worked at Niagara.

6. How did Bennett travel?


Bennett travelled by aero-car.

Supplementary
7. Give three instances of how mechanization has changed life at home in 2889.
First instance is that vision could be transmitted because of which Bennet was able to see his wife
in the telephotic mirror. Second instance was the dressing room where washing, shaving, shoeing,
dressing and buttoning were done by a machine. Third instance was the travel by aero-car or tube
which was faster compared with the past modes of travel.

8. How is advertising in this age different from what we have today?


The advertising gallery was broad about a quarter of a mile long, where there were thousands of
projectors for sending these advertisements to the clouds. These gigantic signs get reflected on the
UNIT 5

clouds so large that they can be seen all over a whole country.

B. Identify the character/speaker.


1. As soon as he woke up, he switched on his phonotelephote. Ans: Francis Bennett
2. Well, Cash, what have you got? Ans: Francis Bennett
3. ‘Phototelegrams from Mercury, Venus and Mars, Sir’. Ans: Cash
4. ‘Interesting! And Jupiter?’ Ans: Francis Bennett 171
ENGLISH

5. ‘Not, yet, Mr. Bennett.’ Ans: Cash


6. ‘No, it’s the inhabitants.’ Ans: Corley
7. ‘Where are we going, Sir?’ Ans: Aero-coachman
10 8. ‘Then, Sir, I shall really have discovered the absolute.’ Ans: A young man
9. ‘Are you saying you’re going to be able to construct a human being?’ Ans: Francis Bennett
10. ‘I’m going to start this moment.’ Ans: Edith
A day in 2889 of an American Journalist

C. Choose the best answer.


1. Bennette’s wife was in _______.
i) Germany ii) Australia iii) France iv) Holland Ans: (iii)

2. The data from the stellar world was gathered by _______.


i) Bennette ii) astronomical reporters
iii) the computer iv) telephote Ans: (ii)

3. The food was being delivered through _______ tubes.


i) pneumatic ii) shallow iii) hollow iv) virtual Ans: (i)

4. The wayfarers were carried to one place to another by the _______.


i) bullet train ii) jet
Ans: (iii)

D. Fill in the story map given below.

Title A day in 2889 of an American journalist by Jule Verne


Setting Year - 2889
Date - 25th July
Place - office block of the Managing Editor of the Earth Herald
Characters Francis Bennet, Edith, Aero-Coachman and Astronomical reporters.
Supplementary

Problems Indifferent to the presence of each marvel


Your views I feel relationships will get affected and man will become lazy when obsessed with
machines.

ADDITIONAL EXERCISES

I. Rearrange the sentences in coherent order.


UNIT 5

I 1. His wife appeared on the screen.


2. His wife went to France eight days ago.
3. Francis Bennet was the Manager Editor of Earth Herald.
4. Then he heard his name being called and immediately went to the mechanised room.
5. He switched on the phototelephote. Answer: 3, 2, 5, 1, 4
172
ENGLISH

II 1. But they did not get anything from the Jupitar and the Moon.
2. He questioned one of the astronomical reporters about the recent discoveries.
3. He said that they got phototelegrams from Mercury, Venus and Mars.
4. Francis Bennet went into the reporters’ room. 10
5. As the Moon’s position was not helping them, they decided to turn the satellite.
 Answer: 4, 2, 3, 1, 5

A day in 2889 of an American Journalist


III 1. The director of Earth Herald went to the dining hall.
2. When he finished his lunch, he went across space by aero-car.
3. The clock struck twelve.
4. After visiting Niagara, he returned to New York by way of Philadelphia and Boston.
5. Food would come to him through a network of pneumatic tubes. Answer: 3, 1, 5, 2, 4

II. Identify the Character.


1. He saw his wife appear in the telephotic mirror. – Francis Bennett
2. ‘No, it’s the inhabitants’ – Corley
3. You know what a passion the public has for these
astronomical questions. – Francis Bennett
4. ‘I’m going to start this moment’. – Edith
t

III. Reading Comprehension.


Read the following passage and answer the questions given below.
1. And that very day, the scientists of the Bennett factory started working out some mechanical
means of turning out satellite right round. On the whole, Francis Bennett had reason to be satisfied.
One of the Earth Herald’s astronomers had just determined the elements of the new planet Gandini.

Supplementary
It is at a distance of 12,841,348,284,623 metres and 7 decimetres that this planet describes its orbit
round the sun in 572 years, 194 days, 12 hours, 43 minutes, 9.8 seconds. Francis Bennett was delighted
with such precision. Good! He exclaimed. ‘Hurry up and tell the reportage service about it. You know
what a passion the public has for these astronomical questions. I’m anxious for the news to appear in
today’s issue.

Questions :
1. What was the work the Bennett factory was doing?
2. What gave Bennett Francis satisfaction?
3. What is the distance of the new planet Gandini?
UNIT 5

4. Was Bennett delighted with the precision of the reportage?


5. For what did the public have passion?

Answers :
1. The work of the Bennett factory was finding out mechanical means of turning out satellite right
round.
2. The works of the scientists gave Bennett Francis satisfaction. 173
ENGLISH

3. The new planet Gandini is at a distance of 12,841,348,284,623 metres and 7 decimetres.


4. Yes, Bennett was delighted with the precision of the reportage.
5. The public had a passion for astronomical questions.
10 IV. Passage for Comprehension.

1. The next room, a broad gallery about a quarter of a mile long, was devoted to publicity, and it
A day in 2889 of an American Journalist

well may be imagined what the publicity for such a journal as the Earth Herald had to be. It brought in a
daily average of three million dollars. They are gigantic signs reflected on the clouds, so large that they
can be seen all over a whole country. For that gallery a thousand projectors were unceasingly employed
in sending to the clouds, on which they were reproduced in colour, these inordinate advertisements.
At that moment the clock struck twelve. The director of the Earth Herald left the hall and sat down in
a rolling armchair. In a few minutes he had reached his dining room half a mile away, at the far end of
the office.

Questions :
1. What was the next room devoted to?
2. What was the breadth of the gallery?
3. How much did the gallery bring in?
4. How was advertisement done here?

Answers :
1. The next room was devoted to publicity.
2. The breadth of the gallery was about a quarter of a mile long.
3. The gallery brought in a daily average of three million dollars.
4. For the gallery a thousand projectors were unceasingly employed in sending to the clouds, on
Supplementary

which they were reproduced in colour.


5. The director of the Earth sat in a rolling armchair.

2. Like everybody else in easy circumstances nowadays, Francis Bennett, having abandoned
domestic cooking, in one of the subscribers to the Society for Supplying Food to the Home, which
distributes dishes of a thousand types through a network of pneumatic tubes. This system is expensive,
no doubt, but the cooking is better. So, not without some regret, Francis Bennett was lunching in
solitude. He was finishing his coffee when Mrs Bennett, having got back home, appeared in the
telephote screen.

Questions :
UNIT 5

1. What did Francis Bennett give up?


2. What did he subscribe to?
3. How does the Society for Supplying Food to the Home distribute dishes?
4. Why did Francis Bennett regret?
5. Where did Mrs Bennett appear?
174
ENGLISH

Answers :
1. Francis Bennett gave up domestic cooking.
2. He subscribed to the Society for Supplying Food to the Home.
3. The Society for Supplying Food to the Home distributes dishes through a network of pneumatic tubes. 10
4. Francis Bennett regretted lunching in solitude.
5. Mrs Bennett appeared in the telephote screen.

A day in 2889 of an American Journalist


ADDITIONAL PARAGRAPH
Answer the following question in about 100-150 words:

1. Discuss how the twenty ninth century world is different from the present.
twenty ninth century fairyland – machine to dress up and bath – phonotelephotic – see the
images of the other party – machine with a tube to get food – facilities to get information
from other planets – travel with aero-cars and pneumatic tubes – faster than human brain
imagines – communicate with phototelegrams – such things are not in present

Title A Day in 2889 of an American Journalist


Author Jules Verne
Incredible Technological advancement in 29th century.

The men of the twenty-ninth century live in a perpetual fairyland, though they do not
seem to realise it. Bored with wonders, they are cold towards everything that progress brings them
every day. If they compare it with the past, these cities are ultra modern, with streets a hundred yards
wide, with buildings a thousand feet high, always at an equable temperature, and the sky furrowed by
thousands of aero-cars and aero-buses! These towns have population up to ten million inhabitants. In
terms of travel, people of twenty ninth century travel faster than what human brain can imagine by the

Supplementary
aero-cars, aero-trains and especially these tubes laid beneath the oceans. They transport them with a
speed of a thousand miles an hour. And in terms of communication, people are able to communicate
with advanced technology using phonotelephote, phototelegrams and so on and feel proximity even if
they are faraway. In the past people had servants to bathe, dress, wash, shave and button. But in this
era machines are doing all kinds of things that men did in the past. It is amazing to know that a broad
gallery about a quarter of a mile long is devoted to publicity. Advertisements are reflected on the clouds
so large that their can be seen all over a whole country.
UNIT 5

175
ENGLISH

UNIT

6
PROSE
10
The Last Lesson
- Alphonse Daudet

B.P.No. 161
WARM UP
The Last Lesson

A. Fill the globe with an acrostic poem. B. How can you make the world a better
place?
People of the world should unite to
 y keeping the world war - free and pollution
B
End all the wars and fighting free, we can make the world a better
place. Universal brotherhood and peaceful
And should they help each other to
co-existence will make it a better place.
Come together to shake hands to make

C. What does the picture symbolise?


Peace.

D. Find the name of the various UN organizations with the logo given below and fill in
the blanks.
Prose

International Labour World Food Programme (WFP) United Nation Environment


Organisation (ILO) Programme
UNIT 6

The United Nations Children’s United Nations Human Rights World Health Organization
Fund (UNICEF)
176
ENGLISH

SUMMARY
The narrator of the story, a little boy Franz, rushed to school one morning. He was in dread 10
of a scolding from his teacher, M. Hamel, who would question him on participles.
When he reached the school he saw a crowd gathered around the bulletin-board. Though
he reached the school a little late he was surprised to note that the school was very quiet. Through
a window, the narrator saw his classmates already seated, and his teacher M. Hamel walking up and
down with his ruler under his arm. To his surprise, M. Hamel spoke to the narrator kindly. Franz took
his seat, and noticed that M. Hamel had clothes on for special occasion.

The Last Lesson


The atmosphere of the classroom was strange. Franz saw some villagers sitting in the
class. M. Hamel sat on his chair and said that it would be the last French lesson. An order had arrived
from Berlin that from the next day onward, only German would be taught in the schools of Alsace-
Lorraine. These words came as a shock to little Franz, who realized that the villagers he had seen by
the bulletin-board outside the town hall had gathered to read the order that had just been posted
there.
He suddenly realized how little he knew of the French language. He was suddenly
remorseful over wasting so much time playing outdoors rather than studying. The villagers gathered
in the room to show their appreciation for the school that they themselves had neglected as children,
and to honour M. Hamel’s forty years of service as teacher. M Hamel called Franz to recite the rule for
n the first words.
M. Hamel did not scold Franz for not knowing the rule, but he used the occasion to
lecture the class. He told the crowd that each day they had been putting off learning until the next
day. That was the trouble with their home and region of Alsace. M. Hamel continued saying that the
children’s parents had not been anxious to have them learn, sending them out to work instead. M.
Hamel also reproached himself, taking responsibility for those times he had encouraged his students to
procrastinate, because he himself wanted to do other things. M. Hamel then went on to speak about
the beauties of the French language. The teacher then proceeded to the French grammar lesson,
reading from a book to the students. Franz understood everything M. Hamel said with extraordinary

Prose
clarity. He thought that he had never listened so carefully to the teacher before.
The teacher had been there for forty years. Franz thought that M. Hamel must be
heartbroken to leave the school. But Franz was impressed by M. Hamel’s determination to oversee
every lesson until the very last. The clock struck twelve. M. Hamel stood up, pale but tall. He began to
address the class as “My friends,” but was unable to continue. He told them that school was dismissed.

B.P.No. 166
GLOSSARY
UNIT 6

chirping (v) - making a short, sharp high pitched sound (usually by small birds or
insects)
bustle (v) - move in an energetic manner
unison (n) - simultaneous utterance of words
rapping (v) - striking with a series of rapid audible blows
177
ENGLISH

thumbed (v) - a book which has been read often and bearing the marks of frequent
handling

Saar (n) - a river in north eastern France and western Germany


10 cranky (adj.) - strange
Angelus (n) - a Roman Catholic devotion commemorating the Incarnation of Jesus
and including the Hail Mary, said at morning, noon, and sunset.
“Vive la France!” - is an expression used in French to show patriotism. It’s difficult to
translate the term literally into English, but it generally means “Long
live France!”
The Last Lesson

ADDITIONAL
dread – fear thunderclap – shock
resist – oppose wretches – rogues
hurried – rushed recite – narrate
blushed – embarrassed
daring – bold
solemn – serious
reproach – criticism
fright – shock
imprisoned
primer – textbook amazed – surprised
grave – serious pale – whitish
pretend – act might – strength

EXERCISE WITH ANSWER

Choose the appropriate synonym of the word underlined.


Prose

1. I started for school very late that morning and was in great dread of a scolding.
a) fear b) expectation c) thought d) happiness Ans: a)

2. ...... but I had the strength to resist, and hurried off to school.
a) accept b) ask c) listen d) prevent Ans: d)

3. ...... but I had the strength to resist, and hurried off to school.
a) stayed b) rushed c) slowed d) pulled Ans: b)
UNIT 6

4. Usually, when school began, there was a great bustle,


a) calmness b) greatness c) commotion d) movement Ans: c)

5. ...... which could be heard out in the street, the opening and closing of desks, lessons
repeated in unison,
178 a) anger b) irritation c) pain d) harmony Ans: d)
ENGLISH

6. ...... and the teacher’s great ruler rapping on the table.


a) knocking b) touching c) painting d) breaking Ans: a)

7. You can imagine how I blushed and how frightened I was.


a) kicked b) gave c) embarrassed d) embraced Ans: c)
10
8. When I had got a little over my fright, did I see that our teacher had on his beautiful green
coat, his frilled shirt,
a) shoulder b) shock c) happiness d) friend Ans: b)

9. Besides, the whole school seemed so strange and solemn.

The Last Lesson


a) carelessness b) events c) commotion d) serious Ans: d)

10. Hauser had brought an old primer,


a) textbook b) picture c) painting d) newspaper Ans: a)

EXERCISE for SELF EVALUATION


Choose the appropriate synonym of the word underlined.
1. ...... in the same grave and gentle tone which he had used to me.
a) trivial b) exited c) serious d) light Ans:

2.
a) noise b) shock c) pleasure d) insult Ans:

3. ...... you pretend to be Frenchmen, and yet you can neither speak nor write your own
language?’
a) act b) say c) mould d) make Ans:

4. Oh, the wretches; that was what they had put up at the town-hall!
a) friends b) soldiers c) superiors d) rogues Ans:

Prose
5. ...... that I should never see him again, made me forget all about his ruler and how cranky
he was.
a) happy b) sad c) irritable d) friendship Ans:

6. It was my turn to recite.


a) write b) narrate c) pass d) inform Ans:

7. ...... my heart beating, and not daring to look up.


a) bold b) frightened c) happy d) difficult Ans:
UNIT 6

8. We’ve all a great deal to reproach ourselves with.”


a) praise b) allow c) criticism d) invitation Ans:

9. When people are enslaved, as long as they hold fast to their language it is as if they had
the key to their prison.
a) free b) killed c) caught d) imprisoned Ans :
179
ENGLISH

10. I was amazed to see how well I understood it.


a) surprised b) joyous c) instigated d) important Ans :

11. M. Hamel stood up, very pale, in his chair. It was so warm, so bright!
10 a) laughter b) whitish c) filed d) agitated Ans :

12. ...... bearing on with all his might, he wrote as large as he could —* “Vive La France!”
a) weakness b) willingness c) strength d) interest Ans :

ANTONYMS
The Last Lesson

warm × cold quickly × slowly


bright × dark
strange × common
resist × accept
grave × excited
hurried × leisurely
patience × agitated
plenty × scarcity remember × forget
bustle × calmness might × weakness
kindly × cruelly

Choose the appropriate antonym of the word underlined.

1. It was so warm, so bright!


a) hot b) cold c) bright d) grim Ans: (a)

2. It was so warm, so bright!


a) light b) neat c) dark d) happy Ans: (c)
Prose

3. ......had the strength to resist, and hurried off to school.


a) appeal b) control c) reject d) accept Ans: (d)

4. Then, I hurried by as fast as I could go,


a) pushed b) leisurely c) pulled d) rushed Ans: (b)

5. ......you’ll get to your school in plenty of time!”


a) much b) more c) little d) useful Ans: (c)
UNIT 6

EXERCISE for SELF EVALUATION


Choose the appropriate antonym of the word underlined.

1. ......when school began, there was a great bustle,


180 a) calmness b) commotion c) instigation d) information Ans:
ENGLISH

2. M. Hamel saw me and said very kindly,


a) happily b) sadly c) coolly d) cruelly Ans:

3. “Go to your place quickly, little Franz.


a) steadily b) swiftly c) slowly d) hastily Ans:
10
4. Besides, the whole school seemed so strange and solemn.
a) common b) calmness c) known d) high Ans:

5. ......in the same grave and gentle tone which he had used to me,
a) serious b) internal c) external d) excited Ans:
The Last Lesson

6. ......he had never explained everything with so much patience.


a) calmness b) agitated c) empower d) bold Ans:

7. Ah, how well I remember it, that last lesson!


a) forget b) stay c) remind d) remain Ans:

8. ......bearing on with all his might, he wrote as large as he could —* “Vive La France!”
a) strength b) kindness c) cruelty d) weakness Ans:

A.
a. What kind of news was usually put up on the bulletin board?  B.P.No. 163

For the last two years all the bad news had been put up on the bulletin board — the lost battles, the
draft, the orders of the commanding officer.

b. What was the usual scene when school began everyday?


Usually, when school began, there was a great commotion, which could be heard out in the street.
Students would open and close the desks and would repeat lessons in unison, very loud, and the

Prose
teacher would go around with a great ruler knocking on the table.

c. Other than the students, who were present in the class?  B.P.No. 164

Some villagers were present in the class along with the students.

d. Why did M. Hamel say it was the last French lesson?


As per the order from Berlin, German should be taught instead of French in the school. So that was
the last French class.
UNIT 6

e. What was Franz asked to tell? Was he able to answer?


Franz was asked to tell the rules of participles and he was not able to answer the question.

f. Why did M. Hamel blame himself?


M. Hamel blamed himself that he often would send his pupils to water his plants instead of study at
school and he would declare a holiday whenever he wanted to go fishing.
181
ENGLISH

g. What did M. Hamel say about the French language?  B.P.No. 165

M. Hamel said that the French language was the most beautiful, clearest and most logical language of
the world.
10 h. How many years had M. Hamel been in the village?
M. Hamel had been in the village for forty years.
A. Answer the following questions in two or three sentences:  B.P.No. 166

1. Why did Franz dread to go to school that day?


Franz did not study participles and the teacher would ask questions on participles. So he was afraid
The Last Lesson

to go to school.

2. What were the various things that tempted Franz to spend his day outdoors?
The weather was warm and bright and birds were chirping. He saw Prussian soldiers doing their drills
in an open field. These things tempted Franz to spend his day outdoors.

3. Why was the narrator not able to get to his desk without being seen?
Unusually the school was very calm and students were sitting on their seats. So he was unable to get
to his desk without being seen.

4. What was Franz sorry for?

The old villagers were sorry that they had not gone to school more. It was their way of thanking Hamel
for his forty years of faithful service. Also they wanted to show their respect for the country that was
theirs no more.

6. What were the thoughts of the narrator’s parents?


The narrator’s parents thought that it would be better to go to work and earn some extra money than
to learn French.
Prose

7. Why does M. Hamel say that we must guard our language?


He tells the class to guard the language because when people are enslaved, so long as they “hold fast
to their language, it is as if they had the key to their prison.”

8. M. Hamel was gazing at many things. What were they?


M. Hamel sat motionless and looked at everything around him one by one.

9. When and how did M. Hamel bid farewell to the class?


UNIT 6

When the bell struck 12 M. Hamel stood up. He addressed the class as friends and wrote on the board
with chalk piece, “Viva La France!”.. Then he said the class was dismissed.

182
ENGLISH

ADDITIONAL

Answer the following questions in two or three sentences: 10


1. What was pasted in the bulletin board?
The order from Berlin to not continue the French lessons and to teach of German was pasted in the
bulletin board.

2. Why was Franz frightened to enter his class?

The Last Lesson


Franz was frightened to enter the class because he was afraid that M. Hamel, his teacher would ask
him questions on participles which he did not know.

3. Describe the dress worn by M. Hamel.


M. Hamel the teacher was wearing his beautiful green coat, frilled shirt with a little black silk cap having
embroidery on it.

4. Did Franz’s parents help him to learn French? How do you know?
Franz’s parents did not help him to learn French. They wanted him to go to work to earn some extra
money.

B.

ain this with 1.


reference to the French language and M.Hamel.

Prose The Last Lesson


Author Alphonse Daudet
Theme Mother tongue is a key to freedom

If you talk to a man in a language he understands, that goes to his head.

Prose
‘The Last Lesson’ was written by Alphonse Daudet. The story is narrated by a French boy Franz.
He was lazy and liked to play. He disliked studying French. After overpowering the districts of Alsace
and Lorraine in France, Berlin had ordered that German should be taught in schools instead of French.
It was the last French class of the teacher M. Hamel who had been there for 40 years. As a mark of
respect for his hard work the village men also attended his last class. The teacher was full of grief and
nostalgia. They were sad that they did not learn French their mother tongue in their childhood. Franz
was shocked to know it was his last French lesson. But he did not learn French. Suddenly he became
UNIT 6

interested in learning French and he understood what was taught that day. He developed an instant
liking for the teacher M. Hamel and respected him for his sincerity and hard work. He felt sad that
M. Hamel was leaving them and ashamed of not being able to recite the lesson on participles.

I was amazed to see how well I understood.

183
ENGLISH

2. Give an account of the last day of M.Hamel in school.

Prose The Last Lesson


Author Alphonse Daudet
10 Theme Mother tongue is a key to freedom

Language is the blood of the soul into which thoughts run


and out of which they grow

When Franz reached the school he saw a crowd gathered around the bulletin-board. Though he
The Last Lesson

reached the school a little late he was surprised to note that the school was very quiet. Franz took his
seat, and noticed that M. Hamel had clothes on for special occasion. The atmosphere of the classroom
was strange. Franz saw some villagers were sitting in the class. M. Hamel sat on his chair and said that
it would be the last French lesson. He suddenly realized how little he knew of the French language.
He was suddenly remorseful over wasting so much time playing outdoors rather than studying. The
villagers gathered in the room to show their appreciation for the school that they themselves had
neglected as children, and to honour M. Hamel’s forty years of service as teacher. M Hamel called Franz
to recite the rule for the participle, which he had neglected to learn. He stood up to recite but stumbled
on the first words. M. Hamel did not scold Franz for not knowing the rule. He told the crowd that each
day they had been putting off learning until the next day. The teacher then proceeded to the French
e, M. Hamel stood
up then he turned to the black board and with a piece of chalk, he wrote ‘Vive La France’. Finally he
said school was dismissed.

My children this is the last lesson I shall give you.

ADDITIONAL PARAGRAPH
Answer in a paragraph of about 100-150 words.
Prose

1. What does the author want to convey in the lesson ‘The Last Lesson’?

Prose The Last Lesson


Author Alphonse Daudet
Theme Mother tongue is a key to freedom

See that your children are properly educated in the rudiments of their
mother tongue, and then let them proceed to higher branches of learning.
UNIT 6

‘The Last Lesson’ was written by Alphonse Daudet. The story is narrated by a French boy
Franz. He was lazy and liked to play. He disliked studying French. After overpowering the districts of
Alsace and Lorraine in France, Berlin had ordered that German should be taught in schools instead of
French. When Franz reached the school he saw a crowd gathered around the bulletin-board. Though
he reached the school a little late he was surprised to note that the school was very quiet. An order
184
had arrived from Berlin that from the next day onward, only German would be taught in the schools
ENGLISH

of Alsace-Lorraine. These words came as a shock to little Franz, who realized that the villagers he had
seen by the bulletin-board outside the town hall had gathered to read the order that had just been
posted there. M. Hamel said that it would be the last French lesson. He told the crowd that each day
they had been putting off learning until the next day. That was the trouble with their home and region
of Alsace. M. Hamel continued saying that the children’s parents had not been anxious to have them
10
learn, sending them out to work instead. M. Hamel also reproached himself, taking responsibility for
those times he had encouraged his students to procrastinate, because he himself wanted to do other
things. M. Hamel then went on to speak about the beauties of the French language. He emphasized
the importance of mother tongue. The message is that when a people are enslaved, as long as they
hold fast to their language, it is as if they had the key to their prison.

The Last Lesson


Then he turned to the blackboard, took a piece of chalk, and bearing on
with all his might, he wrote as large as he could – “Vive La France”!

B.P.No.167
VOCABULARY

C. In column A are some of the idiomatic phrases from the essay. Match them with
equivalent single words in column B:

A B Answer

blow up submit explode


show up explode reveal
call for succeed visit
break off finish finish
knuckle under visit submit

D. Frame sentences of your own using the above idiomatic phrases.

Prose
go for – Ten people tried to go for a single job.
blow up – The terrorist tried to blow up the bridge.
show up – The writing did not show up the idea of the writer.
call on – We need to call on our prinicpal.
break off – The partners broke off their partnership in the business due to some
misunderstanding.
knuckle under – They have to work hard to knuckle under their project in time.
UNIT 6

E. Given below are some idiomatic phrases. Find the meaning of it using the dictionary.

A B
put on walk away
come in time out
try again try again
185
ENGLISH

put on – wearing something


walk away – to leave the place or situation
come in – enter, arrive
10 time out – stop what we usually do

B.P.No.167
LISTENING

F. Listen to the article titled “Remembering Nel Jayaraman”


The Last Lesson

In pairs, present an interview. One student will be the interviewer and the other would be Nel Jayaraman
himself. Two sets of conversations have been given as examples for your help.

Student A : (interviewer) - Vanakkam sir. For what cause do you organise festivals?
Student B : ( NJ) - I organise these festivals with a difference. I present seeds to all the participating
farmers.
Student A : (interviewer) - Oh ! That’s really good, Sir. What do you expect in return ?
Student B : (NJ) - In return I expect them to have double the harvest next year.
Student A : (interviewer) - Where did you organise the NEL festival ?
Student B : (NJ) - Thiruvarur.
he meeting ?
Student B : (NJ) - From 500 it grew to 5000.
Student A : (interviewer) - What did you distribute to the farmers?
Student B : (NJ) - I distributed traditional paddy seeds.
Student A : (interviewer) - How did you commute to each these villages?
Student B : (NJ) - I commuted by cycle or bus.
Student A : (interviewer) - Ayya, Do you plan your schedules?
Prose

Student B : (NJ) - Do you duty and don’t wait for the fruits.
Student A : (interviewer) - How could you remain so cool and calm sir?
Student B : (NJ) - I have an alternative.
Student A : (interviewer) - Where was your heart and soul?
Student B : (NJ) - My heart and soul was in agriculture field.
Student A : (interviewer) - People say when your popularity grew, you spent less time in the field ?
UNIT 6

Student B : (NJ) - That’s wrong. I spent all the time in the field.
Student A : (interviewer) - What is your message to the world?
Student B : (NJ) - If we work selflessly for the society it would come back.
Student A : (interviewer) - Thank you, Sir. Nandri.

186 Student B : (NJ) - Nandri.


ENGLISH

B.P.No.168
SPEAKING

G. A road map is given below. Answer the questions that follow with the help of the road 10
map. Work in pairs and discuss to give directions to get to one place from another.

The Last Lesson


1. You are at the market. You need directions to go the pharmacy.
Walk through George Street and come to the bakery. Walk still further and you will come to South
street. Turn to your right and walk straight. You can see the pharmacy.

2.
Walk through the park street and you will come to George Street. Turn right and walk straight. You will
reach the Art Gallery.

3. Give your partner the directions to go from the Bank to the hotel.
Walk through the park street and you will come to George Street. Turn right and walk straight. You will
come to the bakery. You will find the hotel in front of the bakery.

4. Direct your partner from the post office to the market.

Prose
From post office turn left and come to North street; walk straight, you will come to George Street. Turn
left and walk for some time. You will see to the market.

5. Your partner wants to go the library from school. Give suitable directions.
Walk through the park sheet and you will reach George street. Turn right and walk a few yards. You
will be able to see the library on left.
UNIT 6

187
ENGLISH

B.P.No.169
READING

10 H. Read the poem carefully and answer the questions that follow:

Festivals
Festival of harvest Festival of decorated cars
Celebrations at its best That twinkle like the stars
Festival of Light Festival of Love
To our heart’s delight That spreads treasures on a tree,
The Last Lesson

Festival of Dance To share the word from above


Leaves us in a trance That makes us happy and free.
Festival of Music Festival of sacrifice
Where they sing the joyous lyric To unfurl the joy of giving,
Festival of flowers Celebrate them well and nice
That brightens up with colours To make life worth living.

1. Fill in the blanks.


(a) Festival of light is the festival which fills our hearts with delight.
(b) Joy of giving is referred to as a festival of sacrifice.

Joy of giving is unfurled during the festival of sacrifice.

3. How can we make our life worth living?


By celebrating the festivals we make our life worth living.

4. What does the poet mean by ‘Festival of flowers’?


By Festival of flowers the poet means Navarathiri.

5. When are we in a state of trance?


During the festival of dance, we are in a state of trance.
Prose

6. What do the people do when the festival of Music is celebrated?


When the festival of Music is celebrated people sing joyous lyrics.

7. What makes us happy and free, according to the poet?


Festival of love makes us happy and free according to the poet.

8. Find out the rhyme scheme employed in the fourth stanza.


UNIT 6

abab

9. Pick out the rhyming words from the first stanza of the poem.
harvest - best; light - delight

10. Write down the words that alliterate in the poetic lines below.
(a) Festival of Flowers Festival - Flowers
188
(b) That spreads treasures on a tree treasures - tree
ENGLISH

B.P.No.170
WRITING

Posters 10
What is a poster?
Posters are placards displayed in a public place announcing or advertising something. Posters are
notices, advertisements and invitations – all in one.
What is the purpose of a Poster?

The Last Lesson


It is to create social awareness about current problems and needs, or to advertise or invite and display
something.

I. Create posters for the following.


1. You are Raja/ Ranjani. Draft a poster to create awareness about the harmful effects of
using plastics, in not more than 50 words

Plastic easy to use;


stays for long on earth;
spoils everything;

Should we use it?

2. Say ‘No to Drugs’ – Design a poster for it in not more than 50 words. You may use slogans/
phrases.

Drugs kill;
bring easy death;
invitation to death;

Prose
we are born to live healthy.
Say no to drugs.
3. “ Save our Earth” is the need of the hour. Draft a poster with attractive slogans/ phrases
for the same in not more than 50 words. Use attractive drawings.
UNIT 6

Our Earth is fragile; it needs our help to sustain itself


Let us join together to save our earth for our coming generation 189
ENGLISH

4. You are Sita/ Sudhan. Design a poster in not more than 50 words to focus on not wasting
water. Be creative.

10 Water is the elixir of life;


it is getting out of our planet;
wake up for saving water;
Save water; don’t grieve later.

5. Good handwriting is the index of an individual. Design a poster on the importance of good
The Last Lesson

handwriting. Use catchy slogans or phrases. Your poster should not exceed 50 words.

Handwriting decides your future;


it shows your inner self;
improve your handwriting;
follow the hints.

J. Draft Letters for the following:


1. You are Ajeet, living in a remote village in Tirunelveli. You participated in a health camp
organised by your school. You were surprised to observe that most of the residents were
unaware of health and hygiene. As a concerned citizen, write a letter to the editor stating
the need to organise such camps focusing on the importance of health and hygiene.
18, Appar Street,
K.N. Colony,
Salem - 636 014.

October 10. 2020

The Editor,
Prose

The Hindu,
ABC Street,
Salem - 4.

Sir,
Subject: The need to organise health camps.
I attended a health camp organised by my school in a remote village in Tirunelveli. It was a
successful camp but I was astonished to know the poor knowledge of the villages regarding health and
UNIT 6

hygiene. We have many NGOs who are interested to do something for the welfare of the people. They
could conduct health and hygiene camp in small villages to make the people aware of the importance
of health and hygiene. This will be a good service to the ignorant villagers.
I humbly request you to publish this in your esteemed daily so that some NGOs may take up this
task.
190 Thanking you,
ENGLISH

Yours faithfully,
Ajeet.
2. You are Sanjay. Your colony utilises solar energy to light the common areas. You find
many friends of your colony forgetting to switch off the lights in the common area. As a 10
responsible citizen, write a letter to a newspaper, echoing the importance to conserve and
preserve solar energy.
12, Gandhi Road,
Erode.

October 22, 2020

The Last Lesson


The Editor,
The Hindu,
ABC Street,
Erode.
Sir,
Subject: Awareness to preserve solar energy.
I am very proud to say that our colony MGR Nagar is using solar energy to electrify the common
areas. In this way we have become a model for many colonies. Unless we get the full support of all the
people we may fail in conserving the solar energy. In the recent past it has been found out that many
wasted. I request
the people to take the responsibility to conserve the solar energy by switching off the lights when they
are not needed.
I humbly request you to publish this in your esteemed daily so that the people of that area will
take up the responsibility of preserving solar energy.

Thank you,

Yours faithfully,
Sanjay.

Prose
3. You are Sadasivam. You recently visited your native town in Vellore. You happened to
accompany your grandmother to your family temple. You were shocked to notice the poor
condition and maintenance of the temple. Write a letter to the Editor of local newspaper
highlighting the poor condition of the temple. Also give some suggestions and request the
HRC to take steps to improve the situation.
45, Mark Street,
Sitharipet,
UNIT 6

Chennai.

October 25, 2020

The Editor,
The Hindu,
Vellore.
191
ENGLISH

Sir,
Subject: Maintenance of Hindu temples in Vellore.
I recently visited my family temple in Vellore. I was really shocked to find the poor condition and
10 maintenance of the temple. Water is everywhere and it is difficult to walk to the entrance of the temple.
The wastes are not kept in the proper place. It is in such a bad condition that people may stop going
to the temple. I request the authorities to take steps to clean up the place to avoid logging of water.
I humbly request you to publish this in your esteemed daily so that some positive steps may be
taken by the authorities.

Thank you,
The Last Lesson

Yours faithfully,
Sadasivam.

4. You are Sudha. Your neighbour has a pet dog that barks continuously. Write a letter to
the Editor of a weekly newspaper of your locality, highlighting the nuisance and noise
pollution created thus. Also suggest ways to solve the problem.
22, Court Road,
Nagercoil - 1.

The Editor,
The Hindu,
Nagercoil.
Sir,
Subject: Nuisance created by pet dogs.
I am writing this letter to highlight the nuisance caused by pet dogs.
My neighbour has a pet dog and it barks all the time. Of course he has the right to keep his pet
but I have my right to live in peace. In such a situation it is very difficult for me to stay here peacefully.
Prose

I informed my neighbour many times hut he has not taken any step to avoid this nuisance. It amounts
to noise pollution. The owners of dogs should feel this and try to do something to avoid this. They
can find out from a vet why they bark like this and ask them some suggestions.
I humbly request you to publish this in your esteemed daily so that such people will take some
steps to help the neighbours.

Thanking you,
UNIT 6

Yours faithfully,
Sudha.

5. You are Raja. The street lights of your area do not work properly. As a responsible citizen,
write a letter to the newspaper enlightening them about the problem and also suggest
ways to brighten the area.
192
ENGLISH

46, Rajaji Nager,


Polayankottai,
Tirunelveli.

November 12, 2020 10


The Editor,
The Hindu,
Tirunelveli - 1.

Sir,

The Last Lesson


Subject: Problems of non-working street lights.
I live in Rajai Nagar. Our area street lights do not work properly. So it is difficult to walk in the
night. Lamp posts are kept properly but the lights are not burning. In most of the cases the bulbs are
fused. In some cases the wires are not properly laid. So the authorities are requested to take some
positive steps to brighten our area..
I humbly request you to publish this in your esteemed daily so that the authorities will take necessary
steps.

Thank you,
Yours faithfully,

B.P.No. 174
GRAMMAR

Subject – Verb Agreement


• The subject and verb of a sentence should be in agreement with each other.
• A verb agrees with the subject in number and person. A singular subject takes a singular verb
and a plural subject takes a plural verb.

Prose
A. Fill in the blanks appropriately.
1. Mahatma Gandhi is the father of our nation.
2. There are ten dogs in my street.
3. They have to write the exercises neatly.
4. Butter milk is good for health.
5. Fruits are good for health.

B. Fill in the blanks with the appropriate verb:


UNIT 6

1. The quality of dal is not good. 7. Gullivers Travels is an excellent story.


2. The horse carriage is at the door. 8. Neither food nor water is found here.
3. My friend and teacher has come. 9. Mathematics is a branch of study.
4. Are your father and mother at home? 10. Fifteen minutes is allowed to read the
5. Honour and glory is his reward. question paper.
6. The ship with its crew is sailing good. 193
ENGLISH

C. Change the singular nouns to plurals by either adding ‘s’, ‘ies’, ‘es’, ‘ves’.

Singular Plural
1. leaf leaves
10 2. lorry lorries
3. bat bats
4. clock clocks
5. table tables
6. lamp lamps
Foreign

7. doll dolls
AreLesson

8. biscuit biscuits
9. knife knives
10. loaf loaves

Non Finites:
Last

D. Identify the non-finites in the following sentences and underline them.


Men

E.g: Children love eating chocolates.


NoThe

1. Roshan dreams of becoming an architect.


2. We must aim at fulfilling Dr APJ Abdul Kalam’s dream to make India the most developed country

3. Taking the children to the museum is Seema’s responsibility.


4. Having finished the work, the manager decided to return home.
5. Travelling with her family, Tara enjoyed every minute of it.

F. Fill in the blank with the correct alternative:


1. Having played on the flute, Krishna returned it. (played / having played)
2. We wish she continues to be healthy. (being / be)
3. The doctor advised him against wandering in the sun. (wander / wandering)
Prose
Poem

4. I like drinking rasam. (drinking / drink)


5. Having used the scissors I returned it to her. (using / having used)

F. Tick the correct sentences:

A B

1 I had desired to eat a cake.  I had desired to have eaten cake.


UNIT 6

2 My son is fond of music.  My son is fond to music.

3 Sreena avoids eating fruits.  Sreena avoids to eat fruits.

4 Bravery is not to pick a quarrel. Bravery is not picking a quarrel. 


5 It is easier to say than do. It is easier said than done. 
194
ENGLISH

UNIT

6
POEM
10
No Men Are Foreign
- James Falconer Kirkup

Foreign
SUMMARY

Foreign
AreLesson
The poem ‘No Men are Foreign’ by James Falconer Kirkup points out why it is wrong
to hate others based on differences such as race, culture, or geography. The speaker goes through
points to show how all people are similar and part of the brotherhood of man. In the end, the speaker
mentions how unnatural war is because it is fighting against ourselves.

Are
Last
This poem begins and ends with the same line, with the repetition serving to emphasize
the fact that this is the core message of the poem:

Men
Men
“Remember, no men are strange, no countries foreign”.

NoThe
The subject of this poem is the unity of the human race, despite differences in race,
geography or language. All people are brothers, in that we all walk on the same land and will be buried

No
under it. We are all, ultimately, related, and are all born the same and will die in the same way. Use
uld do well to
remember that under the uniforms, a single body breathes.
The poem covers various points of similarity between people from all countries: people have
hands like ours, they labour as we do, and they have eyes like ours that wake to see a similar world.
Hating other people because they are different, or raising arms against other people, is effectively a
condemnation of ourselves: it is the human earth, our own earth that we defile.
This poem could be understood as a protest against such issues as racial discrimination
and warfare, which threaten the peace and safety of human beings.

Prose
B.P.No. 180

Poem
GLOSSARY
labour - hardwork defile - damage the purity or
betray - disloyal appearance
Condemn - express complete disapproval outrage - offend / scandalize
Based on the understanding of the poem, read the following lines and answer the questions
given below.  B.P.No. 180

1. Beneath all uniforms, a single body breathes


UNIT 6

Like ours: the land our brothers walk upon


Is earth like this, in which we all shall lie.
a) What is found beneath all uniforms?
Beneath all the uniforms we could find same kind of body which breathes.
b) What is same for every one of us?
195
Land or earth is same for every one of us.
ENGLISH

c) Where are we all going to lie finally?


We are all going to lie in the earth finally.

2. They, too, aware of sun and air and water,


10 Are fed by peaceful harvests, by war’s long winter starv’d.
a) What is common for all of us?
The sun, air and water are common for all of us.

b) How are we fed?


No Men Are Foreign

We are fed with the produce of the harvests

c) Mention the season referred here


Winter season is referred to here.

3. Their hands are ours, and in their lines we read


A labour not different from our own.
a) Who does ‘their’ refer to?
‘Their’ refers to enemy soldiers.

b) What does the poet mean by ‘lines we read’?


‘Lines we read’ means their way of life.

The work the enemy soldiers do is not different from our work.

4. Let us remember, whenever we are told


To hate our brothers, it is ourselves
That we shall dispossess, betray, condemn.
a) Who tells us to hate our brothers?
The king or the man on the top tells us to hate our brothers.

b) What happens when we hate our brothers?


Poem

When we hate our brothers we hate ourselves.

c) What do we do to ourselves?
When we hate our brothers we betray and condemn ourselves.

5. Our hells of fire and dust outrage the innocence


Of air that is everywhere our own,
Remember, no men are foreign, and no countries strange.
a) What outrages the innocence?
UNIT 6

The fire and dust which come out due to war outrage the innocence.

b) Who are not foreign?


Men are not foreign to us.

c) What is not strange?


196 Countries are not strange.
ENGLISH

ADDITIONAL
Read the following lines and answer the questions given below. 10
1. Beneath all uniforms, a single body breathes,
Like ours: the land our brothers walk upon.
a) Identify the words in alliteration.
Beneath, body, breathes.
No Men Are Foreign

b) Identify the figure of speech employed here.


Simile.

2. Remember they have eyes like ours that wake


Or sleep and strength that can be won.
a) Who does the word ‘they’ refer to?
‘They’ refers to people of different countries.

b) How can we win the people?


We can win the people by love and kindness.

3. Our hells of fire and dust outrage the innocence


a)
Metaphor.

4. Their hands are ours, and in their lines we read.


a) Identify the figure of speech used in this line.
Metaphor.

b) Whose hands are referred to here?


Other people’s (neighbours) hands are referred to here.

Based on your understanding of the poem complete the following by choosing the

Poem
appropriate words/phrases given in brackets:
This poem is about the dreams and aspirations of all men. The subject of the poem is the unity
of human race, despite the difference in colour, caste, creed , religion , country etc. All human beings
are same. We walk on the same land and we will be buried under it. Each and everyone of us are
related to the other. We all are born same and die in the same way. We may wear different uniforms
like’ brotherhood,’ during wars the opposing side will also have the same breathing body like ours. We
as human do the same labour with our hands and look at the world with the same eyes. Waging war
against others as they belong to a different country is like attacking our own selves. It is the human
UNIT 6

earth we impair. We all share the same language. We are similar to each other. So the poet concludes
that we shouldn’t have wars as it is unnatural to fight against us.
unity of human, dreams and aspirations, same land, our hands, unnatural,
breathing body, same eyes, brotherhood, language, human earth

Based on your understanding of the poem answer the following questions in a


197
paragraph of about 100-150 words.
ENGLISH

1. What is the central theme of the poem ‘No Men Are Foreign’?

Poem No Men Are Foreign


Poet James Falconer Kirkup
10 Theme Unity of human race despite diverse differences

Remember, no men are foreign, no countries foreign


Beneath all uniforms, a single body breathes

James Falconer Kirkup (1918 – 2009) was an English poet, translator and travel wirter. He
No Men Are Foreign

wrote over 30 books including autobiographies, novels and plays.


The central theme of the poem is that war is a mistaken notion and all people on earth
are connected through their common humanity. The poet describes a number of similarities that we
share with our supposed enemies. The poet notes that underneath every soldier’s uniform, the soldier
breathes as we do. The soldiers who oppose us all walk on the earth, as we do, and they also share
the sun, drink water, and enjoy the harvests of the earth. In addition, we share similar body parts,
such as hands that labour and eyes that wake from sleep. After drawing all these similarities, the poet
asks the reader to recall that when we are asked to hate others, it is really ourselves that we hate. If
we fight each other, we destroy the earth that we share and destroy innocence everywhere. When we
fight others, we forget that our enemies are not foreign but are like ourselves. So the poet concludes
that we shouldn’t have wars because it is quite unnatural to fight against us.
Their hands are ours, and in their lines we read

2. The poem ‘No Men Are Foreign’ has a greater relevance in today’s world. Elucidate.
Poem No Men Are Foreign
Poet James Falconer Kirkup
Theme Unity of human race despite diverse differences

In every land is common life


That all can recognise and understand
Poem

James Falconer Kirkup (1918 – 2009) was an English poet, translator and travel wirter. He
wrote over 30 books including autobiographies, novels and plays.

The poem ‘No Men are Foreign’ by James Falconer Kirkup points out why it is wrong to
hate others based on differences such as race, culture, or geography. The speaker stresses that all
people are similar and part of the brotherhood of man. At the end of the poem, the speaker mentions
how unnatural warfare is because it is fighting against ourselves. The poem covers various points
of similarity between people from all countries: people have hands like ours, they labour as we do,
and they have eyes like ours that wake to see a similar world. Hating other people because they are
UNIT 6

different, or raising arms against other people, is a condemnation of ourselves: it is the human earth,
our own earth that we defile. In today’s world each country wants to fight with the other for the sake
of its supremacy. They hate each other to maintain their economy and social status. Even men hate
each other for silly reasons forgetting that we are all brothers and sisters. So this poem is very relevant
in today’s world.
Remember, we who take arms against each other
198
It is the human earth that we defile.
ENGLISH

UNIT

6
SUPPLEMENTARY
10
The Little Hero of Holland
– Mary Mapes Dodge

Holland
ofHolland
No Men Are Foreign

SUMMARY
‘The Little Hero of Holland’ was written by Mary Maps Dodge. She describes how a boy of eight

Heroof
saved Holland from drowning.
Much part of Holland lies below sea level. Great walls called dykes keep the sea running into the

LittleHero
land. People know that they have to take care of the dykes to live safely. Many years ago a boy called
Peter lived in Holland. His father was the gate keeper of the dykes. He opened and closed the gates
to allow the ships to pass.

TheLittle
When Peter was eight years old his mother sent him to his blind friend to give him some cake.
She wanted him to come back before sun set. He walked along the dykes and spend some time with
the blind friend. Then he returned. As he walked along the canal he noticed the rain had swollen the

The
water level. He was happy that the walls were very strong. Suddenly he noticed the sun was setting
and he remembered his mother’s words. So he started running towards home.
Just then he heard the noise of tickling water. When he looked down he saw a small hole in the
dyke and through the hole, a small stream was flowing. He understood the danger. The hole might
become big and water would rush through that and destroy the town. He climbed down the side of the
dyke and thrust his finger into the small hole. The flowing of water stopped.

Supplementary
It was good for sometimes. Then it became dark and nobody was around to help him. He
decided to say there to save Holland. His mother thought that Peter must have stayed with the blind
friend and he would come only in the morning. She was angry and decided to shout at him when he

Poem
came.
Early the next morning a man who was going to work heard a groan. When he looked down he
saw a boy clinging to the wall. The boy answered that he was keeping the water under control. The
news went round the town and people came with shovels to close the hole. The boy was carried home.
Then the whole town came to know that Peter saved the whole country. Still people remember the
bravery of Peter.
UNIT 6

B.P.No. 185
GLOSSARY
dike (n) - an embankment for controlling or holding back the waters of the sea or
a river.
sluices (n) - a sliding gate or other device for controlling the flow of water, especially
one in a lock gate. 199
ENGLISH

trickling (v) - flowing in a small stream (a liquid)


numb (adj.) - deprived of the power of sensation.
chattered (v) - feeling cold and frightened that one can’t stop the upper teeth from
10 against one’s lower teeth.
crouching (v) - adopting a position where the knees are bent and the upper body is
brought forward and down.
The Little Hero of Holland

groan (v) - make a deep inarticulate sound conveying pain


shovels (n) - a tool resembling a spade with a broad blade and typically upturned
side, used for moving earth, coal, snow etc.

A. Based on the understanding of the story, complete the Graphic Organiser suitably.

Title: The Little Hero of Holland


Plot: Saving of Holland from drowning
Setting: A boy walking along the dike
Theme: Save the people from danger
Characters: Peter, his father, his mother, the blind friend, a man, dyke and sluice
Climax: The little boy stopping the water was seen by a man
Values highlighted Responsibility and commitment for the cause of society.
in the story:

B. Based on your understanding of the story answer the following questions in one or
two sentences:

1. What are the little children of Holland, aware of?


The little children of Holland are aware of the importance of dikes. They know that it must be watched
Supplementary

carefully every moment.

2. What was the work assigned to Peter’s father?


Peter’s father was assigned to tender the gates of dike. He had to open and close to allow the ships to
pass out of Holland’s canals.

3. Why did Peter’s mother call him?


Peter’s mother wanted to send some cakes to a blind friend of Peter. So she called him.

4. How did Peter spend his time with his blind friend?
Peter spent his time with his blind friend by telling him about his walk along the dike and about the sun
UNIT 6

and the flowers and the ships far out at sea.

5. Why did the father always say ‘angry waters’?


The waters were angry because the father keeps them always under control.

6. What did Peter see when he stopped near the dikes?


200 Peter saw a small hole in the dike. Through the hole, water was flowing.
ENGLISH

7. What were the thoughts of the mother when Peter didn’t return home?
The mother thought that Peter must have stayed with the blind friend and he would come next day
morning.

8. How did Peter spend his night at the dikes?


10
Peter thrust his little finger into the hole in the dike and spent the whole night there. He called for
others’ help but nobody came to him.

The Little Hero of Holland


9. Who found Peter in the dikes and what did he do?
Next day morning a man was going to his work. He found Peter in the dike and he spread the news
to the town.

10. How did the villagers mend the hole?


The villagers came with the shovels and mended the hole.

C. Based on your understanding of the story answer the following question in about 100-
150 words.

1. Narrate in your own words the circumstances that led Peter to be a brave little hero.
‘The Little Hero of Holland written by Mary Maps Dodge – boy of eight saves Holland –
below sea level – dyke keeps the sea water away – goes to his blind friend – returning
– sees the hole in dyke – water was flowing – may become big – climbs down – keeps
man hears the
groaning – finds a boy – the news reaches others – the boy carried home

Title The Little Hero of Holland


Author Mary Mapes Dodge
Theme A boy’s courage resolve to save the people

‘The Little Hero of Holland’ was written by Mary Maps Dodge. She describes how a boy of

Supplementary
eight saved Holland from drowning. Much part of Holland lay below sea level. Great wall called dike
kept the sea running into the land. When Peter was eight years old his mother sent him to his blind
friend to give him some cake. When he returned he walked along the canal and noticed the rain had
swollen the water level. Just then he heard the noise of tickling water. When he looked down he saw
a small hole in the dike and through the hole, a small stream was flowing. He understood the danger.
The hole might become big and water would rush through that and destroy the town. He climbed down
the side of the dike and thrust his finger into the small hole. The flowing of water stopped. It was
good for sometime. Then it became dark and nobody was around to help him. He decided to stay
there to save Holland. Early the next morning a man who was going to work heard a groan. When
he looked down he saw a boy clinging to the wall. The boy answered that he was keeping the water
UNIT 6

under control. The news went round the town and people came with shovels to close the hole. The boy
was carried home. Thus he became a brave hero.

D. Identify the character/speaker.


1. “I want you to go across the dike and take these
cakes to your friend, the blind man.” Ans: Peter’s mother 201
ENGLISH

2. “I am glad they are so strong”. Ans: Peter


3. “Holland shall not be drowned while I am here.” Ans: Peter
4. “What’s the matter?” he called. “Are you hurt?” Ans : the man going to work in the morning

10 5. “Tell them to come quickly!” Ans: Peter

ADDITIONAL
The Little Hero of Holland

1. The little boy was glad to go on such an errand. Ans: Peter


2. “I’ll stand it somehow.” Ans: Peter
3. “Come Peter, I want you to go across the dike.” Ans: Peter’s mother
4. “I can keep them back with my finger.” Ans: Peter
5. He saw a child clinging to the side of the great wall. Ans: the man going to work in the morning
6. Father always calls them the angry waters Ans: Peter’s father / Peter
7. Mother will be watching for me Ans: Peter / Peter’s mother
8. I’m keeping the water back Ans: Peter

ADDITIONAL EXERCISES

I. Rearrange the sentences in coherent order.

I 1. Peter’s father was the keeper of the dikes.


2. Great walls called dikes keep the sea running into the land.
3. He opened the gates to allow the ships to pass.
4. People know that they have to take care of dikes.
5. Much part of Holland lies below sea level. Answer: 5, 2, 4, 1, 3

II 1. As the sun was setting he started running towards home.


Supplementary

2. On his way back he walked along the canal.


3. His mother sent him to his blind friend to give him some cakes.
4. He noticed the rain had swollen water level.
5. Peter was eight years old.  Answer: 5, 3, 2, 4, 1

III 1. Immediately he kept his finger and stopped the water flowing.
2. Peter was coming back home.
3. He heard the tickling of water.
4. He saw a hole in the dyke.
5. He noticed the rain had swollen water level. Answer: 2, 5, 3, 4, 1
UNIT 6

II. Reading comprehension.


Read the following extract and answer the questions given below.

1.  One afternoon in the early fall, when Peter was eight years old, his mother called him from
his play. “Come, Peter,” she said. “I want you to go across the dike and take these cakes to your friend,
202 the blind man. If you go quickly, and do not stop to play, you will be home again before dark.”
ENGLISH

 The little boy was glad to go on such an errand, and started off with a light heart. He stayed
with the poor blind man a little while to tell him about his walk along the dike and about the sun and
the flowers and the ships far out at sea. Then he remembered his mother’s wish that he should return


before dark and, bidding his friend goodbye, he set out for home.
Questions :
10
1. What did Peter’s mother want him to do? 4. What did the little boy tell his friend?
2. What did Peter’s mother advise him to do? 5. What did the little boy remember?
3. Why was the little boy glad?

The Little Hero of Holland


Answers :
1. Peter’s mother wanted him to walk across the dike and take the cakes to his friend.
2. Peter’s mother advised him to go quickly and return home before dark.
3. The little boy was glad to go out, meet his friend and give him cakes.
4. The little boy told his friend about his walk along the dike and about the sun and the flowers and
the ships far out at sea.
5. The little boy remembered his mother’s wish that he should return before dark.

III. Passage for comprehension.


1.  Peter understood the danger at once. If the water ran through a little hole it would soon make a
larger one, and the whole country would be flooded. In a moment he saw what he must do. Throwing
away his flowers, he climbed down the side of the dike and thrust his finger into the tiny hole.
Questions :
?
2. What would happen if the water ran through 4. What did he throw away?
a little hole? 5. What did he do to prevent the flood?
Answers:
1. Peter understood that the whole country would be flooded.
2. It would soon make a larger one.
3. He must do something to stop the flow of water through a hole.

Supplementary
4. He threw away the flowers.
5. He thrust his finger into the tiny hole.
2.  But his mother had looked anxiously along the dike road many times since sunset for her little
boy, and now she had closed and locked the cottage door, thinking that Peter was spending the night
with his blind friend, and that she would scold him in the morning for staying away from home without
permission. Peter tried to whistle, but his teeth chattered with the cold. He thought of his brother and
sister in their warm beds, and of his dear father and mother. “I must not let them be drowned,” he
thought. “I must stay here until someone comes, if I have to stay all night.”
Questions :
1. Why did the mother look anxious? 3. Why was she going to scold his son?
2. What reason did she give her self for his 4. What did the little boy think of?
UNIT 6

son not returning home before dark? 5. How long did he want to stay there?
Answers:
1. The mother looked anxious as her son had not yet come back home.
2. She thought that Peter was spending the night with his blind friend.
3. She was going to scold him for staying away from home without permission.
4. The little boy thought of his brother and sister and his parents.
5. He wanted to stay there until some came to help him. 203
ENGLISH

UNIT

7
PROSE
10
The Dying Detective
- Arthur Conan Doyle
The Dying Detective

B.P.No. 188
WARM UP

Solve the crossword using the list of words and the clues.

Investigations
I
• infer
I N Q U I R Y
• observe
M Q
• examine
E U D
N E
• revealed
I N C O N C E I V A B L
T • inconceivable

A N A L Y S E I E • aspects
L X T C • link
A I T • detective
R M V I • inquisitive
I N V E S T I G A T E I V • inspect
L I N K V N S N E
• conclude
N O B S E R V E P F
• inquiry
S A D E D U C E
• analyze
Prose

P L C R
• mental
E E T
• deduce
C O N C L U D E S
• investigate
T


Across Down
2. a question 1. curious; wants to understand things
UNIT 7

5. unimaginable 3. related to the mind


6. to examine all the parts of something in order 4. a
 person whose job is to find or recognize the hidden
to understand it information needed to solve a crime
9. to look into a situation (often a crime, but it 7. to look closely at something
can also be a mystery 8. shown or made known
12. a connection; one part of a chain 10. different sides or ways of looking at something
204
ENGLISH

Across Down
14. to notice or watch 10. different sides or ways of looking at something
15. to figure out something unknown by 11. to make a logical guess that something is true based
considering all its known aspects and on the evidence, although the evidence is not clear 10
reasoning it through enough to be absolutely certain
16. to consider the evidence and then decide 13. to look at something carefully to find problems or
what is true or correct (OR to end something) specific information

Answers
The Dying Detective

Across
2. inquiry, 5. inconceivable, 6. analyse, 9. investigate, 12. link, 14. observe, 15. deduce, 16. conclude
Down
1. inquisitive, 2. mental, 4. detective, 7. examine, 8. revealed, 10. aspects, 11. infer, 13. inspect

SUMMARY

Introduction

Mrs. Hudson was the land lady of Holmes. She met his assistant Watson and told him about
Holmes, who was apparently dying of a rare tropical disease, Tapanuli fever, contracted while he was
on a case. Watson was shocked, not having heard about his friend’s illness. Mrs. Hudson said that
Holmes had neither eaten nor drunk anything for three days.

Holmes instructed Watson not to come near him, because the illness was highly infectious.
Although Watson wished to examine Holmes himself or send for a specialist, Holmes demanded that
Watson should wait several hours before seeking help. So, Watson was forced to wait.

While Watson waited, he examined several objects in Holmes’s room. Holmes grew angry when
Watson touched the items explaining that he did not like his things touched. At six o’clock, Holmes
told Watson to turn the gaslight on, but only half-full. He then instructed Watson to bring Mr Culverton
Smith of 13 Lower Burke Street, but he should make sure that Watson would return to Baker Street

Prose
before Smith arrived.

Watson went to Smith’s address. Although Smith refused to see anyone, Watson forced his
way in. Once Watson explained his errand on behalf of Sherlock Holmes, Smith’s attitude changed
drastically. Smith agreed to come to Baker Street within half an hour.

Believing that they were alone, Smith was frank with Holmes. It soon emerged, to Watson’s
horror, that Holmes had been sickened by the same illness that killed Smith’s nephew Victor. Smith
then saw the little ivory box, which he had sent to Holmes by post, and which contained a sharp spring
UNIT 7

infected with the illness. Smith removed it to avoid the evidence of his crime. He then resolved to stay
there and watch Holmes die.

Holmes asked Smith to turn the gas up full, which Smith did. The full gaslight was the signal to
Morton to move in. Holmes told Morton to arrest Culverton Smith for the murder of his nephew, and
perhaps also for the attempted murder of Sherlock Holmes. Smith, still as arrogant as ever, pointed out
that his word was as good as Holmes’s in court, but Holmes then called for Watson to emerge from
205
behind the screen, to present himself as another witness to the conversation.
ENGLISH

Holmes explained that his illness was feigned to induce Smith to confess to his nephew’s murder.
Holmes was not infected by the little box; he had enough enemies to know that he must always
examine his mail carefully before he opened it. Starving himself for three days he appeared as if he

10 had fallen sick. Thus, he revealed the fact that Mr. Smith killed his nephew. He wanted to kill Holmes
the same way to avoid imprisonment.

B.P.No. 194
GLOSSARY
gaunt (adj.) - lean, especially because of suffering, hunger or age.
The Dying Detective

twitched (v) - give short, sudden jerking movements.


contagious (adj.) - spreading of a disease from one person to another by direct contact
groan (v) - a deep inarticulate sound conveying pain or despair.

plague (n) - a contagious bacterial disease characterized by fever .


bolted (v) - closed the door with a bar that slides into a socket.
mantle piece (n) - a structure of wood or marble above or around the fireplace.
half-crown (n) - a former British coin equal to two shillings and sixpence (12½ p).

tongs (n) - a device used for picking up objects consisting of two long pieces free
at one end and pressed together at the other end.
.
frail (adj.) - weak and delicate.
startled (v) - felt sudden shock or alarm.
scuffle (v) - a sudden short fight.

ADDITIONAL
horrified – shocked stipulated – specified
took to bed – stayed in bed hesitant – unwilling
Prose

indeed – certainly delirious – excited


gloomy – dark persuade – influence,
convince
gaunt – lean plead – request, entreat
flushed – red-faced
twitched – trembled frail – weak
listless – inactive mere – bare, sheer
contagious –
spreadable startled –
frightened
UNIT 7

coincidence – fluke, chance


symptoms – indicators evidence – proof
ignorant – unaware scuffle – wrestle
groan – moan
recent – latest
dejection – unhappiness
206
ENGLISH

EXERCISE WITH ANSWER

Choose the appropriate Synonym of the word underlined : 10


1. I told him I could not stand it anymore and would get a doctor.
a) stay b) wait c) talk d) inform Ans: b)

2. I was horrified for I had not heard about his illness before.
The Dying Detective

a) shocked b) unhappy c) joyful d) neglecting Ans: a)

3. He took to bed on Wednesday afternoon and has never moved since


a) carried bed b) broke bed c) stayed in bed d) moved bed Ans: c)

4. He was indeed a sad sight.


a) doubted b) careful c) strong d) certainly Ans: d)

5. In the dim light of a foggy November day, the sick-room was a gloomy spot.
a) dark b) bright c) short d) known Ans: a)

6. But it was the gaunt face staring form the bed that brought chill to my heart.
a) strong b) fat c) lean d) talkative Ans: c)

nd twitched all 7.
the time.
a) red-faced b) whitish c) clean d) dirty Ans: a)

8. His eyes had the brightness of fever, his cheeks were flushed, and his hand twitched all
the time.
a) strong b) trembled c) raised d) lowered Ans: b)

9. He lay listless.
a) active b) noisy c) inactive d) deep Ans: c)

Prose
10. It is deadly and contagious.
a) controllable b) calm c) slight d) spreadable Ans: d)

EXERCISE for SELF EVALUATION

Choose the appropriate Synonym of the word underlined :


UNIT 7

1. I will examine your symptoms and treat you.


a) indicators b) strength c) weakness d) sensitiveness Ans:

2. How ignorant you are! Watson!” he said with a groan.


a) great b) foolish c) unaware d) knowledgeable Ans:

3. How ignorant you are! Watson!” he said with a groan.


a) happiness b) moan c) fever d) understanding Ans: 207
ENGLISH

4. I have learnt that much during my recent researches.


a) important b) old c) powerful d) latest Ans:

5. Then I sat in silent dejection until the stipulated time had passed.
10 a) unhappiness b) happiness c) words d) situation Ans:

6. Then I sat in silent dejection until the stipulated time had passed.
a) known b) unknown c) specified d) wrong Ans:

7. I was hesitant to leave him now.


The Dying Detective

a) sure b) clear c) possible d) unwilling Ans:

8. He was delirious.
a) excited b) weak c) patient d) calm Ans:

9. I hope you will be able to persuade him to come.


a) shout at b) influence c) shame d) walk Ans:

10. You must tell him that I’m dying – plead with him, Watson.
a) compel b) order c) request d) push Ans:

11. I saw a frail man with bald head sitting.


a) strong b) handsome c) clean d) weak Ans:

a) bare b) big c) great d) important Ans:

13. The little man was startled.


a) cool b) frightened c) calm d) crying Ans:

14. What a coincidence indeed!


a) phrase b) move c) fluke d) game Ans:

15. The last piece of evidence!


a) chance b) means c) ways d) proof Ans:
Prose

16. There was a sudden rush and scuffle.


a) wrestle b) noise c) running d) jumping Ans:

ANTONYMS
dim × bright hesitant × willing
gaunt × fat persuade × dissuade
UNIT 7

chill × heat
frail × strong
listless × active
whispered × shouted
confidence × uncertainty
sharp × blunt
ignorant × knowledgeable natural × artificial
unable × able
208 dreadful × comforting
ENGLISH

EXERCISE WITH ANSWER

Choose the appropriate Antonym of the word underlined. 10


1. In the dim light of a foggy November day, the sick-room was a gloomy spot.
a) dark b) small c) slight d) bright Ans: d)

2. But it was the gaunt face staring form the bed that brought chill to my heart.

The Dying Detective


a) fat b) lean c) clean d) part Ans: a)

3. But it was the gaunt face staring form the bed that brought chill to my heart.
a) cold b) heat c) water d) gas Ans: b)

4. He lay listless.
a) inactive b) bold c) active d) speaking Ans: c)

5. “Let me at least have someone in whom I have confidence.”


a) uncertainty b) certainty c) boldness d) sadness Ans: a)

EXERCISE for SELF EVALUATION

Choose the appropriate Antonym of the word underlined.

1. How ignorant you are!


a) foolish b) clear c) weak d) knowledgeable Ans:

2. Unable to settle down to reading, I walked slowly round and round.


a) able b) unknown c) unclear d) unfair Ans:

3. As I held it in my hand to examine it, I heard a dreadful cry.


a) frightened b) comforting c) clear d) loud Ans:

Prose
4. I was hesitant to leave him now.
a) willing b) unwilling c) disheartening d) irregular Ans:

5. I hope you will be able to persuade him to come.


a) follow b) request c) plead d) dissuade Ans:

6. I saw a frail man with bald head sitting.


a) weak b) thin c) strong d) neat Ans:
UNIT 7

7. “Is that you Mr. Smith?” Holmes whispered.


a) murmured b) shouted c) sighed d) suggested Ans:

8. There was a sharp spring inside it.


a) acute b) pointed c) blunt d) thin Ans:

9. “Turn up the gas, Smith,” said Holmes in his natural voice.


a) artificial b) real c) clear d) unclear Ans: 209
ENGLISH

10. I was advancing towards him.


a) progressing b) moving c) retreating d) walking Ans:

10 TEXTUAL QUESTIONS and ANSWERS

a. How did Watson feel when he heard of Holme’s illness? B.P.No. 189

Watson felt horrified when he heard of Holmes’ illness.

b. Why didn’t the landlady call the doctor?


The Dying Detective

Holmes did not allow the landlady to call a doctor. She did not want to disobey Holmes.

c. What was the condition of Holmes when Watson saw him?  B.P.No. 190

Holmes’ face was thin and his eyes were shining with fever. His cheeks were flushed, and his hand
twitched all the time. He lay inactive.

d. What according to Holmes was the disease he was suffering from?


According to Holmes he was suffering from Tarpaunli fever or the black Formosa plague.

e. Who did Watson see when he entered the room? B.P.No. 192

Watson saw the gaunt face of Holmes, his fevenish eyes and flushed cheeks.

Holmes asked Watson to place the ivory box on the table within his reach and slide the lid a bit with
tongs. He further asked him to put the tongs on the table.

g. Why did Holmes plead with Smith? B.P.No. 192

Holmes pleaded with Smith to cure him of the strange disease because he had a knowledge of the
Eastern diseases. Moreover, he was the only one in London who could help him.

h. Who was responsible for Victor Savage’s death? What was the evidence for it?
Smith was responsible for Victor Savage’s death. His self-proclamation was the evidence for it.
Prose

i. What explanation did Holmes give for speaking rudely to Watson? B.P.No. 193

Holmes asked excuse from Watson for ill-treating him. He said that he was rude with him just to get
Smith there and he did not want Watson to know that he was not ill.

j. How was Holmes able to look sick?


Holmes did not eat for three days and he did some make-ups to look sick.

B.P.No. 194
UNIT 7

A. Answer the following questions in one or two sentences. 

1. Who was Mrs. Hudson? Why was she worried?


Mrs. Hudson was the landlady of Holmes. She was worried because Holmes was very ill.

2. Why didn’t Holmes let Watson examine him?


Actually Holmes was not sick; he pretended to be sick to solve a murder case. So he did not let Watson
210
examine him.
ENGLISH

3. Why did Holmes warn Watson against touching his things? What was Watson’s reaction?
Watson touched the ivory box. It had a sharp needle that would pass dangerous disease. So Holmes
was against Watson touching his things.

4. What did Watson find on the table near the mantle-piece? 10


Watson found an ivory box on the table near the mantle-piece.

5. Who is Mr. Culverton Smith?


Mr. Culverton Smith is a planter. He was the murderer of his nephew Victor Savage.

The Dying Detective


6. What did Holmes ask Watson to do before leaving his room?
Holmes asked Watson to keep the ivory box on the table within his reach and slide the lid a bit before
leaving his room.

7. What instructions did Holmes give Watson to get Mr. Smith?


Holmes asked Watson to persuade Mr. Smith to come alone. He wanted Watson to come earlier than
Mr. Smith.

8. Why did Holmes want Smith to treat him?


Holmes wanted Smith to talk about the disease and proclaim he had killed his nephew Savage. So he
wanted Smith to treat him.

9. How did according to Smith Holmes get the disease?


box.

10. Who arrested Smith? What were the charges against him?
Inspector Morton arrested Smith. Smith was the murderer of his nephew Savage and he attempted to
murder Holmes.

ADDITIONAL

Answer the following questions in one or two sentences.

Prose
1. What was the case Holmes had been working?
Holmes had been working on a case down at Rotherhithe near the river. It related to Mr. Culvertion
Smith.

2. What did Holmes say about Watson in relation to treatment of his strange disease?
Holmes said that Watson was a general practitioner, not a specialist of the disease he had been
suffering from.
UNIT 7

3. Whom did Dr. Watson like to bring to specially treat Holmes’ disease?
Dr. Watson liked to brings sir Japer Meck or Penrose Fisher or Dr. Ainstree or any other best man in
London.

4. What did Watson tell Smith about Holmes?


Watson told Smith that Holmes was ill and he had a high opinion of Smith and thought he was the only
man in London who could help him. 211
ENGLISH

5. Why had Smith plotted to kill Holmes?


Smith came to understand that Holmes had known too much about Victor’s death. He feared that he
could be charged with murdering his nephew. In order to avoid imprisonment, he had plotted to kill

10 Holmes.

B. Answer the following questions in a paragraph of about 100-150 words.


1. How did Holmes trap Mr. Culverton Smith to confess the murder?

Prose The Dying Detective


Author Arthur Conan Doyle
The Dying Detective

Theme Deceiver is captured by Deception

Deceits’ favourite role is playing the victim.

Holmes pretended that he was affected by a sever disease. He did not allow Watson to treat
him. At six o’clock, Holmes told Watson to turn the gaslight on, but only half-full. He then instructed
Watson to bring Mr Culverton Smith of 13 Lower Burke Street to see Holmes, but to make sure that
Watson returned to Baker Street before Smith arrived. Watson went to Smith’s address. Although
Smith refused to see anyone, Watson forced his way in. Once Watson explained his errand on behalf
of Sherlock Holmes, Smith agreed to come to Baker Street within half an hour. Believing that they
were alone, Smith was frank with Holmes. It soon emerged, to Watson’s horror, that Holmes had been
sickened by the same illness that killed Smith’s nephew Victor. Smith then saw the little ivory box,
d with the illness.
Smith removed it to avoid the evidence of his crime. He then clearly proclaimed that he had killed
Savage and he tried to kill Holmes. He set a fool-proof trap to make Smith come out with the truth by
feigning illness and pleading cure.

‘He wouldn’t have it, sir. I did not dare to disobey him.”

2. How did Watson help his friend to arrest the criminal?

Prose The Dying Detective


Prose

Author Arthur Conan Doyle


Theme Deceiver is captured by Deception

One of the methods of manipulation is to inoculate individuals


with the bourgeois appetite for personal success.

Watson was shocked to know that Holmes was ill with a dangerous disease. He went to his
house to treat him but Holmes did not let him do it. He looked around the house and saw an ivory box.
When he touched it Holmes asked him not to touch his things. Holmes asked Watson to go to Smith
UNIT 7

and persuade him to come to Holmes. Smith arrived after Watson had returned. Smith thought that
he was alone with Holmes. He was sure that Holmes would die because of the disease he got from the
ivory box which had been sent by him to Holmes. Watson was hiding in the next room. Smith revealed
the truth that he had killed Savage and he was responsible for the trouble of Holmes. By the time he
was arrested. The whole incident was possible only with the help of Watson.

212 Then I sat in silent dejection until the stipulated time had passed.
ENGLISH

ADDITIONAL PARAGRAPH

1. Narrate the story of ‘The Dying Detective’ 10


Prose The Dying Detective
Author Arthur Conan Doyle
Theme Deceiver is captured by Deception

The Dying Detective


Appear weak when you are strong, and strong when you are weak.

Mrs. Hudson, the land lady of Holmes met Watson and told him about Holmes, who was
apparently dying of a rare tropical disease, Tapanuli fever, contracted while he was on a case. Watson
was shocked, not having heard about his friend’s illness. Mrs. Hudson said that Holmes had neither
eaten nor drunk anything in three days. Although Watson wished to examine Holmes himself or sent
for a specialist, Holmes demanded Watson to wait. While Watson waited, he examined several objects
in Holmes’s room. Holmes grew angry when Watson touched items explaining that he did not like his
things touched. He instructed Watson to bring Mr. Culverton Smith to see Holmes, but to make sure
that Watson returned to Holmes before Smith arrived. Watson went to Smith’s address. Once Watson
explained his errand on behalf of Sherlock Holmes, Smith’s attitude changed drastically. Smith agreed
self-proclaimed
that he was the murderer of Savage. Holmes explained that his illness was feigned to induce Smith to
confess to his nephew’s murder.

Three days, fasting and the makeup did the trick.

B.P.No. 194
VOCABULARY

Prose
Homophones

Homophones are words that sound the same but have different meaning and spellings. The text has
many homophones such as : see-sea, hear-here, knew-new.

C. Complete the following sentences by choosing the correct options given.


1. Niteesh bought a new (knew/new) cricket bat.
2. The shepherd heard (herd/heard) the cry of his sheep.
UNIT 7

3. Lakshmi completed her baking course (course/coarse) successfully.


4. Priya has broken her fore (four/fore) limbs.
5. Leaders of the world must work towards the peace (peace/piece) of human race.

213
ENGLISH

Commonly Confused words

D. Complete the tabular column by finding the meaning of both the words given in the
boxes. Use them in sentences of your own.
10
Word Meaning Sentence
pocket (n) a small bag sewn into or on clothing to Santa filled his pocket with candies.
keep carry small things
packet (n) a paper or cardboard container, Maheswari carried a packet of ribbons.
The Dying Detective

typically one in which goods are sold


fond (adj.) having an affection or liking for Puppies are fond of soft balls.
found(v) having been discovered by chance or Rosalin found a 100 rupee note on her
unexpectedly way back home.
lost (v) to stop having something or some People lost confidence in the
quality government.
last (adj.) most recent or nearest to the present My friend was working in a bank when
time I met him last.
paused (v) to stop speaking or doing something She paused for a moment.
for a short time before starting again
passed (v) to come up to a particular place or We passed by a group of students near
person or object and go past them the hall.
pitcher (n) a container for holding and pouring a She carried the water in a pitcher.
liquid
picture (n) shapes lines etc. painted or drawn on I like the picture of the flowers.
a surface showing what something or
someone looks like

B.P.No. 196
LISTENING
Prose

E. Listen to the story and answer the question given below.

1. Where does this story take place?


a. in a bakery b. at the police station
c. in Ms. Gervis’ house d. in Ms. Gervis’ apartment Ans: (d)

2. Near the beginning of the story, “Ms. Gervis’ eyes are full of tears. Her hands are shaking.”
How does Ms. Gervis probably feel?
a. She is upset b. She is tired
UNIT 7

c. She is hungry d. She is confused Ans: (a)

3. What makes the detective sure that the robber did not come through the windows?
a. The windows are locked.
b. The windows face the police station.
c. The windows have not been used in months.
214 d. The windows are too small for a person to fit through. Ans: (c)
ENGLISH

4. What else was stolen from the apartment?


a. crystal b. jewellery c. money d. nothing Ans: (d)

5. “And the robber definitely did not use the front door.” Which is the best way to rewrite this
sentence? 10
a. “And the robber may not have used the front door.”
b. “And the robber probably did not use the front door.”
c. “And the robber was not able to use the front door.”
d. “And the robber certainly did not use the front door.” Ans: (d)

The Dying Detective


6. What does Ms. Gervis do with her cakes?
a. She eats them. b. She sells them.
c. She hides them. d. She gives them away. Ans: (d)

7. What does the detective seem to think will happen if he solves the mystery?
a. Ms. Gervis will start baking cakes again
b. Ms. Gervis will bake him extra cakes
c. Ms. Gervis will give him her secret recipe
d. Ms. Gervis will give him money and jewels Ans: (a)

8. Do you like mysteries? What is your favorite kind of story? Explain.


Yes I like mysteries. My favourite story is detective stories. I like Sherlock Holmes’ stories very much.
e story we cannot
keep it without completing it. Our mind will be thinking about what will happen next.

B.P.No. 197
SPEAKING

REVIEW
A review is a critical assessment of a book, play, film, an event, etc. published in a newspaper or

Prose
magazine.

F. Exercise

1. Present the review of a movie that you have watched recently.

The African Queen

The African Queen (1951) is the uncomplicated tale of two companions with mismatched,
“opposites attract” personalities who develop an implausible love affair as they travel together
UNIT 7

downriver in Africa around the start of World War I. This quixotic film by director John Huston, based
on the 1935 novel of the same name by C. S. Forester, is one of the classics of Hollywood adventure
filmmaking, with comedy and romance besides. It was the first colour film for the two leads and for
director Huston.The acting of the two principal actors - Humphrey Bogart and Katharine Hepburn - is
some of the strongest ever registered on film, although this was their first and only pairing together.
They portray an unshaven, drinking and smoking captain of a cranky tramp steamer, and a prissy and
proper, but imperious and unorthodox WWI-era African missionary spinster. 215
ENGLISH

2. Give the review of a book that has interested you a lot.

If I Never Forever Endeavor

10 This book is about a bird which didn’t yet know how to fly. The bird has to decide if it will try to
fly, but it is not sure if it wants to. The bird thinks, “If I never forever endeavor” then I won’t ever learn.
On one wing, he worries he might fail and on the other wing he thinks of how he may succeed. He
worries that if he tries, he may get lost in the world. That makes him want to stay in his nest where he’s
safe. I think this book would help children to learn that trying new things can be scary, but sometimes
when we try, we can find things that make us happy too. And this book will help others know that
The Dying Detective

mistakes are okay and part of learning.

3. Review an event which your school has hosted recently.

Twenty-first March was the day when my school celebrated its annual day. But the preparations
began almost two to three weeks in advance with lots of rehearsals. The whole school became suddenly
much more active. Since the weather also became pleasant, our enjoyment increased tremendously.
The function began at 5 p.m. as per schedule. First of all our Principal welcomed the Chief Guest,
Minister for Sports and Youths, Government of India, and detailed our school’s achievements in studies
and other activities. Then the Chief Guest gave a short but very hilarious speech full of anecdotes of
his own school life. The programmes began with “Saraswati Vandana”, sung by the choir of our school.
It was followed by a short skit and the play I was acting in was presented. Although I had butterflies in
my stomach when I made entry on the stage, I did my role well. I was awarded for it also. Afterwards
l Anthem. It was a
well-organized show and my parents also admired it. I cannot forget it.

B.P.No. 197
READING

Read the story carefully and answer the questions asked below.
A Mystery Case

For a man of ease, John Mathew kept an arduous schedule. On Wednesdays, for example,
Prose

he was awakened at 9.00 and served breakfast in bed by Emanuel, his chef. Next came a quick fitness
session with Basky, his personal trainer. Then, at 10.30, John Mathew answered his mail, returned
phone calls and rearranged his social calendar helped by Louise, his secretary. At noon, John Mathew
drove his Jaguar to the station and took a commuter train into Guindy for his weekly lunch with Lalli
and Lolly, his two oldest and dearest friends. Then, on to a little shopping. The 4:05 nonstop would
bring him back to Tambaram. As John Mathew drove up to the house at 5:00, Basky would have
already set up the massage table and warmed the scented oils for a soothing herbal wrap. It was a
gruelling life but John seemed to thrive on it. On this Wednesday, however, there was an unexpected
UNIT 7

change of plans. Today John’s shopping errand involved taking his diamond bracelet into the jeweller’s
for cleaning. He threw the expensive jewel into his purse and proceeded on to lunch.

As John waved his friends good-bye and exited the restaurant, he sensed he was being
followed. The feeling continued until he reached Tenth Avenue. Then, as he joined the throng of
shoppers, John felt a hug. Within a split-second, a man riding pillion on a bike rode past him, grabbing
216 his purse. He couldn’t guess who the culprit was.
ENGLISH

G. Match the following.


1. A man of ease – a) Emanuel
2. John’s trainer – b) Lalli and Lolly


3. Mathew’s secretary
4. John’s chef




c)
d)
John Mathew
Louise
10
5. Mathew’s friends – e) Basky Ans: 1-c 2-e 3-d 4-a 5-b

H. State whether the given statements are true or false. If false correct the statements.
1. Mathew is a very busy man. Ans: True

The Dying Detective


2. He woke up very late in the morning. Ans: True
3. He always had lunch with his family. Ans: False
He had weekly lunch with his friends Lalli and Lolly.
4. He exercised with Louise every day. Ans: False
He exercised with Basky.
5. He preferred handling mail by himself. Ans: True

B.P.No. 198
WRITING

Pamphlet
• A Pamphlet is a small booklet or leaflet containing information or arguments about a single

• They are helpful in presenting information in a more attractive way and also easily accessible
and economical to distribute.
• They are generally used for describing the product or instructions, commercial information,
promotion of events or promoting tourism.

I. Create a pamphlet for the following:


1. Make a pamphlet on ‘Dengue Awareness’ (Focus on its causes, preventions, symptoms
and precautions).

Prose
UNIT 7

217
ENGLISH

2. Make an attractive pamphlet for your school Fair organised for raising funds for (any)
relief (Specify the date, time, types of stalls and the reasons for the fair).

10 ANB School, Erode.


Food Fair on 12.07.2020
For fund raising for
FLOOD RELIEF
 All kinds of food will be available.
both veg and non-veg.
The Dying Detective

 Eat and donate for the flood relief.

3. Make a pamphlet on the latest gadgets (Mention the variety of models, uses, need and
availability).

Letter of Enquiry
A letter of enquiry is a formal letter, written to get more details / information about something. In
Prose

this letter the word limit should not exceed 200 words. It is used to enquire and get details to purchase
an item, to know about a course for study, a place for a trip, etc It must include sender’s details.

J. Write a letter of enquiry for the following.

1. You are a librarian in a newly established school. Write a letter to the book dealer inquiring
about the list of newly arrived English children’s story books and various subject books
relevant to 10-14 age groups.
UNIT 7

S. Manohar,
Librarian,
SST Higher Secondary School,
27, Amman Street,
Namakkal.

218 June 22, 2020


ENGLISH

To
The Manager,
Higginbotham,
Chennai. 10
Sir,
Subject: Availability of children story books
I am the new librarian of SST Higher Secondary School, Namakkal. Our library needs some
good reading books for children. I would like to know the titles and the price of children’s story books
The Dying Detective

available with you. Will you please send the list of the books so that I will be able to choose the books
for the library.

Thanking you,

Yours faithfully,
S. Manohar.

2. Venkat hails from a remote village of Kancheepuram District, Tamil Nadu who aspires to
become an IAS officer. Currently, he is in class X. He notices an advertisement on free
classes for the IAS aspirants by a trust in a newspaper. He writes a letter to the coordinator
of the trust inquiring for further details.
M. Kumaran,
35 – Park Avenue,
Coimbatore – 8.

Aug 12, 2020


To
The Coordinator,
Nehru Coaching Centre,
Coimbatore.

Prose
Sir,
Subject: Enquiring details about IAS coaching.

I am Kumaran studying Std. 10. I am interested to become an IAS officer. I know I have
to prepare well under the guidance of coaching centres like you. I saw the ad saying that you are giving
free coaching to the IAS aspirants. I would like to know some more details about this. I will be grateful
to you if you send the details regarding the following.
1. Eligibility Criteria
UNIT 7

2. Duration of Classes
3. Weekend classes
4. Separate classes for boys and girls

Thanking you,

Yours faithfully,
Kumaran. 219
ENGLISH

3. Write a letter to the head of the BSNL office enquiry regarding about the internet broadband
scheme launched recently.
T. Rajavinayagam,
10 22, Nadesan Street,
Amman Palayam,
Tirunelveli – 2.

Aug 2, 2020
The Dying Detective

The Manager,
BSNL,
Tirunelveli.

Sir,
Subject: Detail about the internet broadband scheme
I have been using the BSNL broadband for nearly five years. I want to continue to get
your service. From the newspapers, I have come to know that you have recently launched some new
scheme with the broadband internet. Will you please give detail about the schemes launched recently?

Thanking you,

Rajavinayagam

B.P.No. 200
GRAMMAR

A. Transform the following sentences as instructed.

1. On seeing the teacher, the children stood up. (into Complex)


When the children saw the teacher they stood up.
Prose

2. At the age of six, Varsha started learning music. (into Complex)


When Varsha was six she started learning music.

3. As Varun is a voracious reader, he buys a lot of books. (into Simple)


Being a voracious reader, Varun buys a lot of books.

4. Walk carefully lest you will fall down. (into Complex)


Unless you walk carefully you will fall down.
UNIT 7

5. Besides being a dancer, she is a singer. (into Compound)


She is not only a dancer but also a singer.

6. He is sick but he attends the rehearsal. (into Simple)


Inspite of being sick, he attends the rehearsal.
220
ENGLISH

7. If Meena reads more, she will become proficient in the language. (into Compound)
Meena should read more and she will become proficient in the language.

8. He confessed that he was guilty. (into Simple)


He confessed his guilt.
10
9. The boy could not attend the special classes due to his mother’s illness. (into Compound)
The boy’s mother was ill so he could not attend the special classes.

10. He followed my suggestion. (into Complex)

The Dying Detective


He followed what I suggested.

B. Combine the pairs of sentences below into simple, complex and compound.

1. Radha was ill. She was not hospitalised.


In spite of being ill Radha was not hospitalised. (Simple)
Radha was ill but she was not hospitalised. (Compound)
Though Radha was ill she was not hospitalised. (Complex)

2. The students were intelligent. They could answer the questions correctly.
Being intelligent, the students could answer the questions correctly. (Simple)
The students were intelligent and so they could answer the questions correctly. (Compound)
x)

3. I must get a visa. I can travel abroad.


I must get a visa to travel abroad. (Simple)
I have to travel abroad so I must get a visa. (Compound)
If I get a visa, I can travel abroad. (Complex)

4. I saw a tiger. I was wounded.


I saw a wounded tiger. (Simple)
I saw a tiger and it was wounded. (Compound)

Prose
I saw a tiger which was wounded. (Complex)

5. There was a bandh. The shops remained closed.


Because of bandh, the shops remained closed. (Simple)
There was a bandh so the shops remained closed. (Compound)
The shops remained closed because there was a bandh. (Complex)
UNIT 7

221
ENGLISH

UNIT

7
POEM
10
The House on Elm Street
- Nadia Bush
The House on Elm Street

SUMMARY
Nadia Bush in her poem ‘The House on Elm Tree’ describes a mysterious house. Nobody knows
what happens inside the house. It is still there but still people do not know what happens inside the
house. The house has plenty of space inside and it is a very big house. But nobody lives there; it is bare.
At night the house seems to have some life. Some light comes and goes. The poet is often tempted to
go inside the house just to see what is inside. But fear never allows her to take any step towards that.
Every day the poet drives past the house. In summer the house seems to be little brighter. It is always
in her mind; it never leaves her mind.
Near the house there is a tree. It never has any leaves on it. In all the seasons it looks the same.
It neither grows nor becomes small. She wonders how it is possible. Some rumors are going round
saying that the house begins to fade away. The poet does not know what is going on inside the house
and it will always remain a mystery to the poet.

A. Read the given lines and answer the questions given below.  B.P.No. 203

1. It sat alone.
What happened there is still today unknown.
It is a very mysterious place,
And inside you can tell it has a ton of space,
But at the same time it is bare to the bone.
Poem

a. What does ‘It’ refer to?


‘It’ refers to the mysterious house.

b. Pick out the line that indicates the size of the house.
‘And inside you can tell it has a ton of space.’

2. I drive past the house almost every day.


The house seems to be a bit brighter.
On this warm summer day in May.
UNIT 7

It plays with your mind.


a. Who does ‘I’ refer to?
‘I’ refers to the poet.

b. Pick out the alliterated words in the 2nd line.


222 be – bit – brighter.
ENGLISH

3. It never grows leaves,


Not in the winter, spring, summer or fall.
It just sits there never getting small or ever growing tall
a. What does ‘it’ refer to? 10
‘It’ refers to the tree.

b. In what way is the tree a mystery?


The tree does not have any leaves. It never grows nor does it become small.

The House on Elm Street


4. Rumors are constantly being made,
And each day the house just begins to fade.
What happened inside that house?
a. Does the house remain the same every day?
No, the house seems to begin to fade.

b. Why does the poet consider the house to be a mystery?


Nobody knows what happens inside the house. So the poet considers the house a mystery.

5. What happened inside that house?


I really don’t know
I guess it will always be a mystery
a. Does the poet know what happened in the house?

b. What is the mystery about the house?


As nobody knows what happens inside the house, it is a mystery.

ADDITIONAL

Read the given lines and answer the questions given below.

1. Beside the house sits a tree.

Poem
It never grows leaves,
Not in the winter, spring, summer or fall.
a. Do leaves grow in the tree?
No, leaves don’t grow on the tree.

b. What is near the house?


A tree is near the house.
UNIT 7

2. And each day the house just begins to fade.


What happened inside that house?
a. When does the house seem to fade?
Every day the house seems to fade.

b. Identify the words in alliteration.


happened, house. 223
ENGLISH

3. How could this be?


a. What is the figure of speech.
“Rhetorical Question”.
10 4. It just sits there, never getting small or ever growing tall.
a. What is the figure of speech mentioned here.
Paradox
The House on Elm Street

B. Answer the following in a paragraph.


1. Where is the house located? Why is it a mysterious place?

Poem The House on Elm Street


Poet Nadia Bush
Theme An enigmatic mysterious house

It sat alone
What happened there is still today unknown

Nadia Bush in her poem ‘The House on Elm Tree’ describes a mysterious house. Nobody knows
what happens inside the house. It is still there but still people do not know what happens inside the
house. The house has plenty of space inside and it is a very big house. But nobody lives there; it is
bare. At night the house seems to have some life. Some light comes and goes. In summer the house
seems to be little brighter. It is always in her mind; it never leaves her mind.

Near the house there is a tree. It never has any leaves on it. In all the seasons it looks the same.
It neither grows nor becomes small. She wonders how it is possible. Some rumors are going round
saying that the house begins to fade away. The poet does not know what is going on inside the house
and it will always remain a mystery to the poet.

Besides the house sits a tree


It never grows leaves
Poem

2. How is the mystery depicted in the poem?

Poem The House on Elm Street


Poet Nadia Bush
Theme An enigmatic mysterious house

At night the house seems to be alive


Lights flicker on and off
UNIT 7

Nadia Bush in her poem ‘The House on Elm Tree’ describes a mysterious house. Nobody knows
what happens inside the house. It is still there but still people do not know what happens inside the
house. The house has plenty of space inside and it is a very big house. But nobody lives there; it
is bare. At night the house seems to have some life. Some light comes and goes. The poet is often
tempted to go inside the house just to see what is inside. But fear never allows her to take any step
224
towards that. Every day the poet drives past the house. In summer the house seems to be little
ENGLISH

brighter. It is always in her mind; it never leaves her mind. Near the house there is a tree. It never has
any leaves on it. In all the seasons it looks the same. It neither grows nor becomes small. She wonders
how it is possible. Some rumors are going round saying that the house begins to fade away. The poet
does not know what is going on inside the house and it will always remain a mystery to the poet.
10
What happened inside that house?
I really don’t know.
I guess it will always be a mystery.

The House on Elm Street


C. Read the poem and write the rhyming words and rhyme scheme for the given stanzas.

Stanza Rhyming words Rhyme scheme


alone - unknown, bone aabba
1
place - space
day - May abacc
3
mind - kind
tree - be abcca
4
fall - tall

D. Identify the poetic lines where the following figures of speech are employed and complete

Figure of speech Meaning Lines


Synecdoche A figure of speech in which a part is made But at the same time it is bare
to represent the whole or vice versa. to the bone
e.g. “The Western wave was all a-flame.”
The “Western wave” is a synecdoche as it
refers to the sea by the name of one of its
parts i.e. wave.
Paradox A figure of speech in which a statement It just sits there, never getting
small or ever growing tall.

Poem
appears to contradict itself.
e.g. To bring peace we must war.
Be cruel to be kind.
Onomatopoeia A figure of speech wherein the word Lights flicker on and off
imitates the sound associated with the
object it refers to.
e.g. P itter patter, pitter patter Raindrops
on my pane.
UNIT 7

Rhetorical Questions A figure of speech in the form of a question How could this be?
that is asked to make a point rather than to
elicit an answer.
e.g. And what is so rare as a day in June?

225
ENGLISH

UNIT

7
SUPPLEMENTARY
10
A Dilemma
- Silas Weir Mitchell

SUMMARY
A Dilemma

Silas Weir wrote the short story ‘A Dilemma’ in which he brings out a real dilemma in the life
of a man. The author is the narrator and he talks about his uncle who had no attachment with his
family members. He was an inventor and an able mechanical engineer. He was quite rich and he had
the habit of collecting precious stones. He was a bachelor and lived alone cooking his own food. The
author’s mother told him that his uncle would never help him.
His uncle called him one week before his death. When he met his uncle he told him that he had
been living with the annuity he was getting from his fortune. He accepted that he had been working
only to gather wealth. He felt sorry that he had not helped any of his relatives. He was not very rich
and he had all the precious stones intact with him. He would officially give all the precious stones to

The author thanked him for giving the stones to him. His uncle told him that he would pay for
his uncle’s funeral. All the stones were kept in a box. Before opening the box he should read a letter
which was kept on the top of the box. He died the next week and he was handsomely buried. The
author opened the safe and found an iron box. There was a letter on the box. The letter said that the
box contained many valuable stones. At the same time it had a dynamite which would explode when
the box was opened. So he could own the stones; he could not take them out of the box.
Supplementary

He thought of many methods to open without being killed. But he could not get any idea. In his
despair he consulted some educated and knowledgeable people. He could not get help from anybody.
One doctor advised him not to think too much about that. He spent his leisure time in the library
reading about dynamite. It was absurd to have wealth and not having chance to use it. He thought of
finding out a person who would be ready to open the box. He became weak and thin. He took it to
the confession table but there too nothing good happened.
The girl Susan cancelled her engagement with him saying that he was half insane. He made his
will leaving the rubies and pearls to the Society of Preservation of Human Vivisection.

B.P.No. 210
UNIT 7

GLOSSARY
ingenious (adj.) - clever, original and inventive
mania (n) - an extensive, persistent desire, an obsession
malicious (adj.) - spiteful, intended to harm or upset someone
queer (adj.) - strange, odd
226
ENGLISH

appalled (adj.) - horrified, shocked


oddity (n) - the quality of being strange or peculiar
closet (n) - cupboard
incredible (adj.)
contrive (v)
-
-


impossible to believe
cook up, hatch a plan by deliberate use of skills
10
despotic (adj.) - tyrannical, autocratic
avarice (n) - extreme greed for wealth
jest (n) - a joke
vivisection (n) - a surgery conducted on a living organism for experimental purposes.

A. Read the given lines carefully and identify the character / speaker: B.P.No. 210

1. I suppose you think me queer. I will explain. Ans: Uncle Philip

A Dilemma
2. Don’t come back. It won’t hasten things. Ans: Uncle Philip
3. He thought it simply a cruel jest. Ans: Father confessor
4. He did not desire to do so. Ans: Professor Clinch
5. He would think it over and come back later. Ans: Tax collector

ADDITIONAL
1. I shall carry with me to the other world the satisfaction of

2. When I thanked him he grinned all over his face. Ans: Author / Narrator (Tom)
3. It was silly tale and altogether incredible. Ans: Professor Clinch
4. At last the doctor kindly warned me that I was in danger
of losing my mind. Ans: Dr. Schaff
5. I was half cracked like Uncle Philip. Ans: Susan

B. Based on your understanding of the story, answer the following briefly.

Supplementary
1. What did the uncle do as soon as he bought a stone?
When he bought a new stone, he carried it in his pocket for a month and now and then took it out and
looked at it. Then it was added to the collection in his safe at the trust company.

2. What did the uncle bequeath to the narrator?


The uncle bequeathed all the precious stones to the narrator.

3. What was the condition laid by the uncle to inherit his property?
The condition laid by the uncle to inherit his property was that the narrator should pay for his funeral.

4. Why do you think Tom happily looked forward to the expenditure for his uncle’s funeral?
UNIT 7

Tom happily looked forward to the expenditure for his uncle’s funeral because after the funeral he
would inherit the property of his uncle.

5. Write a few words about the mechanism used in the iron box.
The box was heavy and strong, about ten inches long, eight inches wide and ten inches high. One had
to be careful to open it with a key because it had a dynamite inside which might explode if the box
was shaken. 227
ENGLISH

6. What was the counsel offered to the narrator?


All the people to whom he talked about the box advised him to leave it to itself.

7. Why and when was the narrator shocked?


10 The narrator was shocked when he read the letter found on the iron box. It made it clear that if the
box was opened there was every possiblity of the dynamite getting exploded.

8. What was the doctor’s warning to Tom?


The doctor kindly warned Tom that he was in danger of losing his mind with too much thought about
his rubies.

9. Why didn’t Tom dare to assign the task of unlocking the box to someone?
A Dilemma

He thought it was not fair on his part to assign someone to open the box as he was afraid of the
dynamite getting exploded.

C. Answer the questions given below in a paragraph of 150 words.

1. Describe briefly the contents of the letter written by Tom’s uncle.

‘A Dilemma – uncle not attached with family – collected precious stones – died – box with
stones came to the author – saw a letter – explained about rubies – many rubies and
pearls – blue diamond – a necklace of blue pearls – interesting mechanism – dynamite

Title A Dilemma
Poet Silas Wier Mitchell
Theme Vain efforts to possess valuable stones

Silas Weir wrote the short story ‘A Dilemma’ in which he brings out a real dilemma in the life of
a man. The author is the narrator and he talks about his uncle who had no attachment with his family
Supplementary

members. He was an inventor and an able mechanical engineer. He was quite rich and he had the
habit of collecting precious stones. After his death Tom got the iron box which had the precious stones
from his uncle. Before opening the box he saw a letter. It explained about the rubies and diamonds.
One blue diamond was very beautiful. Apart from those there were hundreds of pearls. There was a
green pearl. There was a necklace of blue pearls which every woman would like to have. He did not
want to leave those valuable properties to charity because he hated the poor. The box contained an
interesting mechanism, which would act with certainty. In case someone unlocked it, ten ounces of
improved supersensitive dynamite would explode. One who opened it would be blown to atoms. Tom
would continue to nourish expectations which would never be fulfilled.
UNIT 7

2. Explain the efforts taken by Tom to open the iron box? Did he succeed? Why?

‘A Dilemma – uncle not attached with family – collected precious stones – died – box with
stones came to the author – opening would be dangerous – many methods – consulted
others – not much help – Dr. advised him not to think too much – free time in the library –
tried to find out the right person – no chance – became weak – Susan cancelled engagement
228 – will to the Society of Preservation of Human Vivisection
ENGLISH

Title A Dilemma
Poet Silas Wier Mitchell
Theme Vain efforts to possess valuable stones
10
Silas Weir wrote the short story ‘A Dilemma’ in which he brings out a real dilemma in the life
of a man. The author is the narrator and he talks about his uncle who had no attachment with his
family members. He was an inventor and an able mechanical engineer. He was quite rich and he had
the habit of collecting precious stones. Tom got the iron box which had the precious stones from his
uncle. He knew that opening the box was dangerous to life. He thought of many methods to open
without being killed. But he could not get any idea. In his despair he consulted some educated and
knowledgeable people. He could not get help from anybody. One doctor advised him not to think
too much about that. He spent his leisure time in the library reading about dynamite. It was absurd to

A Dilemma
have wealth and not having chance to use it. He thought of finding out a person who would be ready
to open the box. He became weak and thin. He took it to the confession table but there too nothing
good happened. The girl Susan cancelled her engagement with him saying that he was half insane. He
made his will leaving the rubies and pearls to the Society of Preservation of Human Vivisection.

ADDITIONAL PARAGRAPH
1. Narrate the story of ‘A Dilemma’ in your own words.
s – bachelor –
cooked his food – called the author – willing to give the box of precious stones – should
take care of funeral – died – spent on funeral – opened the box – an iron box – a letter
– many stones and a dynamite – could not open – tried many methods –failed – became
weak – engagement cancelled – will to the Society of Preservation of Human Vivisection

Title A Dilemma
Poet Silas Wier Mitchell

Supplementary
Theme Vain efforts to possess valuable stones

Silas Weir wrote the short story ‘A Dilemma’ in which he brings out a real dilemma in the life of
man. The author’s uncle was an inventor and an able mechanical engineer. He was quite a rich and
he had the habit of collecting precious stones. He was a bachelor and lived alone cooking his own
food. The author’s mother told him that his uncle would never help him. His uncle called him one
week before his death. When he met his uncle he told him that he had been working only to gather
wealth. He felt sorry that he had not helped any of his relatives. He was not very rich and he had all
the precious stones intact with him. He would officially give all the precious stones to the author. His
uncle told him that he would pay for his uncle’s funeral. He died the next week and he was handsomely
UNIT 7

buried. The author opened the safe and found an iron box. There was a letter on the box. The letter
said that the box contained many valuable stones. At the same time it had a dynamite which would
explode when the box was opened. So he could own the stones but he could not take them out of
the box. He thought of many methods to open the box without being killed. But he could not get any
idea. He became weak and thin. The girl Susan cancelled her engagement with him saying that he
was half insane. He made his will leaving the rubies and pearls to the Society of Preservation of Human
Vivisection. 229
ENGLISH

D. Fill in the blanks with the right option and write down the summary of the story ‘A
dilemma’.
1. The narrator was sent for, by his uncle when he was on his deathbed.

10 (on his deathbed / on his travels / in his workplace)


2. The uncle had collected precious stones.
(jewels / stones / articles)
3. His uncle announced Tom as his heir and wanted him to pay for his funeral.
(rented house / marriage / funeral)
4. Leaving an iron box for Tom, his uncle instructed him not to shake the box.
(throw / carry / shake)
5. The letter read that the box contained a sensitive dynamite.
A Dilemma

(a sensitive dynamite / jewels / money)


6. He started thinking of all possible ways to open the box without being killed.
(wounded / killed / maimed)
7. He planned to explode the box at a safe distance but dropped the plan in fear of losing the rubies.
(home / a safe distance / a waste land)
8. His consultation with Professor Clinch did not yield him any fruitful solution.
(Uncle Philip / Professor Clinch / Susan)
9. He failed in his attempts to open the box. His efforts to read about explosives led to suspicions (hopes

(name and occupation / lodgings / appearance)


10. At last, he bequeathed the box to the Society.
(his offspring / his friends / the Society)

ADDITIONAL EXERCISES
Supplementary

I. Rearrange the sentences in coherent order.


I 1. But he had no attachment with his family members.
2. Tom’s uncle was a bachelor.
3. Tom’s mother told him that his uncle would never help him.
4. He had the habit of collecting precious stones.
5. He was an inventor and mechanical engineer. Answer: 2, 5, 4, 1, 3

II 1. But Tom should spend for his funeral and then get the box containing the precious stones.
2. When his uncle died Tom spent most of his money on his uncle’s funeral.
3. He told Tom that he would give all the precious stones to him.
4. Tom accepted the deal.
UNIT 7

5. One day Tom’s uncle called him. Answer: 5, 3, 1, 4, 2

III 1. Along with them it had a dynamite which would explode when the box was opened.
2. Before opening the box he should read the letter kept on the box.
3. So he could own the stones, he could not take them out of the box.
4. Tom’s uncle told him that the box containing precious stones would belong to him.
230
5. The letter said that the box contained many valuable stones. Answer: 4, 2, 5, 1, 3
ENGLISH

II. Identify the speaker / character.


1. He was a bachelor – Uncle Philip
2. You will have to pay for my funeral – Uncle Philip
3. I stood appalled, the key in my hand – The Narrator 10
4. He did not desire to do so – Professor Clinch
5. At last the doctor kindly warned me – Dr. Schaff

III. Reading comprehension.


Read the following passage and answer the questions given below:
1. He was a bachelor; lived alone, cooked his own meals, and collected precious stones, especially
rubies and pearls. From the time he made his first money he had this mania. As he grew richer, the
desire to possess rare and costly gems became stronger. When he bought a new stone, he carried

A Dilemma
it in his pocket for a month and now and then took it out and looked at it. Then it was added to the
collection in his safe at the trust company.
Questions :
1. Who is ‘he’? 4. What did he do when he bought a new
2. What kind of stones did he collect? stone?
3. What mania is referred to here? 5. What did he do with the new stone?

Answers:
1. ‘He’ is Uncle Philip
2. He collected precious stones especially rubies and pearls.

ked at it.
5. The new stone was added to the collection in his safe at the trust company.

IV. Passage for comprehension.


1. Now I repent of my wickedness to you all, and desire to live in the memory of at least one of my
family. You think I am poor and have only my annuity. You will be profitably surprised. I have never

Supplementary
parted with my precious stones; they will be yours. You are my sole heir. I shall carry with me to the
other world the satisfaction of making one man happy.
Questions :
1. What did he repent of? 4. What kind of satisfaction was he going to
2. What didn’t he part with? have?
3. Who would be his sole heir? 5. Was he a poor man?

Answers:
1. He repented of his wickedness to the narrator and his family.
2. He did not part with his precious stones.
3. The narrator.
UNIT 7

4. He would carry with him to the other world the satisfaction of making one man happy.
5. No, he was not a poor man.
2. At last I hung the key on my watch-guard; but then it occurred to me that it might be lost or
stolen. Dreading this, I hid it, fearful that someone might use it to open the box. This state of doubt
and fear lasted for weeks, until I became nervous and began to dread that some accident might
happen to that box. A burglar might come and boldly carry it away and force it open and find it was
a wicked fraud of my uncle’s. Even the rumble and vibration caused by the heavy vans in the street
231
became at last a terror.
ENGLISH

Questions :
1. What did the narrator do with the key in 3. What did he dread about?
the beginning? 4. What did he think the burglar might do?
2. Why did he hide the key? 5. What created terror to the narrator?
10 Answers :
1. In the beginning, the narrator hung the key on his watch-guard.
2. He hid the key being fearful that some one would use it to open the box.
3. He dreaded that some accident might happen to the box.
4. He thought that a burglar might come and boldly carry the box away and force it open and find
that it was a wicked fraud of his uncle’s.
5. The rumble and vibration caused by the heavy vans in the street caused terror to him.
3. Indeed, I talked of it until the library attendants, believing me a lunatic or a dynamite fi end,
A Dilemma

declined to humor me, and spoke to the police. I suspect that for a while I was “shadowed” as a
suspicious, and possibly criminal, character. I gave up the libraries, and, becoming more and more
fearful, set my precious box on a down pillow, for fear of its being shaken; for at this time even the
absurd possibility of its being disturbed by an earthquake troubled me. I tried to calculate the amount
of shake needed to explode my box.
Questions :
1. Why didn’t the library attendants humour 3. What did he do to avoid the box being shaken?
him? 4. What absurd possibility did he think of?
2. What did the narrator suspect? 5. What did he try to calculate finally?

1. The library attendants believed him a lunatic or a dynamite fiend.


2. The narrator suspected that he was shadowed as a suspicious and criminal character.
3. He set the box on a down pillow.
4. He thought of the box being disturbed by an earthquake.
5. He tried to calculate the amount of shake needed to explode the box.
4. Susan said I was half cracked like Uncle Philip, and broke off her engagement. In my despair I
Supplementary

advertised in the Journal of Science, and have had absurd schemes sent me by the dozen. At last, as
I talked too much about it, the thing became so well known that when I put the horror in a safe, in a
bank, I was promptly desired to withdraw it. I was in constant fear of burglars, and my landlady gave
me notice to leave, because no one would stay in the house with that box. I am now advised to print
my story and await advice from the ingenuity of the American mind.
Questions :
1. Who was Susan? 4. Why did the landlady ask him to vacate
2. Why did she break off her engagement? the house?
3. What was the narrator afraid of constantly? 5. What was he advised to do?
UNIT 7

Answers :
1. Susan was the narrator’s finance.
2. Susan thought that the narrator was half cracked like Uncle Philip.
3. He was afraid of burglars constantly.
4. Because no one would stay in the house with that box.
5. He was advised to print his story and await advice from the ingenuity of the American mind.

232

You might also like